Upper Limb - MCQ

अब Quizwiz के साथ अपने होमवर्क और परीक्षाओं को एस करें!

*The answer is B.* The lateral rotators of the arm include the teres minor, deltoid, and infraspinatus muscles, but the infraspinatus muscle is innervated by the suprascapular nerve.

A boy has weakness in rotating his arm laterally. Which of the following muscles are paralyzed? (A) Teres major and teres minor (B) Teres minor and deltoid (C) Infraspinatus and deltoid (D) Supraspinatus and subscapularis (E) Teres minor and infraspinatus

*The answer is E.* The scaphoid (or the older term, navicular) bone is the most commonly fractured carpal bone.

A 17-year-old boy suffered the most common of fractures of the carpal bones when he fell on his outstretched hand. Which bone would this be? A. Trapezium B. Lunate C. Pisiform D. Hamate E. Scaphoid

*The answer is C.* The radial nerve acts to extend the forearm at the elbow. This nerve is derived from all the ventral rami of the brachial plexus C5 to T1. None of the other answers includes all the ventral rami and are therefore incorrect.

A 22-year-old man is admitted to the hospital after a car collision. Radiographic examination reveals an oblique fracture of his humerus. Upon physical examination the patient is unable to extend his forearm. The damaged nerve was most likely composed of fibers from which of the following spinal levels? A. C5, C6 B. C5, C6, C7 C. C5, C6, C7, C8, T1 D. C6, C7, C8, T1 E. C7, C8, T1

*The answer is C.* The long thoracic nerve arises from the upper three ventral rami to the brachial plexus (C5 to C7) and supplies the serratus anterior, which protracts the scapula. The diaphragm is innervated by the phrenic nerve, which also arises from ventral rami (C3-C5).

A 55-year-old man is admitted to hospital after blunt trauma at the junction of his neck and shoulder on the right side. Examination reveals winging of the scapula and partial paralysis of the right side of the diaphragm. Which part of the brachial plexus has been injured? A. Cords B. Divisions C. Ventral rami D. Terminal branches E. Trunks

*The answer is A.* The supraspinatus initiates abduction of the arm during the first 15 degrees of abduction; palpation of the tendon during this phase would result in pain from a tendinopathy of the supraspinatus.

A 43-year-old man visits the outpatient clinic with a painful shoulder. Physical examination reveals a painful arc syndrome due to supraspinatus tendinopathy. Which of the following conditions are expected to be present during physical examination as the patient abducts his arm? A. Painful abduction 0 to 15 degrees B. Painful abduction 0 to 140 degrees C. Painful abduction 70 to 140 degrees D. Painful abduction 15 to 140 degrees E. Painful abduction 40 to 140 degrees

*The answer A.*

A 12-year-old boy lacerated the palmar surface of the wrist while playing with a sharp knife. The cut ends of a tendon could be seen within the wound in the exact midline of the wrist. Which tendon lies in this position in most people? A. Palmaris longus B. Flexor carpi radialis C. Abductor pollicis longus D. Flexor carpi ulnaris E. Flexor pollicis longus

*The answer is C.* The axillary nerve passes posteriorly around the surgical neck of the humerus and the radial nerve lies in the radial groove of the middle of the shaft of the humerus. The ulnar nerve passes behind the medial epicondyle and the median nerve is vulnerable to injury by supracondylar fracture of the humerus, but they lie close to or in contact with the lower portion of the humerus. The musculocutaneous is not in direct contact with the humerus.

A 12-year-old boy walks in; he fell out of a tree and fractured the upper portion of his humerus. Which of the following nerves are intimately related to the humerus and are most likely to be injured by such a fracture? (A) Axillary and musculocutaneous (B) Radial and ulnar (C) Radial and axillary (D) Median and musculocutaneous (E) Median and ulnar

*The answer is C.* The radial nerve innervates the extensor com-partments of the arm and the forearm. It supplies the triceps brachii proximal to the spiral groove, so elbow extension is intact here. It also provides sensory innervation to much of the posterior arm and forearm as well as the dorsal thumb, index, and middle fingers up to the level of the fingernails. Symptoms are described only in the distal limb due to the mid-humeral location of the lesion. The median nerve innervates flexors of the forearm and thenar muscles and provides sensory innervation to the lateral palmar hand. The ulnar nerve supplies only the flexor carpi ulnaris and the medial half of the flexor digitorum profundus in the forearm. Additionally, its sensory distribution is to both the palmar and dorsal aspects of the medial hand. It does not supply extensor muscles.

A 13-year-old boy is brought to the emergency department after losing control during a motorbike race in which he was hit by several of the other racers. Physical examination reveals several cuts and bruises. He is unable to extend the left wrist, fingers, and thumb, although he can extend the elbow. Sensation is lost in the lateral half of the dorsum of the left hand. Which of the following nerves has most likely been injured to result in these signs, and in what part of the arm is the injury located? A. Median nerve, anterior wrist B. Median nerve, arm C. Radial nerve, midhumerus D. Ulnar nerve, midlateral forearm E. Ulnar nerve, midpalmar region

*The answer is C.* Trauma both to the median and ulnar nerves at the wrist results in total clawing of the fingers. The metacarpophalangeal joints of all digits are extended by the unopposed extensors because the radial nerve is intact. All interossei and lumbricals are paralyzed because the deep branch of the ulnar nerve supplies all of the interossei; lumbricals I and II are paralyzed, for they are innervated by the median nerve; lumbricals III and IV are paralyzed, for they receive supply from the deep ulnar nerve. The interossei and lumbri- cals are responsible for extension of the interphalangeal joints. When they are paralyzed, the long flexor tendons pull the fingers into a position of flexion, completing the "claw" appearance. If the median nerve were intact, the clawing would be less noticeable in the index and long fingers because the two lumbricals would still be capable of some degree of extension of those interphalangeal joints. If the median nerve alone is injured in the carpal tunnel, there would be loss of thenar opposition but not clawing. If the median and ulnar nerves are both transected at the elbow, the hand appears totally flat because of the loss of long flexors, in addition to intrinsic paralysis.

A 15-year-old boy received a shotgun wound to the ventral surface of the upper limb. Three months after the injury the patient exhibits a complete claw hand but can extend his wrist. What is the nature of this patient's injury? A. The ulnar nerve has been severed at the wrist. B. The median nerve has been injured in the carpal tunnel. C. The median and ulnar nerves are damaged at the wrist. D. The median and ulnar nerves have been injured at the elbow region. E. The median, ulnar, and radial nerves have been injured at midhumerus.

*The answer is B.* The abductor pollicis longus and extensor pollicis brevis muscles are the occupants of the first dorsal compartment of the wrist. The extensor carpi radialis longus and brevis are in the second compartment. The extensor digitorum is in the third compartment, as is the extensor indicis. The extensor carpi ulnaris is located in the sixth dorsal compartment.

A 15-year-old girl was brought to the emergency department with a tear of the tendons in the first dorsal compartment of the wrist from a severe bite by a pit bull dog. The injured tendons in this compartment would include which of the following muscles? A. Extensor carpi radialis longus and brevis B. Abductor pollicis longus and extensor pollicis brevis C. Extensor digitorum D. Extensor indicis proprius E. Extensor carpi ulnaris

*The answer is A.* Injury to the radial nerve results in loss of wrist extension, leading to wrist drop. The median nerve innervates the pronator teres, pronator quadratus, and opponens pollicis muscles and the skin over the ventral aspect of the thumb. The ulnar nerve innervates the dorsal interosseous muscles, which act to abduct the fingers.

A 16 year-old boy fell from a motorcycle and his radial nerve was severely damaged because of a fracture of the midshaft of the humerus. Which of the following conditions would most likely result from this accident? (A) Loss of wrist extension, leading to wrist drop (B) Weakness in pronating the forearm (C) Sensory loss over the ventral aspect of the base of the thumb (D) Inability to oppose the thumb (E) Inability to abduct the fingers

*The answer is D.* The musculocutaneous nerve innervates the brachialis and biceps brachii muscles, which are the main flexors at the elbow. The biceps brachii inserts on the radius and is an important supinator. Because the musculocutaneous nerve is damaged in this case, it leads to loss of sensory perception to the lateral forearm, which is supplied by the distal continuation of the musculocutaneous nerve (known as the lateral antebrachial cutaneous nerve). The name "musculocutaneuous" indicates it is "muscular" in the arm and "cutaneous" in the forearm. Adduction and abduction of the fingers are mediated by the ulnar nerve and would not be affected in this instance. The flexor pollicis brevis muscle flexes the thumb and is mainly innervated by the recurrent branch of the median nerve. Flexion of the fingers is performed by the long flexors of the fingers and lumbrical muscles, inner- vated by the median and ulnar nerves. Sensory innervation of the medial forearm is provided by the medial antebrachial cutaneous nerve, usually a direct branch of the medial cord of the brachial plexus.

A 17-year-old boy has weakness of elbow flexion and supination of the left hand after sustaining a knife wound in that arm in a street fight. Examination in the emergency department indicates that a nerve has been severed. Which of the following conditions will also most likely be seen during physical examination? A. Inability to adduct and abduct his fingers B. Inability to flex his fingers C. Inability to flex his thumb D. Sensory loss over the lateral surface of his forearm E. Sensory loss over the medial surface of his forearm

*The answer is E.* Striking the concrete blocks with the medial side of her hand has injured the ulnar nerve in Guyon's canal. This is the triangular tunnel formed by the pisiform bone medially, the flexor retinaculum dorsally, and the deep fascia of the wrist ventrally. This injury would result in loss of sensation to the medial palm and the palmar surface of the medial one and a half digits and motor loss of the hypothenar muscles, the interossei, and the medial two lumbricals. The median nerve is not involved because the thenar muscles and lateral palmar sensations are intact. The dorsal ulnar nerve arises proximal to the wrist, thus it would not be lost. Carpal bone dislocation is unlikely. If the lunate bone were dislocated, it would not cause compression of the ulnar nerve at the wrist. There is no indication of fifth metacarpal fracture, the so-called boxer's fracture.

A 17-year-old female student of martial arts entered the emergency department with a complaint of pain in her hand. Patient history reveals that she had been breaking concrete blocks with her hand. Examination reveals that the patient has weak abduction and adduction of her fingers but has no difficulty in flexing them. The patient also has decreased sensation over the palmar surfaces of the fourth and fifth digits. Which of the following best describes the nature of her injury? A. Compression of the median nerve in the carpal tunnel B. Fracture of the triquetrum, with injury to the dorsal ulnar nerve C. Dislocation of a bone in the proximal row of the carpus D. Fracture of the shaft of the fifth metacarpal E. Injury of the ulnar nerve in Guyon's canal

*The answer is D.* In a lesion of the lower trunk of the brachial plexus, or the C8 and T1 ventral rami, there is sensory loss on the medial forearm and the medial side of hand (dorsal and ventral). The medial cord is an extension of the lower trunk. The medial cord gives origin to the medial antebrachial cutaneous nerve, which supplies the T1 dermatome of the medial side of the antebrachium. The lower lateral brachial cutaneous nerve arises from the radial nerve, C5 and C6. The musculocutaneous nerve arises from the lateral cord, ending in the lateral antebrachial cutaneous nerve, with C5 and C6 dermatome fibers. The intercostobrachial nerve is the lateral cutaneous branch of the T2 ventral primary ramus and supplies skin on the medial side of the arm. The median nerve distrib- utes C6 and C7 sensory fibers to the lateral part of the palm, thumb, index, long finger, and half of the ring finger.

A 19-year-old man fell from a cliff when he was hiking in the mountains. He broke his fall by grasping a tree branch, but he suffered injury to the C8 to T1 spinal nerve ventral rami. Sensory tests would thereafter confirm the nature of his neurologic injury by the sensory loss in the part of the limb supplied by which of the following? A. Lower lateral brachial cutaneous nerve B. Musculocutaneous nerve C. Intercostobrachial nerve D. Medial antebrachial cutaneous nerve E. Median nerve

*The answer is D.* Injury to the deep branch of the ulnar nerve results in paralysis of all interosseous muscles and the lumbrical muscles of digits 4 and 5. Extension of the metacarpophalangeal joints is intact, a function of the radial nerve. Interphalangeal extension of digits 4 and 5 is absent, due to the loss of all interosseous muscle and the lumbricals of digits 4 and 5. Some weak interphalangeal joint extension is still present in digits 2 and 3 because the lumbricals of these two fingers are innervated by the median nerve. The radial nerve and the median nerve appear to be intact in this case. If the ulnar nerve were injured in the midfore- arm region, there would be sensory loss in the palm and digits 4 and 5 and on the dorsum of the hand. The recurrent branch of the median nerve supplies the thenar muscles; it does not supply lumbricals. Moreover, paralysis of this nerve would have no effect on the interphalangeal joints.

A 19-year-old man had suffered a deep laceration to an upper limb when he stumbled and fell on a broken bottle. On examination of hand function it is observed that he is able to extend the metacarpophalangeal joints of all his fingers in the affected limb. He cannot extend the interphalangeal (IP) joints of the fourth and fifth digits, and extension of the IP joints of the second and third digits is very weak. There is no apparent sensory deficit in the hand. Which of the following nerves has most likely been injured? A. Radial nerve at the elbow B. Median nerve at the wrist C. Ulnar nerve in midforearm D. Deep branch of ulnar nerve E. Recurrent branch of the median nerve

*The answer is D.* The supraspinatus muscle is one of the four rotator cuff muscles—the other three being the infraspinatus, teres minor, and subscapularis muscles. The tendon of the supraspinatus muscle is relatively avascular and is often injured when the shoulder is dislocated. This muscle initiates abduction of the arm, and damage would impair this movement. The coracobrachialis muscle, which runs from the coracoid process to the humerus, functions in adduction and flexion of the arm. The main function of the triceps brachii muscle is to extend the elbow, and damage to its long head would not affect abduction. The pectoralis minor muscle functions as an accessory respiratory muscle and to stabilize the scapula and is not involved in abduction. The teres major muscle functions to adduct and medially rotate the arm.

A 19-year-old man is brought to the emergency department after dislocating his shoulder while playing soccer. Following reduction of the dislocation, he has pain over the dorsal region of the shoulder and cannot abduct the arm normally. An MRI of the shoulder shows a torn muscle. Which of the following muscles is most likely to have been damaged by this injury? A. Coracobrachialis B. Long head of the triceps brachii C. Pectoralis minor D. Supraspinatus E. Teres major

*The answer is E.* The pectoralis major is innervated by the lateral and medial pectoral nerves originated from the lateral and medial cords of the brachial plexus, respectively. The subscapularis, teres major, latissimus dorsi, and teres minor muscles are innervated by nerves originating from the posterior cord of the brachial plexus.

A 20-year-old man fell from the parallel bar during the Olympic trial. A neurologic examination reveals he has a lesion of the lateral cord of the brachial plexus. Which of the following muscles is most likely weakened by this injury? (A) Subscapularis (B) Teres major (C) Latissimus dorsi (D) Teres minor (E) Pectoralis major

*The answer is A.* The flexor carpi ulnaris muscle is not innervated by the radial nerve but rather by the ulnar nerve. The brachioradialis, extensor carpi radialis longus and brevis, and supinator muscles are all innervated by the radial nerve distal to the spiral groove.

A 21-year-old female softball pitcher is examined in the emergency department after she was struck in the arm by a line drive. Plain radiographic and MRI studies show soft tissue injury to the region of the spiral groove, with trauma to the radial nerve. Which of the following muscles would be intact after this injury? A. Flexor carpi ulnaris B. Extensor indicis C. Brachioradialis D. Extensor carpi radialis longus E. Supinator

*The answer is C.* A lesion of the upper trunk of the brachial plexus results in a condition called "waiter's tip hand" in which the arm tends to lie in medial rotation because of paralysis of lateral rotators and abductors of the arm. The long thoracic nerve, which arises from the root (C5-C7) of the brachial plexus, innervates the serratus anterior muscle that can elevate the arm above the horizontal. The dorsal scapular nerve, which arises from the root (C5), innervates the rhomboid major. The medial side of the arm receives cutaneous innervation from the medial brachial cutaneous nerve of the medial cord. Nerve fibers from dorsal primary rami of C5 and C6 supply the deep muscles of the back.

A 21-year-old patient has a lesion of the upper trunk of the brachial plexus (Erb-Duchenne paralysis). Which of the following is the most likely diagnosis? (A) Paralysis of the rhomboid major (B) Inability to elevate the arm above the horizontal (C) Arm tending to lie in medial rotation (D) Loss of sensation on the medial side of the arm (E) Damage to nerve fibers from dorsal primary rami of CS and C6

*The answer is C.* The latissimus dorsi adducts the arm, and the supraspinatus muscle abducts the arm. The infraspinatus and the teres minor rotate the arm laterally. The serratus anterior rotates the glenoid cavity of the scapula upward, abducts the arm, and elevates it above a horizontal position.

A 21-year-old woman walks in with her shoulder and arm injury after falling from a horseback riding. Examination indicates that she cannot adduct her arm because of paralysis of which of the following muscles? (A) Teres minor (B) Supraspinatus (C) Latissimus dorsi (D) Infraspinatus (E) Serratus anterior

*The answer is A.* The nail was fired explosively from the nail gun and then pierced the ulnar nerve near the coro- noid process of the ulna and the trochlea of the humerus. Paralysis of the medial half of the flexor digitorum profundus muscle would result (among other significant deficits), with loss of flexion of the distal interphalangeal joints of digits 4 and 5. Ulnar trauma at the wrist would not affect the interphalan- geal joints, although it would cause paralysis of interossei, hypothenar muscles, and so on. Median nerve damage proximal to the pronator teres would affect proximal interphalangeal joint flexion and distal inter- phalangeal joint flexion of digits 2 and 3 as well as thumb flexion. Median nerve injury at the wrist would cause loss of thenar muscles but not long flexors of the fingers. Trauma to spinal nerve ventral ramus C8 would affect all long finger flexors.

A 22-year-old male construction worker is admitted to the hospital after he suffers a penetrating injury to his upper limb from a nail gun. Upon physical examination, the patient is unable to flex the distal interphalangeal joints of digits 4 and 5. What is the most likely cause of his injury? A. Trauma to the ulnar nerve near the trochlea B. Trauma to the ulnar nerve at the wrist C. Median nerve damage proximal to the pronator teres D. Median nerve damage at the wrist E. Trauma to spinal nerve root C8

*The answer is B.* As the radial artery passes from the ventral surface of the wrist to the dorsum, it crosses through the anatomic snuffbox, passing over the scaphoid bone. The ulnar artery at the wrist is located on the medial side of the wrist, passing from beneath the flexor carpi ulnaris to reach Guyon's canal between the pisiform bone and the flexor retinaculum. Guyon's canal is adjacent to but not in communication with the carpal tunnel. The anterior interosseous and posterior interosseous arteries arise from the common interosseous branch of the ulnar artery and pass proximal to distal in the forearm between the radius and ulna, in the flexor and extensor compartments, respectively. The deep palmar branch of the ulnar artery passes between the two heads of the adductor pollicis to anastomose with the radial artery in the palm

A 22-year-old male football player suffered a wrist injury while falling with force on his outstretched hand. When the anatomic snuffbox is exposed in surgery, which artery is visualized crossing the fractured bone that provides a floor for this space? A. Ulnar B. Radial C. Anterior interosseous D. Posterior interosseous E. Deep palmar arch

*The answer is B.* During a breech delivery as described here, downward traction is applied to the shoulders and upper limbs as the baby is forcibly extracted from the birth canal. This exerts traction on the upper cord of the brachial plexus, often causing a traction injury from which the baby can often recover. If the ventral rami of C5 and C6 are avulsed from the spinal cord, the injury is permanent.

A 22-year-old pregnant woman was admitted urgently to the hospital after her baby had begun to appear at the introitus. The baby had presented in the breech position, and it had been necessary to exert considerable traction to complete the delivery. The newborn is shown in Fig. 6-7. Which of the following structures was most likely injured by the trauma of childbirth? A. Radial nerve B. Upper trunk of the brachial plexus C. Lower trunk of the brachial plexus D. Median, ulnar, and radial nerves E. Upper and lower trunks of the brachial plexus

*The answer is C.* The injury is at the second part of the axillary artery. The suprascapular artery is a branch of the thyrocervical trunk off the subclavian artery, proximal to the axillary artery. The subscapular artery is the major branch of the third part of the axillary artery, giving off the thoracodorsal and the circumflex scapular arteries. In this case blood would be flowing from the circumflex scapular artery in a retrograde direction into the axillary artery, supplying blood

A 22-year-old woman had suffered a severe knife wound to the upper lateral portion of her pectoral region, with entry of the knife at the deltopectoral groove. Pressure applied to the wound had prevented further profuse bleeding. In the emergency department, vascular clamps were applied to the axillary artery, proximal and distal to the site of injury, which had occurred between the second and third parts of the axillary artery. The vascular surgeon knew there was time to repair the wound of the artery because of the rich collateral pathway provided by the anastomoses between which of the following arteries? A. Transverse cervical and suprascapular B. Posterior circumflex humeral and profunda brachii C. Suprascapular and circumflex scapular D. Supreme (superior) thoracic and thoracoacromial E. Lateral thoracic and suprascapular

*The answer is D.* The location for palpation of the radial pulse is lateral to the tendon of the flexor carpi radialis, where the radial artery can be compressed against the distal radius. The radial pulse can also be felt in the anatomic snuffbox between the tendons of the extensor pollicis brevis and extensor pollicis longus muscles, where the radial artery can be compressed against the scaphoid.

A 22-year-old woman is admitted to the emergency department in an unconscious state. The nurse takes a radial pulse to determine the heart rate of the patient. This pulse is felt lateral to which tendon? A. Palmaris longus B. Flexor pollicis longus C. Flexor digitorum profundus D. Flexor carpi radialis E. Flexor digitorum superficialis

*The answer is C.* The contraction of the extensor mechanism produces extension of the distal interphalangeal joint. When it is torn from the distal phalanx, the digit is pulled into flexion by the flexor digitorum profundus muscle. If a piece of the distal phalanx is attached to the torn tendon it is an avulsion fracture. The proper palmar digital branches of the median nerve supply lumbrical muscles and carry sensation from their respective digits. Vincula longa are slender, bandlike connections from the deep flexor tendons to the phalanx that can carry blood supply to the tendons. The insertions of the flexor digitorum superficialis and profundus are on the flexor surface of the middle and distal phalanges, respectively, and act to flex the interphalangeal joints.

A 23-year-old female maid was making a bed in a hotel bedroom. As she straightened the sheet by running her right hand over the surface with her fingers extended, she caught the end of the index finger in a fold. She experienced a sudden, severe pain over the base of the terminal phalanx. Several hours later when the pain had diminished, she noted that the end of her right index finger was swollen and she could not completely extend the terminal interphalangeal joint. Which one of the following structures within the digit was most likely injured A. The proper palmar digital branch of the median nerve B. The vinculum longa C. The insertion of the tendon of the extensor digitorum onto the base of the distal phalanx D. The insertion of the flexor digitorum profundus tendon E. The insertion of the flexor digitorum superficialis tendon

*The answer is C.* The radial nerve is the most likely nerve compressed to cause these symptoms. This type of nerve palsy is often called "Saturday night palsy." One reason for this nickname is that people would supposedly fall asleep after being intoxicated on a Saturday night with their arm over the back of a chair or bench, thereby compressing the nerve in the spiral groove. The radial nerve innervates all of the extensors of the elbow, wrist, and fingers. It innervates the triceps brachii muscle but the motor branch typically comes off proximal to the site of compression, so the patient can still extend the elbow. Paralysis of the lateral cord of the brachial plexus would result in loss of the musculocutaneous nerve and the pectoral nerves, which do not mediate extension of the forearm or hand. The medial cord of the brachial plexus branches into the median nerve and ulnar nerve. Neither of these nerves innervates muscles that control extension. The median nerve innervates flexors of the forearm and the thenar muscles. The lateral and median pectoral nerves do not extend into the arm and innervate the pectoralis major and minor muscles.

A 23-year-old male medical student fell asleep in his chair with Netter's Atlas wedged into his axilla. When he awoke in the morning, he was unable to extend his wrist or fingers. Movements of the ipsilateral shoulder joint appear to be normal. Which of the following nerves was most likely compressed, producing the symptoms described? A. Lateral cord of the brachial plexus B. Medial cord of the brachial plexus C. Radial nerve D. Median nerve E. Lateral and medial pectoral nerves

*The answer is A.* The axillary nerve is a direct branch of the posterior cord and wraps around the surgical neck of the humerus to innervate the teres minor and the deltoid muscles. With this anatomic arrangement, the axillary nerve is tightly "tethered" to the proximal humerus. When the head of the humerus is dislocated, it often puts traction on the axillary nerve.

A 24-year-old female basketball player is admitted to the emergency department after an injury to her shoulder. Radiographic examination reveals a shoulder dislocation. What is the most commonly injured nerve in shoulder dislocations? A. Axillary B. Radial C. Median D. Ulnar E. Musculocutaneous

*The answer is D.* The recurrent branch of the median nerve is motor to the muscles of the thenar eminence, which is an elevation caused by the abductor pollicis brevis, flexor pollicis brevis, and opponens pollicis muscles. If the opponens pollicis is paralyzed, one cannot oppose the pad of the thumb to the pads of the other digits because this is the only muscle that can oppose the thumb by moving the first metacarpal on the trapezium. The recurrent branch does not have a cutaneous distribution. Holding a piece of paper between the fingers is a simple test of adduction of the fingers. These movements are controlled by the deep branch of the ulnar nerve, which is not injured in this patient.

A 24-year-old man is admitted with a wound to the palm of his hand. He cannot touch the pad of his index finger with his thumb but can grip a sheet of paper between all fingers and has no loss of sensation on the skin of his hand. Which of the following nerves has most likely been injured? A. Deep branch of ulnar B. Anterior interosseous C. Median D. Recurrent branch of median E. Deep branch of radial

*The answer is B.* The anatomic snuffbox is formed by the tendons of the extensor pollicis brevis, the abductor pollicis longus, and the extensor pollicis longus. The floor is formed by the scaphoid bone, and it is here that one can palpate for a possible fractured scaphoid.

A 25-year-old male athlete is admitted to the emergency department after a bad landing while performing the pole vault. Radiographic examination of his hand reveals a fractured carpal bone in the floor of the anatomic snuffbox (Fig. 6-2). Which bone has most likely been fractured? A. Triquetral B. Scaphoid C. Capitate D. Hamate E. Trapezoid

*The answer is A.* The nerve most likely affected is the radial nerve which innervates the triceps brachii muscle. The axillary, median and ulnar nerves do not innervate any muscles in the arm. The musculocutaneous nerve is innervating the coracobrachialis, biceps brachii, and brachialis muscles in the anterior compartment of the arm.

A 25-year-old man falls on a slippery trail and injures his upper limb. Inspection reveals abrasions over his arm at the area of the triceps muscle. Which of the following nerves innervates the triceps muscle? A. Radial B. Axillary C. Median D. Ulnar E. Musculocutaneous

*The answer is A.* The nerve most likely affected in the radiograph is the radial nerve as it travels in the radial groove along the humerus as it descends to the forearm. The radial nerve innervates the triceps brachii muscle (and extensors in the forearm), whereas the biceps brachii, coracobrachialis, and brachialis muscles are innervated by the musculocutaneous nerve and the deltoid muscle by the axillary nerve.

A 25-year-old man falls on a slippery trail and injures his upper limb. Inspection reveals abrasions over his arm. A radiograph shows a fracture at the radial groove of the humerus. Which of the following muscles will most likely be paralyzed? A. Triceps brachii B. Biceps brachii C. Coracobrachialis D. Brachialis E. Deltoid

*The answer is B.* The brachialis and supinator muscles form the floor of the cubital fossa. The brachioradialis and pronator teres muscles form the lateral and medial boundaries, respectively. The pronator quadratus is attached to the distal ends of the radius and the ulna.

A 27-year-old baseball player is hit on his forearm by a high-speed ball during the World Series, and the muscles that form the floor of the cubital fossa appear to be torn. Which of the following groups of muscles has lost their functions? (A) Brachioradialis and supinator (B) Brachialis and supinator (C) Pronator teres and supinator (D) Supinator and pronator quadratus (E) Brachialis and pronator teres

*The answer is C.* Fracture of the surgical neck of the humerus often injures the axillary nerve, which innervates the deltoid and teres minor muscles. Abduction of the humerus between 15 degrees and the horizontal is performed by the deltoid muscle. Lateral rotation of the humerus is mainly performed by the deltoid muscle, teres minor, and the infraspinatus. The deltoid and teres minor are both lost in this case. Fracture of the glenoid fossa would lead to drooping of the shoul- der. Fracture of the anatomic neck of the humerus will similarly lead to a drooping of the shoulder but would not necessarily affect abduction of the humerus. It is also quite unusual. Fracture of the middle third of the humerus would most likely injure the radial nerve. The ulnar nerve would be potentially compromised in a fracture of the medial epicondyle of the humerus.

A 27-year-old male painter is admitted to the hospital after falling from a ladder. Physical examination reveals that the patient is unable to abduct his arm more than 15 degrees and he cannot rotate the arm laterally. A radiographic examination reveals an oblique fracture of the humerus. He has associated sensory loss over the shoulder area. Which of the following injuries will most likely correspond to the symptoms of the physical examination? A. Fracture of the medial epicondyle B. Fracture of the glenoid fossa C. Fracture of the surgical neck of the humerus D. Fracture of the anatomic neck of the humerus E. Fracture of the middle third of the humerus

*The answer is E.* The surgeon took the distal segments of the median nerves from both forearms, mistakenly believing them to be palmaris longus tendons. Both of the structures lie in the midline of the ventral surface of the distal forearm and are often of similar appearance in color and diameter. The nerve is located deep to the tendon, when the tendon is present, but when the tendon is absent, the nerve appears to be where the tendon belongs. There is no evidence of rib fractures; even so, a fractured rib would not explain loss of sensation on the lateral portion of the palm. Lower plexus trauma (C8, T1) would result in paralysis of forearm flexor muscles and all intrinsic hand muscles and sensory loss over the medial dorsum of the hand, in addition to palmar sensory loss. Dupuytren's contracture is a flexion contracture of (usually) digits four and five from connective tissue disease in the palm. Radial nerve injury in the posterior forearm would affect metacarpophalangeal joint extension, thumb extension, and so on, not palmar disturbances.

A 27-year-old man had lost much of the soft tissue on the dorsum of his left hand in a motorcycle crash. Imaging studies show no other upper limb injuries. Because function of the left extensor carpi radialis longus and brevis tendons was lost, it was decided to replace those tendons with the palmaris longus tendons from both forearms because of their convenient location and relative unimportance. Postoperatively, it is found that sensation is absent in both hands on the lateral palm and palmar surfaces of the first three digits; there is also paralysis of thumb opposition. What is the most likely cause of the sensory deficit and motor loss in both thumbs? A. Bilateral loss of spinal nerve T1 with fractures of first rib bilaterally B. Lower plexus (lower trunk) trauma C. Dupuytren contracture D. Left radial nerve injury in the posterior compartment of the forearm E. The palmaris longus was absent bilaterally; the nerve normally beneath it looked like a tendon and was cut

*The answer is A.* The lower subscapular nerve arises from the cervical spinal nerves 5 and 6. It innervates the subscapularis and teres major muscles. The subscapularis and teres major muscles are both responsible for adducting and medially rotating the arm. A lesion of this nerve would result in weakness in these motions. The axillary nerve also arises from cervical spinal nerves 5 and 6 and innervates the deltoid and teres minor muscles. The deltoid muscle is large and covers the entire surface of the shoulder, and contributes to arm movement in any plane. The teres minor muscle is a lateral rotator and a member of the rotator cuff group of muscles. The radial nerve arises from the posterior cord of the brachial plexus. It is the largest branch, and it innervates the triceps brachii and anconeus muscles in the arm. The spinal accessory nerve is cranial nerve XI, and innervates the trapezius muscle, which elevates and depresses the scapula. The ulnar nerve arises from the medial cord of the brachial plexus and runs down the medial aspect of the arm. It innervates muscles of the forearm and hand.

A 27-year-old man was admitted to the emergency department after an automobile collision in which he suffered a fracture of the lateral border of the scapula. Six weeks after the accident, physical examination reveals weakness in medial rotation and adduction of the humerus. Which nerve was most likely injured? A. Lower subscapular B. Axillary C. Radial D. Spinal accessory E. Ulnar

*The answer is C.* The pectoralis minor inserts on the coracoid process, originates from the second to the fifth ribs, and is innervated by the medial and lateral pectoral nerves that arise from the medial and lateral cords of the brachial plexus. It depresses the shoulder and forms the anterior wall of the axilla. The pectoralis minor has no attachment on the clavicle.

A 27-year-old patient presents with an inability to draw forward and downward the scapula because of paralysis of the pectoralis minor. Which of the following would most likely be a cause of his condition? (A) Fracture of the clavicle (B) Injury to the posterior cord of the brachial plexus (C) Fracture of the coracoid process (D) Axillary nerve injury (E) Defects in the posterior wall of the axilla

*The answer is D.* Winged scapula is caused by paralysis of the serratus anterior muscle that results from damage to the long thoracic nerve, which arises from the roots of the brachial plexus (C5-C7).

A 29-year-old man comes in with a stab wound, cannot raise his arm above horizontal, and exhibits a condition known as "winged scapula." Which of the following structures of the brachial plexus would most likely be damaged? (A) Medial cord (B) Posterior cord (C) Lower trunk (D) Roots (E) Upper trunk

*The answer is A.* The patient exhibits the classic "benediction attitude" of the thumb and fingers from injury to the median nerve proximally in the forearm. The thumb is somewhat extended (radial supplied abductor and extensors unopposed); digits 2 and 3 are extended (by intact interossei); digits 4 and 5 are partially flexed (by their intact flexor digitorum profundus). A lesion of the median nerve would result in weakened flexion of the proximal interphalangeal joints of all digits (flexor digitorum superficialis muscle), loss of flexion of the interphalangeal joint of the thumb, the distal interphalangeal joints of digits 2 and 3 (flexor digitorum profundus muscle), and weakened flexion of the metacarpophalangeal joints of the second and third digits (first and second lumbricals). A lesion of both the ulnar and median nerves would cause weakness or paralysis of flexion of all of the digits. A lesion of the ulnar nerve would mostly cause weakness in flexion of the DIP of the fourth and fifth digits and would affect all of the interosseous muscles and the lumbricals of the third and fourth digits. A lesion of the radial nerve would cause weakness in extension of the wrist, thumb, and metacarpophalangeal joints

A 29-year-old woman had sustained a deep laceration in the proximal part of the forearm. After the wound is closed, the following functional deficits are observed by the neurologist on service: the first three digits are in a position of extension and cannot be flexed; digits 4 and 5 are partially flexed at the metacarpophalangeal (MCP) joints and noticeably more flexed at the distal interphalangeal joints; sensation is absent in the lateral side of the palm and the palmar surfaces of digits 1 to 3 and half of digit 4. Which of the following nerve(s) has (have) most likely been injured? A. Median nerve B. Ulnar and median nerves C. Ulnar nerve D. Radial and ulnar nerves E. Radial nerve

*The answer is D.* The subclavian vein traverses between the clavicle and first rib and is the most superficial structure to be damaged following a fracture of the clavicle. The subclavian artery runs posterior to the subclavian vein, and though it is in the appropriate location, it would likely not be damaged because of its deep anatomic position. The cephalic vein is a tributary to the axillary vein after ascending on the lateral side of the arm. Its location within the body is too superficial and lateral to the site of injury. The lateral thoracic artery is a branch from the axillary artery that runs lateral to the pectoralis minor. It courses inferior and medial from its point of origin from the axillary artery, and it does not maintain a position near the clavicle during its descent. The internal thoracic artery arises from the first part of the subclavian artery before descending deep to the costal cartilages. Its point of origin from the subclavian artery is lateral to clavicular injury. Furthermore, its course behind the costal cartilages is quite medial to the clavicular fracture.

A 29-year-old woman is examined in the emergency department after falling from her balcony. Radiographic examination reveals that she has suffered a broken clavicle, with associated internal bleeding. Which of the following vessels is most likely to be injured in clavicular fractures? A. Subclavian artery B. Cephalic vein C. Lateral thoracic artery D. Subclavian vein E. Internal thoracic artery

*The answer is A.* Because the median nerve is injured within the cubital fossa, the long flexors are paralyzed, including the flexor pollicis longus muscle. The flexor pollicis longus would not be paralyzed if the median nerve were injured at the wrist. Lateral palm sensory loss confirms median nerve injury. If only the anterior interosseous nerve were damaged, there would be no cutaneous sensory deficit. The radial nerve supplies wrist extensors, long thumb abductor, and metacarpophalangeal joint extensors. The ulnar nerve does not supply sensation to the lateral palm.

A 31-year-old female figure skater is examined in the emergency department following an injury that forced her to withdraw from competition. When her male partner missed catching her properly from an overhead position, he grasped her powerfully, but awkwardly, by the forearm. Clinical examination demonstrated a positive Ochsner test, inability to flex the distal interphalangeal joint of the index finger on clasping the hands. In addition, she is unable to flex the terminal phalanx of the thumb and has loss of sensation over the thenar half of the hand. What is the most likely nature of her injury? A. Median nerve injured within the cubital fossa B. Anterior interosseous nerve injury at the pronator teres C. Radial nerve injury at its entrance into the posterior forearm compartment D. Median nerve injury at the proximal skin crease of the wrist E. Ulnar nerve trauma halfway along the forearm

*The answer is C.* The surgical neck of the humerus is a typical site of fractures. The fracture line lies above the insertions of the pectoralis major, teres major, and latissi- mus dorsi muscles. The supraspinatus muscle abducts the proximal fragment, whereas the distal fragment is elevated and adducted. The elevation results from contraction of the deltoid, biceps brachii, and coraco- brachialis muscles. The adduction is due to the action of pectoralis major, teres major, and latissimus dorsi.

A 31-year-old male hockey player fell on his elbow and is admitted to the emergency department. Radiographic examination reveals a fracture of the surgical neck of the humerus, producing an elevation and adduction of the distal fragment. Which of the following muscles would most likely cause the adduction of the distal fragment? A. Brachialis B. Teres minor C. Pectoralis major D. Supraspinatus E. Pectoralis minor

*The answer is B.* The musculocutaneous nerve supplies the biceps brachii and brachialis muscles, which are the flexors of the forearm at the elbow. The musculocutaneous nerve continues as the lateral antebrachial cutaneous nerve, which supplies sensation to the lateral side of the forearm (with the forearm in the anatomic position). The biceps brachii muscle is the most powerful supinator muscle. Injury to this nerve would result in weakness of supination and forearm flexion and lateral forearm sensory loss. Injury to the radial nerve would result in weakened extension and a characteristic wrist drop. Injury to the median nerve causes paralysis of flexor digitorum superficialis muscle and other flexors in the forearm and results in a characteristic flattening of the thenar eminence. The lateral cord of the brachial plexus gives origin both to the musculocutaneous and lateral pectoral nerves. There is no indication of pectoral paralysis or weakness. Injury to the lateral cord can result in weakened flexion and supination in the forearm, and weakened adduction and medial rotation of the arm. The lateral cutaneous nerve of the forearm is a branch of the musculocutaneous nerve and does not supply any motor innervation. Injury to the musculocutaneous nerve alone is unusual but can follow penetrating injuries.

A 32-year-old woman is admitted to the emergency department after an automobile collision. Radiologic examination reveals multiple fractures of thehumerus. Flexion and supination of the forearm are severely weakened. She also has loss of sensation on the lateral surface of the forearm. Which of the following nerves has most likely been injured? A. Radial B. Musculocutaneous C. Median D. Lateral cord of brachial plexus E. Lateral cutaneous nerve of the forearm

*The answer is A.* The quadrangular space is bordered medially by the long head of the triceps brachii muscle, later- ally by the surgical neck of the humerus, superiorly by the teres minor and subscapularis muscles, and inferiorly by the teres major muscle. Both the axillary nerve and posterior circumflex humeral vessels tra- verse this space. The other structures listed are not contained within the quadrangular space. The cephalic vein is located in the deltopectoral triangle, and the radial nerve is located in the triangular interval.

A 35-year-old male body builder has enlarged his shoulder muscles to such a degree that the size of the quadrangular space is greatly reduced. Which of the following structures would most likely be compressed in this condition? A. Axillary nerve B. Anterior circumflex humeral artery C. Cephalic vein D. Radial nerve E. Subscapular artery

*The answer is C.* The common palmar digital branch comes off the superficial branch of the ulnar nerve and supplies the skin of the little finger and the medial side of the ring finger. The superficial branch of the radial nerve provides cutaneous innervation to the radial (lateral) dorsum of the hand and the radial two and a half digits over the proximal phalanx. The common palmar digital branch of the median nerve innervates most of the lateral aspect of the palmar hand and the dorsal aspect of the second and third finger as well as the lateral part of the fourth digit. The deep radial nerve supplies the extensor carpi radialis brevis and supinator muscles and continues as the posterior interosseous nerve. The recurrent branch of the median nerve supplies the abductor pollicis brevis, flexor pollicis brevis, and opponens pollicis muscles.

A 35-year-old man has a small but painful tumor under the nail of his little finger. Which of the following nerves would have to be anesthetized for a painless removal of the tumor? A. Superficial radial B. Common palmar digital of median C. Common palmar digital of ulnar D. Deep radial E. Recurrent branch of median

*The answer is E.* The contents of the cubital fossa from medial to lateral side are the median nerve, the brachial artery, the biceps brachii tendon, and the radial nerve. Thus, the median nerve is damaged. The radial recurrent artery ascends medial to the radial nerve.

A 35-year-old man walks in with a stab wound to the most medial side of the proximal portion of the cubital fossa. Which of the following structures would most likely be damaged? (A) Biceps brachii tendon (B) Radial nerve (C) Brachial artery (D) Radial recurrent artery (E) Median nerve

*The answer is E.* The ulnar nerve innervates the palmar interossei, which adduct the fingers. This is the movement that would maintain the paper between the fingers. The axillary nerve does not innervate muscles of the hand. The median nerve supplies the first and second lumbricals, the opponens pollicis, abductor pollicis brevis, and the flexor pollicis brevis muscles. None of these muscles would affect the ability to hold a piece of paper between the fingers. The musculocutaneous and radial nerves do not supply muscles of the hand.

A 36-year-old man is brought to the emergency department because of a deep knife wound on the medial side of his distal forearm. He is unable to hold a piece of paper between his fingers and has sensory loss on the medial side of his hand and little finger. Which nerve is most likely injured? A. Axillary B. Median C. Musculocutaneous D. Radial E. Ulnar

*The answer is E.* The lateral fragment of the clavicle is displaced downward by the pull of the deltoid muscle and gravity. The medial fragment is displaced upward by the pull of the sternocleidomastoid muscle. None of the other muscles are involved.

A 37-year-old female patient has a fracture of the clavicle. The junction of the inner and middle third of the bone exhibits overriding of the medial and lateral fragments. The arm is rotated medially, but it is not rotated laterally. The lateral portion of the fractured clavicle is displaced downward by which of the following? (A) Deltoid and trapezius muscles (B) Pectoralis major and deltoid muscles (C) Pectoralis minor muscle and gravity (D) Trapezius and pectoralis minor muscles (E) Deltoid muscle and gravity

*The answer is D.* The sternocleidomastoid muscle is attached to the superior border of the medial third of the clavicle, and the medial fragment of a fractured clavicle is displaced upward by the pull of the muscle.

A 37-year-old female patient has a fracture of the clavicle. The junction of the inner and middle third of the bone exhibits overriding of the medial and lateral fragments. The arm is rotated medially, but it is not rotated laterally. Which of the following muscles causes upward displacement of the medial fragment? (A) Pectoralis major (B) Deltoid (C) Trapezius (D) Sternocleidomastoid (E) Scalenus anterior

*The answer is D.* The supinator and biceps brachii muscles, which are innervated by the radial and musculocutaneous nerves, respectively, produce supination of the forearm. This is a question of two muscles that can supinate the forearm.

A 38-year-old homebuilder was involved in an accident and is unable to supinate his forearm. Which of the following nerves are most likely damaged? (A) Suprascapular and axillary (B) Musculocutaneous and median (C) Axillary and radial (D) Radial and musculocutaneous (E) Median and ulnar

*The answer is A.* The circumflex scapular artery passes through the triangular space after arising from the subscapular artery. It provides superficial branches to the overlying latissimus dorsi, whereas its deep portion passes into the infraspinous fossa to anastomose with the suprascapular artery. The dorsal scapular artery passes between the ventral rami of the brachial plexus and then deep to the medial border of the scapula. The transverse cervical artery arises from the thyro- cervical trunk at the root of the neck and can provide origin for a dorsal scapular branch. The lateral thoracic and thoracoacromial arteries are branches of the second part of the axillary artery and provide no supply to the latissimus dorsi.

A 41-year-old woman is scheduled for a latissimus dorsi muscle flap to cosmetically augment the site of her absent left breast after mastectomy. Part of the latissimus dorsi muscle is advanced to the anterior thoracic wall, based upon arterial supply provided in part by the artery that passes through the triangular space of the axilla. Which artery forms the vascular base of this flap? A. Circumflex scapular artery B. Dorsal scapular artery C. Transverse cervical artery D. Lateral thoracic artery E. Thoracoacromial artery

*The answer is C.* C7 is the main spinal nerve that contributes to the radial nerve and innervates the triceps brachii. Absence of the triceps reflex is usually indicative of a C7 radiculopathy or injury.

A 43-year-old man is admitted to the hospital, having suffered a whiplash injury when his compact automobile was struck from behind by a sports utility vehicle. MRI examination reveals some herniation of a disc in the cervical region. Physical examination reveals that the patient has lost elbow extension; there is absence of his triceps reflex and loss of extension of the metacarpophalangeal joints on the ipsilateral side. Which of the following spinal nerves is most likely affected? A. C5 B. C6 C. C7 D. C8 E. T1

*The answer is B.* Froment's sign is positive for ulnar nerve palsy. More specifically it tests the action of the adductor pollicis muscle. The patient is asked to hold a sheet of paper between the thumb and a flat palm. The flexor pollicis longus is innervated by the anterior interosseous branch of the median nerve. The flexor digiti minimi is innervated by the deep branch of the ulnar nerve and would not be used to hold a sheet of paper between the thumb and palm. The flexor carpi radialis is innervated by the median nerve, and the extensor indicis is innervated by the radial nerve

A 43-year-old woman visits the outpatient clinic with a neurologic problem. Diagnostically, she cannot hold a piece of paper between her thumb and the lateral side of her index finger without flexing the distal joint of her thumb. This is a positive Froment's sign, which is consistent with ulnar neuropathy. Weakness of which specific muscle causes this sign to appear? A. Flexor pollicis longus B. Adductor pollicis C. Flexor digiti minimi D. Flexor carpi radialis E. Extensor indicis

*The answer is C.* Inability to extend the metacarpophalangeal (MCP) joints. The tendons of the extensor digitorum and extensor digiti minimi muscles, innervated by the radial nerve, are responsible for extension of the MCP and, to a much lesser degree, the proximal (PIP) and distal (DIP) interphalangeal joints. Abduction and adduction of the MCP joints are functions of the interossei, all of which are innervated by the deep ulnar nerve. Extension of the PIP and DIP joints is per- formed by the lumbricals and interossei. The first two lumbricals are supplied by the median nerve; the other lumbricals and the interossei, by the deep branch of the ulnar nerve.

A 44-year-old woman is diagnosed with radial nerve palsy. When muscle function is examined at the metacarpophalangeal (MCP), proximal interphalangeal (PIP), and distal interphalangeal (DIP) joints, what findings are most likely to be present? A. Inability to abduct the digits at the MCP joint B. Inability to adduct the digits at the MCP joint C. Inability to extend the MCP joints only D. Inability to extend the MCP, PIP, and DIP joints E. Inability to extend the PIP and DIP joints

*The answer is D.* Fracture of the medial epicondyle often causes damage to the ulnar nerve due to its position in the groove behind the epicondyle. The ulnar nerve innervates one and a half muscles in the forearm, the flexor carpi ulnaris and the medial half of the flexor digitorum profundus muscles. The nerve continues on to innervate most of the muscles in the hand. The flexor digitorum superficialis is innervated by the median nerve and the biceps brachii muscle by the musculocutaneous. The radial nerve innervates both the brachioradialis and supinator muscles.

A 45-year-old man arrived at the emergency department with injuries to his left elbow after he fell in a bicycle race. Plain radiographic and magnetic resonance imaging (MRI) examinations show a fracture of the medial epicondyle and an injured ulnar nerve. Which of the following muscles will most likely be paralyzed? A. Flexor digitorum superficialis B. Biceps brachii C. Brachioradialis D. Flexor carpi ulnaris E. Supinator

*The answer is E.* In Smith's fracture, the distal fragment of the radius deviates palmarward, often displacing the lunate bone. The other listed bones are unlikely to be displaced in a palmar direction by Smith's fracture.

A 45-year-old man had fallen on his outstretched hand, resulting in Smith's fracture of the distal end of the radius. The fractured bone displaced a carpal bone in the palmar direction, resulting in nerve compression within the carpal tunnel. Which of the following carpal bones will most likely be dislocated? A. Scaphoid B. Trapezium C. Capitate D. Hamate E. Lunate

*The answer is D.* The medial antebrachial cutaneous nerve carries sensory fibers derived from the C8 and T1 levels. The lateral antebrachial cutaneous nerve is the distal continuation of the musculocutaneous nerve, carrying fibers from the C5, C6, and C7 levels. The deep branch of the ulnar nerve and the anterior interosseous nerves carry predominantly motor fibers. The sensory fibers coursing in the radial nerve are derived from the C5 to C8 levels.

A 45-year-old man is admitted to the hospital after a car crash. Radiographic examination reveals mild disc herniations of C7, C8, and T1. The patient presents with a sensory deficit of the C8 and T1 spinal nerve dermatomes. The dorsal root ganglia of C8 and T1 would contain cell bodies of sensory fibers carried by which of the following nerves? A. Medial antebrachial cutaneous nerve B. Long thoracic nerve C. Lateral antebrachial cutaneous nerve D. Deep branch of ulnar nerve E. Anterior interosseous nerve

*The answer is A.* The flexor digitorum profundus muscle is dually innervated by the ulnar nerve to the medial two fingers and the median nerve for the long and index fingers. Because of the superficial course of the ulnar nerve, it is vulnerable to laceration. Such an injury would result in an inability to flex the distal interphalangeal joints of the fourth and fifth digits because the flexor digitorum profundus muscle is the only muscle that flexes this joint. The flexor digitorum superficialis muscle is innervated by the median nerve only, and the course of this nerve runs too deep to be usually affected by lacerations. The lumbricals function to flex the MP joints and assist in extending the IP joints. The interossei adduct and abduct the fingers.

A 45-year-old man is admitted to the hospital after accidentally walking through a plate glass door in a bar while intoxicated. Physical examination shows multiple lacerations to the upper limb, with inability to flex the distal interphalangeal joints of the fourth and fifth digits. Which of the following muscles is most likely affected? A. Flexor digitorum profundus B. Flexor digitorum superficialis C. Lumbricals D. Flexor digitorum profundus and flexor digitorum superficialis E. Interossei

*The answer is C.* The spinal accessory nerve (CN XI) arises from the ventral rootlets of C1 to C4 that ascend through the foramen magnum to then exit the cranial cavity through the jugular foramen. It innervates the sternocleidomastoid and trapezius muscles, which function in head rotation and raising of the shoulders. The suprascapular nerve receives fibers from C5-6 (occasionally from C4 if the plexus is "prefixed") and innervates the supraspinatus muscle, which is responsible for the first 15 degrees of arm abduction. Erb's point of the brachial plexus is at the union of C5-6 spinal nerves. The long thoracic nerve arises from plexus routes C5, 6, and 7, and supplies the serratus anterior.

A 45-year-old woman is admitted to the hospital with neck pain. A computed tomography (CT) scan reveals a tumor on the left side of her oral cavity. The tumor and related tissues are removed and a radical neck surgical procedure is performed. Two months postoperatively the patient's left shoulder droops quite noticeably. Physical examination reveals distinct weakness in turning her head to the right and impairment of abduction of her left upper limb to the level of the shoulder. Which of the following structures was most likely injured during the radical neck surgery? A. Suprascapular nerve B. Long thoracic nerve C. Spinal accessory nerve D. The junction of spinal nerves C5 and C6 of the brachial plexus E. Radial nerve

*The answer is C.* The seventh cervical nerve makes a major contribution to the radial nerve, and this nerve is the prime mover in wrist extension. The dermatome of C7 is in the region described.

A 45-year-old woman is admitted to the hospital with neck pain. An MRI examination reveals a herniated disc in the cervical region. Physical examination reveals weakness in wrist extension and paraesthesia on the back of her arm and forearm. Which of the following spinal nerves is most likely injured? A. C5 B. C6 C. C7 D. C8 E. T1

*The answer is C.* The thenar muscles (and lumbricals I and II) are innervated by the median nerve, which runs through the carpal tunnel. The carpal tunnel is formed anteriorly by the flexor retinaculum and posteriorly by the carpal bones. Carpal tunnel syndrome is caused by a compression of the median nerve, due to reduced space in the carpal tunnel. The carpal tunnel contains the tendons of flexor pollicis longus, flexor digitorum profundus, and flexor digitorum superficialis muscles and their synovial sheaths. The dorsal interossei, lumbricals III and IV, palmar interossei, and hypothenar muscles are all innervated by the ulnar nerve.

A 48-year-old female court stenographer is admit- ted to the orthopedic clinic with symptoms of carpal tunnel syndrome, with which she has suffered with for almost a year. Which muscles most typically become weakened in this condition? A. Dorsal interossei B. Lumbricals III and IV C. Thenar D. Palmar interossei E. Hypothenar

*The answer is C.* The recurrent branch of the median nerve innervates the thenar muscles (opponens pollicis, abductor pollicis brevis, and flexor pollicis brevis) and is not responsible for any cutaneous innervation. Damage to the palmar cutaneous branches of the median nerve or to the ulnar nerve would not cause weakness of opposition of the thumb for they are principally sensory in function. The deep branch of the ulnar nerve supplies the hypothenar muscles, adductor and abductor muscles of digits 2-5, and does not innervate the abductor pollicis brevis.

A 48-year-old female piano player visited the out-patient clinic with numbness and tingling in her left hand. A diagnosis was made of nerve compression in the carpal tunnel, and the patient underwent an endoscopic nerve release. Two weeks postoperatively the patient complained of a profound weakness in the thumb, with loss of thumb opposition. The sensation to the hand, however, was unaffected. Which of the following nerves was injured during the operation? A. The first common digital branch of the median nerve B. The second common digital branch of the median nerve C. Recurrent branch of median nerve D. Deep branch of the ulnar nerve E. Anterior interosseus nerve

*The answer is C.* The anterior intercostal arteries are twelve small arteries, two in each of the upper six intercostal spaces at the upper and lower borders. The upper artery lying in each space anastomoses with the posterior intercostal arteries, whereas the lower one usually joins the collateral branch of the posterior intercostal artery. The musculophrenic artery is a terminal branch of the internal thoracic artery (also known as the internal mammary artery), and it supplies the pericardium, diaphragm, and muscles of the abdominal wall. It anastomoses with the deep circumflex iliac artery. The superior epigastric artery is the other terminal branch of the internal thoracic artery, and it supplies the diaphragm, peritoneum, and the anterior abdominal wall and anastomoses with the inferior epigastric artery. The lateral thoracic artery runs along the lateral border of the pectoralis minor muscle and supplies the pectoralis major, pectoralis minor, and serratus anterior. The thoracodorsal artery accompanies the thoracodorsal nerve in supplying the latissimus dorsi muscle and lateral thoracic wall.

A 49-year-old woman who had suffered a myocardial infarction must undergo a bypass graft procedure using the internal thoracic artery. Which vessels will most likely continue to supply blood to the anterior part of the upper intercostal spaces? A. Musculophrenic B. Superior epigastric C. Posterior intercostal D. Lateral thoracic E. Thoracodorsal

*The answer is E.* During a fall on an outstretched upper limb, the forces are conducted through the hand on up through the bones of the limb in succession. Often these bones do not fracture but rather pass the compressive forces proximally. The appendicular skeleton joins with the axial skeleton at the sternoclavicular joint. The forces are not sufficiently transferred to the sternum, causing the clavicle to absorb the force, resulting in common pediatric fracture of this sigmoidal-shaped bone.

A 5-year-old boy is admitted to the emergency department after falling from a tree. The parents are informed by the radiologist that their son's fracture is the most common fracture that occurs in children. Which of the following bones was broken? A. Humerus B. Radius C. Ulna D. Scaphoid E. Clavicle

*The answer is C.* The radial nerve descends posteriorly between the long and lateral heads of the triceps brachii muscle and passes inferolaterally on the back of the humerus between the medial and lateral heads of the triceps brachii muscle. It eventually enters the anterior com- partment and descends to enter the cubital fossa, where it divides into superficial and deep branches. The deep branch of the radial nerve winds laterally around the radius and runs between the two heads of the supinator muscle and continues as the posterior interosseous nerve, innervating extensor muscles of the forearm. Because this injury does not result in loss of sensation over the skin of the upper limb, it is likely that the superficial branch of the radial nerve is not injured. If the radial nerve were injured very proximally, the woman would not be able to extend her elbow. The branches of the radial nerve to the triceps brachii muscle arise proximal to where the nerve runs in the spiral groove. The anterior interosseous nerve arises from the median nerve and supplies the flexor digitorum profundus, flexor pollicis longus, and pronator quadratus muscles, none of which seem to be injured in this example. Injury to the median nerve causes a characteristic flattening (atrophy) of the thenar eminence.

A 52-year-old female band director suffered problems in her right arm several days after strenuous field exercises for a major athletic tournament. Examination in the orthopedic clinic reveals wrist drop and weakness of grasp but normal extension of the elbow joint. There is no loss of sensation in the affected limb. Which nerve was most likely affected? A. Ulnar B. Anterior interosseous C. Posterior interosseous D. Median E. Superficial radial

*The answer is B.* The lunate is the most commonly dislocated carpal bone because of its shape and relatively weak ligaments anteriorly. Dislocations of the scaphoid and triquetrum are relatively rare. The trapezoid and capitate bones are located in the distal row of the carpal bones.

A 54-year-old female marathon runner presents with pain in her right wrist that resulted when she fell with force on her outstretched hand. Radiographic studies indicate an anterior dislocation of a carpal bone (Fig. 6-4). Which of the following bones is most likely dislocated? A. Capitate B. Lunate C. Scaphoid D. Trapezoid E. Triquetrum

*The answer is E.* The anterior interosseous nerve is a branch of the median nerve that supplies the flexor pollicis longus, the lateral half of the flexor digitorum profundus, and the pronator quadratus muscles. If it is injured, flexion of the interphalangeal joint of the thumb will be compromised. The median nerve gives rise to the anterior interosseous nerve but is not a direct enough answer as injury to it would result in more widespread effects. The posterior interosseous nerve supplies extensors in the forearm, not flexors. The radial nerve gives rise to the posterior interosse- ous nerve and is not associated with the anterior interosseous nerve; therefore, it would not have any effect on the flexors of the forearm. The recurrent median nerve is also a branch of the median nerve but supplies the thenar eminence muscles, and its injury would result in problems with opposable motion of the thumb

A 54-year-old male cotton farmer visits the outpatient clinic because of a penetrating injury to his forearm from a baling hook. After the limb is anesthetized, the site of the wound is opened and flushed thoroughly to remove all debris. The patient is not able to oppose the tip of the thumb to the tip of the index finger, as in making the OK sign. He is able to touch the tips of the ring and little fingers to the pad of his thumb. What nerve has most likely been injured? A. Median B. Posterior interosseous C. Radial D. Recurrent median E. Anterior interosseous

*The answer is B.* The C6 spinal nerve is primarily responsible for the brachioradialis reflex. C5 and C6 are both involved in the biceps brachii reflex; C5 for motor, C6 for the sensory part of the reflex arc; C7 is the key spinal nerve in the triceps reflex.

A 54-year-old woman was found unconscious on the floor, apparently after a fall. She was admitted to the hospital, and during physical examination it was observed that she had unilateral absence of her brachioradialis reflex. Which spinal nerve is primarily responsible for this reflex in the majority of cases? A. C5 B. C6 C. C7 D. C8 E. T1

*The answer is D.* Normally the distal part of the ulna articulates only with the radius at the distal radioulnar joint at the wrist, a joint that participates in pronation and supination. The head of the ulna does not articulate with any of the carpal bones; instead, it is separated from the triquetrum and lunate bones by the triangular fibrocartilage complex between it and the radius. The pisiform articulates with the triquetrum. The carpal articulation of the radius is primarily that of the scaphoid (the old name is navicular) bone.

A 55-year-old female choreographer had been treated in the emergency department after she fell from the stage into the orchestra pit. Radiographs revealed fracture of the styloid process of the ulna. Disruption of the triangular fibrocartilage complex is suspected. With which of the following bones does the ulna normally articulate at the wrist? A. Triquetrum B. Hamate C. Radius and lunate D. Radius E. Pisiform and triquetrum

*The answer is C.* The winged scapula results from a lesion of the long thoracic nerve, which supplies the serratus anterior muscle. This muscle is responsible for rotating the scapula upward, which occurs during abduction of the arm above the horizontal. The long thoracic nerve arises from the ventral rami of C5 to C7 of the brachial plexus. The diaphragm is supplied by the phrenic nerve, which comes from the ventral rami of C3 to C5 (mnemonic: C3, 4 and 5 keep the diaphragm alive).

A 55-year-old male firefighter is admitted to the hospital after blunt trauma to his right axilla. Examination reveals winging of the scapula and partial paralysis of the right side of the diaphragm. Which of the following parts of the brachial plexus have been injured? A. Cords B. Divisions C. Ventral rami D. Terminal branches E. Trunks

*The answer is B.* The condition described in this patient is called "winging" of the scapula. "Winging" of the scapula occurs when the medial border of the scapula lifts off the chest wall when the patient pushes against resistance, such as a vertical wall. The serratus anterior muscle holds the medial border of the scapula against the chest wall and is innervated by the long thoracic nerve. The serratus anterior assists in abduction of the arm above the horizontal plane by rotating the scapula so that the glenoid fossa is directed more superiorly.

A 55-year-old man is examined in a neighborhood clinic after receiving blunt trauma to his right axilla in a fall. He has difficulty elevating the right arm above the level of his shoulder. Physical examination shows that the inferior angle of his right scapula protrudes more than the lower part of the left scapula. The right scapula protrudes far more when the patient pushes against the examiner's hand with resistance. Which of the following neural structures has most likely been injured? A. The posterior cord of the brachial plexus B. The long thoracic nerve C. The upper trunk of the brachial plexus D. The site of origin of the middle and lower subscapular nerves E. Spinal nerve ventral rami C7, C8, and T1

*The answer is C.* The superior ulnar collateral branch of the brachial artery accompanies the ulnar nerve in its path posterior to the medial epicondyle and is important in the blood supply of the nerve. The profunda brachii artery passes down the arm with the radial nerve. The radial collateral artery arises from the profunda brachii artery and anastomoses with the radial recurrent branch of the radial artery proximal to the elbow laterally. The inferior ulnar collateral artery arises from the brachial artery and accompanies the median nerve into the forearm. The anterior ulnar recurrent artery arises from the ulnar artery and anastomoses with the inferior ulnar collateral artery anterior to the elbow.

A 58-year-old convenience store operator had received a superficial bullet wound to the soft tissues on the medial side of the elbow in an attempted robbery. A major nerve was repaired at the site where it passed behind the medial epicondyle. Bleeding was stopped from an artery that accompanied the nerve in its path toward the epicondyle. Vascular repair was performed on this small artery because of its important role in supplying blood to the nerve. Which of the following arteries was most likely repaired? A. The profunda brachii artery B. The radial collateral artery C. The superior ulnar collateral artery D. The inferior ulnar collateral artery E. The anterior ulnar recurrent artery

*The answer is C.* Adduction of the fifth digit is produced by contraction of the third palmar interosseous muscle. All of the interossei are innervated by the deep branch of the ulnar nerve. Flexion of the proximal interphalangeal joint is a function of the flexor digitorum superficialis, supplied by the median nerve. Opposition of the thumb is a function of the opponens pollicis, supplied by the recurrent branch of the median nerve.

A 60-year-old male butcher accidentally slashed his wrist with his butcher's knife, partially dividing the ulnar nerve. Which of the following actions would most likely be lost as a result of this injury? A. Flexion of the proximal interphalangeal joint of the fifth digit (little finger) B. Extension of the thumb C. Adduction of the fifth digit D. Abduction of the thumb E. Opposition of the thumb

*The answer is A.* The patient has suffered injury to the radial nerve in the midhumeral region. The nerve that provides sensation to the dorsum of the hand proximal to the thumb and index finger is the superficial branch of the radial nerve. The posterior interosseous nerve supplies a strip of skin on the back of the forearm and wrist extensors. The lateral antebrachial cutaneous nerve is a continuation of the musculocutaneous nerve and supplies the lateral side of the forearm. The medial antebrachial cutaneous is a direct branch of the medial cord and supplies skin of the medial side of the forearm. The dorsal cutaneous branch of the ulnar nerve supplies the medial side of the dorsum of the hand.

A 61-year-old man was hit in the midhumeral region of his left arm by a cricket bat. Physical examination reveals an inability to extend the wrist and loss of sensation on a small area of skin on the dorsum of the hand proximal to the first two fingers. What nerve supplies this specific region of the hand? A. Radial B. Posterior interosseous C. Lateral antebrachial cutaneous D. Medial antebrachial cutaneous E. Dorsal cutaneous of ulnar

*The answer is A.* The force of the woman's fall on the out- stretched hand was transmitted up through the forearm, in this case resulting in dislocation of the olecranon at the elbow, putting traction on the ulnar nerve as it passes around the medial epicondyle of the humerus. Ulnar trauma at the elbow can cause weakness in medial flexion (adduction) at the wrist, from loss of the flexor carpi ulnaris. Ulnar nerve injury also results in sensory loss in the medial hand and paralysis of the interossei and medial two lumbricals, with clawing especially of digits 4 and 5. Injury of the ulnar nerve at the pisiform bone would not affect the flexor carpi ulnaris, nor would it produce sensory loss on the dorsum of the hand because the dorsal cutaneous branch of the ulnar branches off proximal to the wrist. Carpal tunnel problems affect median nerve function, which is not indicated here. The ulnar nerve passes medial to the cubital fossa between the heads of the flexor carpi ulnaris, not between the heads of the flexor digitorum superficialis. Injuries at the radial neck affect the site of division of the radial nerve, and its paralysis would not result in the clinical problems seen in this patient.

A 67-year-old woman had a bad fall while walking her dog the evening before. She states that she fell on her outstretched hand. Radiographs do not demonstrate any bony fractures. The clinician observes the following signs of neurologic injury: weakness of flexion of her wrist in a medial direction, a loss of sensation on the medial side of the hand, and clawing of the fingers. Where is the most likely place of nerve trauma? A. Behind the medial epicondyle B. Between the pisiform bone and the flexor retinaculum C. Within the carpal tunnel D. At the cubital fossa, between the ulnar and radial heads of origin of flexor digitorum superficialis E. At the radial neck, 1 cm distal to the humerocapitellar joint

*The answer is D.* Inferior dislocation of the head of the humerus may damage the axillary nerve, which arises from the posterior cord of the brachial plexus, runs through the quadrangular space accompanied by the posterior humeral circumflex vessels around the surgical neck of the humerus, and supplies the deltoid and teres minor.

A 7-year-old boy falls from a tree house and is brought to an emergency room of a local hospital. On examination, he has weakness in rotating his arm laterally because of an injury of the axillary nerve. Which of the following conditions is most likely to cause a loss of the axillary nerve function? (A) Injury to the lateral cord of the brachial plexus (B) Fracture of the anatomic neck of the humerus (C) Knife wound on the teres major muscle (D) Inferior dislocation of the head of the humerus (E) A tumor in the triangular space in the shoulder region

*The answer is B.* The fracture line of the upper third of the radius lies between the bony attachments of the supinator and the pronator teres muscles. The distal radial fragment and hand are pronated due to unopposed contraction of pronator teres and pronator quadratus muscles. The proximal fragment deviates laterally by the unopposed contraction of the supinator muscle. The brachioradialis inserts distally on the radius. The brachialis inserts on the coronoid process of the ulna and would not be involved in the lateral deviation of the radius.

A 74-year-old woman is admitted to the emergency department after stumbling over her pet dog. Radiographic examination reveals a fracture of the upper third of the right radius, with the distal fragment of the radius and hand pronated. The proximal end of the fractured radius deviates laterally. Which of the following muscles is primarily responsible for the lateral deviation? A. Pronator teres B. Supinator C. Pronator quadratus D. Brachioradialis E. Brachialis

*The answer is A.* The median nerve supplies sensory innervation to the thumb, index, and middle fingers and also to the lateral half of the ring finger. The median nerve also provides motor innervation to muscles of the thenar eminence. Compression of the median nerve in the carpal tunnel explains these deficits in conjunction with normal functioning of the flexor compartment of the forearm because these muscles are innervated by the median nerve proximal to the carpal tunnel. Also, sensory innervation in the proximal palm will be normal because the palmar branch of the radial nerve usually branches off proximal to the flexor retinaculum. The ulnar nerve is not implicated in these symptoms. It does not provide sensation to digits 1 to 3. Compression of the brachial plexus could not be attributed to pressure from the triceps brachii because this muscle is located distal to the plexus. In addition, brachial plexus symptoms would include other upper limb deficits, rather than the focal symptoms described in this case. Osteoarthritis of the cervical spine would also lead to increasing complexity of symptoms.

A 79-year-old man has numbness in the middle three digits of his right hand and finds it difficult to grasp objects with that hand. He states that he retired 9 years earlier, after working as a carpenter for 50 years. He has atrophy of the thenar eminence (Fig. 6-3). Which of the following conditions is the most likely cause of the problems in his hand? A. Compression of the median nerve in the carpal tunnel B. Formation of the osteophytes that compress the ulnar nerve at the medial epicondyle C. Hypertrophy of the triceps brachii muscle compressing the brachial plexus D. Osteoarthritis of the cervical spine E. Repeated trauma to the ulnar nerve

*The answer is C.* The axillary nerve passes dorsally around the surgical neck of the humerus (accompanied by the posterior circumflex humeral artery) and can be injured when the humerus is fractured at that location. The axillary nerve provides sensation to the skin over the upper, lateral aspect of the shoulder. Therefore, although the patient might not be able to abduct the arm because of the injury, a simple test of skin sensation can indicate whether there is associated nerve injury of the axillary nerve (CN XI). Shrugging the shoulders can help assess trapezius function, thereby testing the spinal accessory nerve. Intact sen- sation of the skin on the medial aspect of the axilla and arm is an indication that the radial and intercos- tobrachial nerves are functional. Pushing against an immovable object tests the serratus anterior muscle and the long thoracic nerve.

A male skier had a painful fall against a rocky ledge. Radiographic findings revealed a hairline fracture of the surgical neck of the humerus. The third-year medical student assigned to this patient was asked to determine whether there was injury to the nerve associated with the area of injury. Which of the following tests would be best for checking the status of the nerve? A. Have the patient abduct the limb while holding a 10 lb weight B. Have the patient shrug the shoulders C. Test for presence of skin sensation over the lateral side of the shoulder D. Test for normal sensation over the medial skin of the axilla E. Have the patient push against an immovable object like a wall and assess the position of the scapula

*The answer is C.* The subscapularis muscle inserts on the lesser tubercle of the humerus. The supraspinatus, infraspinatus, and teres minor muscles insert on the great tubercle of the humerus. The coracohumeral ligament attaches to the greater tubercle.

A rock climber falls on his shoulder, resulting in chipping off the lesser tubercle of the humerus. Which of the following structures would most likely have structural and functional damage? (A) Supraspinatus muscle (B) Infraspinatus muscle (C) Subscapularis muscle (D) Teres minor muscle (E) Coracohumeral ligament

*The answer is E.* The injury being described is also known as Erb-Duchenne paralysis or "waiter's tip hand" and is relatively common in children after a difficult delivery. This usually results from an injury to the upper trunk of the brachial plexus, presenting with loss of abduction, flexion, and lateral rotation of the arm. The superior trunk of the brachial plexus consists of spinal nerve ventral rami C5-6.

After a forceps delivery of an infant boy, the baby presents with his left upper limb adducted, internally rotated, and flexed at the wrist. The startle reflex is not seen on the ipsilateral side. Which part of the brachial plexus was most likely injured during this difficult delivery? A. Lateral cord B. Medial cord C. Ventral rami of the lower trunk D. Ventral ramus of the middle trunk E. Ventral rami of the upper trunk

*The answer is C.* The supraspinatus muscle inserts on the greater tubercle of the humerus and is said to initiate abduction of the arm at the shoulder. It is supplied principally by spinal nerve C5. The subscapularis muscle is the only muscle that inserts on the lesser tubercle. The subscapularis muscle is innervated by the upper and lower subscapular nerves. The teres minor muscle takes origin from the lateral border of the scapula; the teres major muscle takes origin from the region of the inferior angle and the lateral border of the scapula.

After an orthopedic surgeon examined the MRI of the shoulder of a 42-year-old woman he informed her that the supraspinatus tendon was injured and needed to be repaired surgically. Which of the following is true of the supraspinatus muscle? A. It inserts onto the lesser tubercle of the humerus. B. It initiates adduction of the shoulder. C. It is innervated chiefly by the C5 spinal nerve. D. It is supplied by the upper subscapular nerve. E. It originates from the lateral border of the scapula.

*The answer is D.* The scapular notch transmits the suprascapular nerve below the superior transverse ligament, whereas the suprascapular artery and vein run over the ligament. The suprascapular nerve supplies the supraspinatus and infraspinatus muscles. The axillary nerve innervates the deltoid and teres minor muscles. The subscapular nerves innervate the teres major and subscapularis muscles.

An 11-year-old boy falls down the stairs. A physician examines a radiograph of the boy's shoulder region (below). If the structure indicated by the letter A is calcified, which of the following muscles is most likely paralyzed? (A) Deltoid (B) Teres major (C) Teres minor (D) Infraspinatus (E) Subscapularis

*The answer is A.* The abductors of the arm are the deltoid and supraspinatus muscles, which are innervated by the axillary and suprascapular nerves, respectively. The thoracodorsal nerve supplies the latissimus dorsi, which can adduct, extend, and rotate the arm medially. The upper and lower subscapular nerves supply the subscapularis, and the lower subscapular nerve also supplies the teres major; both of these structures can adduct and rotate the arm medially. The musculocutaneous nerve supplies the flexors of the arm, and the radial nerve supplies the extensors of the arm. The dorsal scapular nerve supplies the levator scapulae and rhomboid muscles; these muscles elevate and adduct the scapula, respectively.

An 18-year-old boy involved in an automobile accident presents with arm that cannot abduct. His paralysis is caused by damage to which of the following nerves? (A) Suprascapular and axillary (B) Thoracodorsal and upper subscapular (C) Axillary and musculocutaneous (D) Radial and lower subscapular (E) Suprascapular and dorsal scapular

*The answer is B.* A supracondylar fracture often results in injury to the median nerve. The course of the median nerve is anterolateral, and at the elbow it lies medial to the brachial artery on the brachialis muscle. The axillary nerve passes posteriorly through the quadrangular space, accompanied by the posterior circumflex humeral artery, and winds around the surgical neck of the humerus. Injury to the surgical neck may damage the axillary nerve. The musculocutaneous nerve pierces the coracobrachialis muscle and descends between the biceps brachii and brachialis muscle. It continues into the forearm as the lateral antebrachial cutaneous nerve. The ulnar nerve descends behind the medial epicondyle in its groove and is easily injured and produces "funny bone" symptoms.

An 18-year-old man is brought to the emergency department after an injury while playing rugby. Imaging reveals a transverse fracture of the humerus about 1 inch proximal to the epicondyles. Which nerve is most frequently injured by the jagged edges of the broken bone at this location? A. Axillary B. Median C. Musculocutaneous D. Radial E. Ulnar

*The answer is D.* The interossei are the most important muscles in extension of the interphalangeal (IP) joints because of the manner of their insertion into the extensor expansion of the fingers, which passes dorsal to the transverse axes of these joints. The lumbrical muscles assist in IP extension, in addition to flexing the meta- carpophalangeal joints. Ulnar nerve injury at the wrist results in paralysis of all the interossei and the medial two lumbricals. Extensors of the metacarpophalangeal (MCP) joints are innervated by the deep radial nerve. Unopposed extension of the MCP joints causes them to be held in extension whereas unopposed long flexors of the fingers (supplied by median and ulnar nerves proximally in the forearm) cause them to be flexed into the "claw" position. The lumbricals of digits two and three are still intact because they are supplied by the median nerve, so clawing is not seen as much on these digits. Loss of opposition would result from median or recurrent nerve paralysis. If the ulnar nerve is cut at the wrist, its dorsal cutaneous branch to the dorsum of the hand is unaffected.

An 18-year-old man suffered a significant laceration through the skin and underlying tissues at the distal crease of the wrist. The medical student rotating through the emergency department suspected (correctly) that the ulnar nerve was cut completely through at this location. Which of the following would most likely occur? A. The patient could not touch the tip of the thumb to the tips of the other digits B. There would be loss of sensation on the dorsum of the medial side of the hand C. The patient would be unable to flex the interphalangeal joints D. There would be decreased ability to extend the interphalangeal joints E. There would be no serious functional problem at all to the patient

*The answer is A.* The tendon of the long head of the biceps brachii muscles runs in the intertubercular groove on the proximal humerus as it changes direction and turns medially to attach to the supraglenoid tubercle of the scapula. This change in direction within an osseous structure predisposes the tendon to wear and tear, particularly in people who overuse the biceps brachii muscle. This type of injury presents with a characteristic sign called the "Popeye sign" after the cartoon character.

An 85-year-old man is admitted to the hospital with a painful arm after lifting a case of wine. Physical examination gives evidence of a rupture of the long tendon of the biceps brachii (Fig. 6-6). Which of the following is the most likely location of the rupture? A. Intertubercular groove B. Midportion of the biceps brachii muscle C. Junction with the short head of the biceps brachii muscle D. Proximal end of the combined biceps brachii muscle E. Bony insertion of the muscle

*The answer is E.* The suprascapular artery passes over, and the suprascapular nerve passes under, the superior transverse scapular ligament. This ligament bridges the suprascapular notch in the upper border of the scapula, converting the notch to foramen. The artery and nerve then pass deep to the supraspinatus muscle, thereafter supplying it and then passing through the spinoglenoid notch to supply the infraspinatus. The subscapular artery is a branch of the third part of the axillary artery; it divides into circumflex scapular and thoracodorsal branches. The transverse cervical artery courses anterior to this site. The dorsal scapular artery and nerve pass deep to the medial border of the scapula. The posterior circumflex humeral branch of the axillary artery passes through the quadrangular space with the axillary nerve.

An orthopedic surgeon exposed a muscle in the supraspinous fossa so that she could move it laterally while repairing an injured rotator cuff. As she reflected the muscle from its bed, an artery was exposed crossing the ligament that bridges the notch in the superior border of the scapula. What artery was this? A. Subscapular B. Transverse cervical C. Dorsal scapular D. Posterior circumflex humeral E. Suprascapular

*The answer is C.* The tendon of the long head of the biceps brachii muscle passes through the glenohumeral joint, surrounded by synovial membrane. The glenohumeral is a ligament that attaches to the glenoid labrum. The long head of the triceps brachii arises from the infraglenoid tubercle, beneath the glenoid fossa. The infraspinatus tendon passes posterior to the head of the humerus to insert on the greater tubercle. The coracobrachialis arises from the coracoid process and inserts on the humerus.

Arthroscopic examination of the shoulder of a 62-year-old woman clearly demonstrated erosion of the tendon within the glenohumeral joint. What tendon was this? A. Glenohumeral B. Long head of triceps brachii C. Long head of biceps brachii D. Infraspinatus E. Coracobrachialis

*The answer is E.* The medial cord has been injured by traction on the lower trunk of the brachial plexus. The medial cord is the continuation of the inferior (lower) trunk of the brachial plexus, which is formed by C8 and T1. C5 and C6 are typically associated with the superior (upper) trunk level and thus the lateral cord. C7 forms the middle trunk. An injury to the posterior cord would usually involve the C7 spinal nerve. This is a typical Klumpke paralysis.

As a girl fell from the uneven parallel bars, a 17-year-old female gymnast grasped the lower bar briefly with one hand but then fell painfully to the floor. An MRI examination reveals an injury to the medial cord of the brachial plexus. Which of the following spinal nerve levels would most likely be affected? A. C5, C6 B. C6, C7 C. C7, C8 D. C7, C8, T1 E. C8, T1

*The answer is D.* The left spinal accessory nerve (CN XI) has been injured distal to the sternocleidomastoid muscle resulting in paralysis of the trapezius, allowing the shoulder to droop and the superior angle to push out posteriorly. The sternocleidomastoid muscles are intact, as demonstrated by symmetry in strength in turning the head to the right and left. There is no indication of paralysis of the lateral rotators of the shoulder or elbow flexors (suprascapular nerve or upper trunk). Thoracodorsal nerve injury would result in paralysis of the latissimus dorsi muscle, an exten-sor, and medial rotator of the humerus.

During a fight in a tavern, a 45-year-old male construction worker received a shallow stab wound from a broken beer bottle at a point near the middle of the left posterior triangle of his neck. Upon physical examination, it is observed that the left shoulder is drooping lower than the right shoulder, and the superior angle of the scapula juts out slightly. Strength in turning the head to the right or left appears to be symmetric. Which of the following nerves is most likely injured? A. Suprascapular nerve in the supraspinous fossa B. The terminal segment of the dorsal scapular nerve C. The upper trunk of the brachial plexus D. The spinal accessory nerve in the posterior cervical triangle E. The thoracodorsal nerve in the axilla

*The answer is A.* Injury to the radial nerve in the spiral groove will paralyze the abductor pollicis longus muscle and both extensors of the thumb. This injury will also lead to wrist drop (inability to extend the wrist). Weakness of grip would also occur, although this is not mentioned in the question. If the wrist is flexed, finger flexion and grip strength are weakened because the long flexor tendons are not under tension. Note how much your strength of grip is increased when your wrist is extended versus when it is flexed.

Examination of a 21-year-old female athlete with an injury of the radial nerve in the spiral groove would typically demonstrate which of the following physical signs? A. Weakness of thumb abduction and thumb extension B. Weakness of thumb opposition C. Inability to extend the elbow D. Paralysis of pronation of the hand E. Paralysis of abduction and adduction of the arm

*The answer is E.* Scoliosis (severe lateral curvature of the spine) in the patient is causing compression or stretching of the T1 spinal nerve ramus by the first rib as the nerve ascends to join C8 and form the lower trunk of the brachial plexus. T1 provides sensation for the medial side of the forearm, via the medial antebrachial cutaneous nerve from the medial cord of the brachial plexus. T1 is the principal source of motor supply to all of the intrinsic muscles in the palm. Its dysfunction affects all fine motor movements of the digits. Long flexors of the fingers are intact; therefore, the median nerve and ulnar nerve are not injured. The extensors of the wrist are functional; therefore, the radial nerve is not paralyzed. The only sensory disturbance is that of the T1 dermatome.

Fine motor function in the right hand of a 14-year- old girl with scoliosis since birth appeared to be quite reduced, including opposition of the thumb, abduction and adduction of the digits, and interphalangeal joint extension. Radiography confirmed that her severe scoliosis was causing marked elevation of the right first rib. Long flexor muscles of the hand and long extensors of the wrist appear to be functioning within normal limits. There is notable anesthesia of the skin on the medial side of the forearm; otherwise, sensory function in the limb is intact. Which of the following neural structures is most likely impaired? A. Median nerve B. Middle trunk of the brachial plexus C. Radial nerve D. Lower trunk of the brachial plexus E. T1 ventral ramus

*The answer is E.* The deep branch of the radial nerve courses between the two heads of the supinator muscle and is located just medial and distal to the lateral epicon-dyle. After the nerve emerges from the supinator it is called the posterior interosseous nerve. It can be irritated by hypertrophy of the supinator, which compresses the nerve, causing pain and weakness. The ulnar nerve courses laterally behind the medial epicondyle and continues anterior to the flexor carpi ulnaris muscle. The median nerve passes into the forearm flexor compartment; the superficial radial nerve courses down the lateral aspect of the posterior forearm and would not cause pain due to pressure applied to the posterior forearm.

Following several days of 12-hour daily rehearsals of the symphony orchestra for a performance of a Wagnerian opera, the 52-year-old male conductor experienced such excruciating pain in the posterior aspect of his right forearm that he could no longer direct the musicians. When the maestro's forearm was palpated 2 cm distal and posteromedial to the lateral epicondyle, the resulting excruciating pain caused him to grimace. Injections of steroids and rest were recommended to ease the pain. Which of the following injuries is most likely? A. Compression of the ulnar nerve by the flexor carpi ulnaris B. Compression of the median nerve by the pronator teres C. Compression of the median nerve by the flexor digitorum superficialis D. Compression of the superficial radial nerve by the brachioradialis E. Compression of the deep radial nerve by the supinator

*The answer is A.* This proximal injury to the median nerve would paralyze all of the long flexors of the digits, except for the muscle that flexes the distal interphalangeal joints of digits 4 and 5, thereby swinging the "balance of power" to the muscles that extend the digits, all of which are innervated by the radial nerve. The intrinsic hand muscles can aid in flexion of the metacarpophalangeal joints, and are innervated by the ulnar nerve. However, they are of insufficient size to compensate for the extensor forces exerted on fingers.

In a penetrating wound to the forearm of a 24-year-old man, the median nerve is injured at the entrance of the nerve into the forearm. Which of the following would most likely be apparent when the patient's hand is relaxed? A. The MCP and IP joints of the second and third digits of the hand will be in a condition of extension. B. The third and fourth digits will be held in a slightly flexed position. C. The thumb will be flexed and slightly abducted. D. The first, second, and third digits will be held in a slightly flexed position. E. The MCP and IP joints of the second and third digits of the hand will be in a condition of flexion.

*The answer is A.* The three chief contents of the cubital fossa are the biceps brachii tendon, brachial artery, and median nerve (lateral to medial). The common and anterior interosseous arteries arise distal to the cubital fossa; the ulnar and radial arteries are the result of the bifurcation of the brachial artery distal to the cubital fossa.

Laboratory studies in the outpatient clinic on a 24-year-old woman included assessment of circulating blood chemistry. Which of the following arteries is most likely at risk during venipuncture at the cubital fossa? A. Brachial B. Common interosseous C. Ulnar D. Anterior interosseous E. Radial

*The answer is A.* The ulnar nerve enters the forearm by passing between the two heads of the flexor carpi ulnaris and descends between and innervates the flexor carpi ulnaris (for medial wrist deviation) and flexor digitorum profundus (medial half) muscles. Injuring the ulnar nerve results in claw hand. It enters the hand superficial to the flexor retinaculum and lateral to the pisiform bone, where it is vulnerable to damage. The ulnar nerve also enters Guyon's canal, but damage to it here would not present with the aforementioned symptoms. The median nerve enters the carpal tunnel and the radial nerve passes deep to the brachioradialis.

Physical examination reveals weakness of medial deviation of the wrist (adduction), loss of sensation on the medial side of the hand, and clawing of the fingers. Where is the most likely place of injury? A. Compression of a nerve passing between the humeral and ulnar heads of origin of flexor carpi ulnaris B. Compression of a nerve passing at Guyon's canal between the pisiform bone and flexor retinaculum C. Compression of a nerve passing through the carpal tunnel D. Compression of a nerve passing between the ulnar and radial heads of origin of flexor digitorum superficialis E. Compression of a nerve passing deep to brachioradialis muscle

*The answer is C.* The long thoracic nerve was injured during the axillary dissection, resulting in paralysis of the serratus anterior muscle. The serratus anterior is important in rotation of the scapula in raising the arm above the level of the shoulder. Its loss results in protrusion of the medial border ("winging" of the scapula), which is more obvious when one pushes against resistance. The long thoracic nerve arises from the ventral rami of C5, C6, and C7. The upper trunk (C5, C6) supplies rotator and abductor muscles of the shoulder and elbow flexors. The posterior division of the middle trunk contains C7 fibers for distribution to extensor muscles; likewise, the posterior cord supplies extensors of the arm, forearm, and hand. The lateral cord (C5, C6, and C7) gives origin to the lateral pec-toral nerve, the musculocutaneous nerve, and the lateral root of the median nerve. There is no sensory loss in the limb in this patient; injury to any of the

Several weeks after surgical dissection of her left axilla for the removal of lymph nodes for staging and treatment of her breast cancer, a 32-year-old woman was told by her general physician that she had "winging" of her left scapula when she pushed against resistance during her physical examination. She told the physician that she had also experienced difficulty lately in raising her left arm above her head when she was combing her hair. In a subsequent consult visit with her surgeon, she was told that a nerve was accidentally injured during the diagnostic surgical procedure and that this produced her scapular abnormality and inability to raise her arm normally. What was the origin of this nerve? A. The upper trunk of her brachial plexus B. The posterior division of the middle trunk C. Ventral rami of the brachial plexus D. The posterior cord of the brachial plexus E. The lateral cord of the brachial plexus

*The answer is A.* The basilic vein can be used for dialysis, especially when the cephalic vein is judged to be too small, as in this case. The basilic vein can be elevated from its position as it passes through the fascia on the medial side of the arm (brachium). The cephalic vein passes more laterally up the limb. The lateral cubital vein is a tributary to the cephalic vein, and the medial cubital vein joins the basilic vein, both of which are rather superficial in position. The medial antebrachial vein courses up the midline of the forearm (antebrachium) ventrally.

The kidneys of a 32-year-old woman were failing, and she needed to be placed on dialysis. However, the search in her upper limb for a suitable vein was unexpectedly difficult. The major vein on the lateral side of the arm was too small; others were too delicate. Finally, a vein was found on the medial side of the arm that passed through the superficial and deep fascia to join veins beside the brachial artery. Which of the following veins was this? A. Basilic B. Lateral cubital C. Cephalic D. Medial cubital E. Medial antebrachial

*The answer is C.* The injury has occurred just beyond the third part of the axillary artery. The only collateral arterial channel between the third part of the axillary artery and the brachial artery is between the posterior circumflex humeral and the ascending branch of the profunda brachii, and this anastomotic path is often inadequate to supply the arterial needs of the limb. The posterior circumflex humeral arises from the third part of the axillary artery. It typically anastomoses with a variably small, ascending branch of the profunda brachii branch of the brachial artery. The supra- scapular artery anastomoses with the circumflex scapular deep to the infraspinatus. The dorsal scapular artery (passing beneath the medial border of the scapula) has no anastomosis with thoracodorsal within the scope of the injury. The lateral thoracic artery has no anastomoses with the brachial artery. The supreme thoracic artery (from first part of axillary) has no helpful anastomoses with the thoracoacromial (second part of axillary).

The shoulder of a 44-year-old deer hunter had been penetrated by a bolt released from a crossbow. The bolt had transected the axillary artery just beyond the origin of the subscapular artery. A compress is placed on the wound with deep pressure. After a clamp is placed on the bleeding artery, thought is given to the anatomy of the vessel. What collateral arterial pathways are available to bypass the site of injury? A. Suprascapular with circumflex scapular artery B. Dorsal scapular with thoracodorsal artery C. Posterior circumflex humeral artery with deep brachial artery D. Lateral thoracic with brachial artery E. Supreme thoracic artery with thoracoacromial artery

*The answer is B.* Each of the five termi- nal branches of the brachial plexus (musculocutaneous, median, ulnar, radial, and axillary nerves) passes through a muscular or osseofascial tunnel at some point in its distribu- tion, where it may be subject to entrapment in a tunnel syn- drome. The pronator teres muscle arises via two heads, one from the medial epicondyle of the humerus and the other from the coronoid process of the ulna, with a tendinous arch connecting them. The median nerve exits the cubital fossa and enters the forearm by passing between these heads, where it may be unduly compressed in a pronator teres syn- drome. This condition would influence much of the median nerve territory in the forearm plus the entire median nerve territory in the hand. Choice A (Deep branch of the radial nerve) is incorrect. The radial nerve descends from the arm into the cubital fossa, where it divides into superficial and deep branches. The deep branch of the radial nerve pierces the supinator muscle, winds around the proximal end of the radius within the substance of that muscle, and passes into the deep posterior compartment of the forearm as the poste- rior interosseous nerve. The nerve may be entrapped within the supinator, resulting in a supinator syndrome. Such a condition would affect the deeper, more distal extensor muscles arising in the forearm and some sensory areas in the wrist joints. Choice C (Deep branch of the ulnar nerve) is incorrect. The ulnar nerve enters the hand superficial to the flexor retinaculum, runs through a groove between the pisiform and hook of the hamate (Guyon canal), and divides into superficial and deep branches at the base of the hypothenar eminence. The deep branch curls deeply there and enters the deep-lying adductor/interosseous compartment in the palmar aspect of the hand. Compression of the ulnar nerve in Guyon canal may cause a Guyon tunnel syndrome, which affects the hypothenar muscles, medial two lumbricals, all interossei, adductor pollicis, and a large sensory area on both palmar and dorsal sides of the hand. Choice D (Superficial branch of the ulnar nerve) is incorrect. The superficial branch of the ulnar nerve does not enter a tunnel and is not subject to a tunnel syndrome. This nerve supplies the palmaris brevis muscle but is mostly cutaneous across the palmar and dorsal aspects of the medial third of the hand. Choice E (Musculocutaneous nerve) is incorrect. This nerve penetrates the coracobrachialis muscle, supplies the three flexor muscles in the anterior compartment of the arm, and continues into the forearm as the lateral cutaneous nerve of the forearm. Entrapment of the nerve within the coracobrachialis is rare.

"Pronator teres syndrome" is a condition in which one of the following nerves is excessively compressed where it passes between the two heads of the pronator teres muscle. Which of the following nerves is entrapped? (A) Deep branch of radial nerve (B) Median Nerve (C) Deep branch of ulnar nerve (D) Superfi cial branch of ulnar nerve (E) Musculocutaneous nerve

*The answer is C.* The recurrent (thenar) branch of the median nerve lies subcutaneously in the thenar eminence and can be damaged by lac- erations in this area. This nerve innervates most of the thenar muscles, including the Opponens pollicis, Abductor pollicis brevis, and superficial head of the Flexor pollicis brevis (mnemonic = "OAF"). Opposition of the thumb would be lost by cutting the recurrent branch of the median nerve. Though the abductor pollicis brevis is denervated, the abductor pollicis longus, innervated by the deep branch of the radial nerve, is still intact. Also, the flexor pollicis longus, innervated by the anterior interosseous nerve, would still allow flexion at the interphalangeal joint of the thumb. Cutting the recur- rent branch of the median nerve would lead to atrophy of the thenar muscular complex, a condition known as "ape hand." Choice A (Deep branch of the radial nerve) is incorrect. The deep branch of the radial nerve is also called the posterior interosseous nerve as it exits from the supinator muscle. It innervates the abductor pollicis longus and extensor pol- licis longus and brevis muscles, which form the boundaries of the anatomical snuffbox. Cutting this nerve would cause loss of extension of the thumb and weakness in abduction. However, the puncture was on the thenar eminence, and the radial nerve does not travel there. Choice B (Superficial branch of the radial nerve) is incorrect. The superficial branch of the radial nerve arises from the radial nerve in the cubital fossa. This nerve is purely cutaneous, supplying sensation to the dorsum of the hand and fingers. Because it does not have a motor component, this nerve would not be responsible for the deficit in thumb function. Choice D (Deep branch of ulnar nerve) is incorrect. The deep branch of the ulnar nerve innervates most of the intrinsic muscles of the hand, including the hypothenar muscles, medial two lumbrical muscles, the palmar and dorsal interossei, adductor pollicis, and the deep head of the flexor pollicis brevis. With a small motor supply to the thenar compartment (specifically the deep head of the flexor pollicis brevis), this nerve could not cause a substantial loss of function in the thenar eminence. Choice E (Superficial branch of the ulnar nerve) is incorrect. The superficial branch of the ulnar nerve arises from the ulnar nerve distal to the flexor retinaculum. This nerve supplies cutaneous branches to the anterior surface of the medial one and a half fingers. The palmaris brevis is the only muscle supplied by this nerve, and this muscle tightens the skin of the medial surface of the palm. Therefore, this nerve would not be responsible for substantial loss of motor function in the thumb.

A 10-year-old boy was running across a parking lot when he tripped and received lacerations on the base of his thumb from a broken glass bottle. On examination, his thumb was unable to oppose to his fingers, and the thumb also showed weakness when abducting and flexing. No sensory deficits were reported. What nerve was most likely severed? (A) Deep branch of radial nerve (B) Superficial branch of radial nerve (C) Recurrent branch of median nerve (D) Deep branch of ulnar nerve (E) Superficial branch of ulnar nerve

*The answer is C.* The median nerve and brachial artery were injured. Injury to the median nerve is indicated by the weakness of pronation, wrist flexion, and grip strength. The median nerve innervates the muscles that govern or carry out these movements. The bra- chial artery is near the median nerve and can also be injured. Flexion at the distal interphalangeal joints of the ring and little fingers indicates that the ulnar nerve is intact. The radiograph indicates a supracondylar fracture of the humerus, which is the region in which the median nerve passes. A fracture at the surgical neck would injure the axillary nerve and posterior circumflex humeral artery. A fracture of the shaft of the humerus would injure the radial nerve and deep brachial artery. The superficial radial nerve and artery would be damaged by injury over or in the anatomical snuffbox.

A 14-year-old boy falls on his outstretched hand. Examination reveals point tenderness above the humeral epicondyles and a pulsatile mass just above the cubital fossa. Neurological examination of the upper limb reveals weakness of pronation, wrist flexion, and grip strength. Only flexion at the distal interphalangeal joints of the ring and little fingers is intact. Thumb flexion and opposition are also impaired. A radiograph reveals a supracondylar fracture of the humerus. Which structures injured by the fracture best account for the findings? A. Axillary nerve and posterior circumflex humeral artery B. Radial nerve and deep brachial artery C. Median nerve and brachial artery D. Superficial radial nerve and radial artery E. Ulnar nerve and ulnar artery

*The answer is E.* Opposition, a complex movement, begins with the thumb in the extended position and initially involves abduction and medial rotation of the first metacarpal. This is produced by the action of the opponens pollicis muscle at the carpometacarpal joint by the flexor pollicis brevis muscle and then by flexion at the metacarpophalangeal joint. The opponens pollicis and flexor pollicis brevis muscles are supplied by the recurrent branch of the median nerve (C8, T1). The median nerve is the principal nerve of the anterior compartment of the forearm and the thenar muscles of the hand. It passes through the carpal tunnel with the tendons of the flexor digitorum pro- fundus, flexor digitorum superficialis, and flexor pol- licis longus to supply the thenar muscles of the hand. Abduction and adduction of the fingers is done by the palmar and dorsal interossei muscles, which are sup- plied by the median nerve. The extensor indicis extends the index finger. The ring and little fingers are flexed by the medial two tendons of the flexor digitorum superficialis (supplied by the median nerve) and the medial two tendons of the flexor digitorum profundus (supplied by the ulnar nerve).

A 16-year-old girl is brought to the emergency department after attempting suicide by cutting her wrist. The deepest part of the wound is between the tendons of the flexor carpi radialis and the flexor digitorum superficialis. This patient is most likely to have a deficit of which of the following? A. Adduction and abduction of the fingers B. Extension of the index finger C. Flexion of the ring and little finger D. Sensation over the base of the little finger E. Opposition of the thumb and slightly weakened flexion of the second and third digits

*The answer is A.* The axillary nerve may be damaged in approximately one of seven shoulder dislocations. This nerve innervates the deltoid and teres minor muscles as well as supplying innervation to the skin overlying the deltoid in the superolateral aspect of the arm. Loss of innervation to the deltoid muscle would explain the weakness in abduction of the upper limb. The teres minor assists the infraspinatus muscle in external rotation of the shoulder. Choice B (Median nerve) is incorrect. The median nerve does not branch proximal to the elbow, and its sensory distribution is limited to the hand. The median nerve could not be responsible for the patient's motor and sensory deficits. Choice C (Ulnar nerve) is incorrect. The ulnar nerve can be damaged during shoulder dislocations; however, its sensory distribution is limited to distal to the wrist, and the first muscle it innervates is in the forearm (flexor carpi ulnaris). Therefore, the ulnar nerve could not be responsible for this patient's motor and sensory deficits. Choice D (Radial nerve) is incorrect. The radial nerve supplies motor innervation to the posterior compartments of the arm and forearm. Damage to this nerve would cause weakness in extending at the elbow and wrist joints. The radial nerve gives rise to the posterior cutaneous nerves of the arm and forearm as well as the inferior lateral cutaneous nerve of the arm. However, it would not affect the superior lateral cutaneous nerve of the arm, arising from the axillary nerve, which was damaged in this patient. The radial nerve would also not affect abduction and external rotation of the shoulder. Choice E (Musculocutaneous nerve) is incorrect. The musculocutaneous nerve can be damaged during shoulder dislocations; however, this nerve supplies the motor innervation to the anterior compartment of the arm. Trauma to this nerve would lead to weakness in flexing the elbow and supinating the forearm. Its sensory distribution is limited to the lateral aspect of the forearm, so it was not the nerve damaged in this patient.

A 17-year-old male football player suffers a shoulder injury and arrives at the ER 2 hours after the injury. The physician diagnoses a shoulder dislocation, and after administration of a local anesthetic solution, the doctor repositions the head of the humerus into the glenoid cavity of the scapula (reduction). No fractures are seen on X-rays. However, the patient displays weakness in abduction and external rotation at the shoulder. A loss of sensation is also noted at the superior and lateral aspects of the arm. What nerve was most likely damaged in this injury? (A) Axillary nerve (B) Median nerve (C) Ulnar nerve (D) Radial nerve (E) Musculocutaneous nerve

*The answer is E.* Fracture of the fifth metacarpal bone is called a "boxer's fracture" because this injury is often seen after an individual improperly punches a solid object with a clenched fist. The impact on the head of the fifth metacarpal causes the distal shaft of this bone to fracture. Skilled pugilists are trained to direct the impact of the clenched fist on the heads of the first and second metacarpals to avoid this type of injury. Choice A (Dislocation of the fifth metacarpophalangeal joint) is incorrect. This type of injury is rare, and a dislocation of the fifth metacarpo- phalangeal joint would have been detected by the physician on examination. Choice B (Fracture of the triquetral bone) is incorrect. This carpal bone is rarely fractured in this type of impact. Choice C (Fracture of the proximal phalanx of the ring finger) is incorrect. The impact of the clenched fist would have been localized to the head of the fifth metacarpal. Damage to the proximal phalanx of the ring finger would have occurred only if the fist was not clenched during impact. Choice D (Fracture of the proximal phalanx of the little fin- ger) is incorrect. The impact of the clenched fist would have been localized to the head of the fifth metacarpal. Damage to the proximal phalanx of the little finger would have occurred only if the fist was not clenched during impact.

A 17-year-old man has pain and moderate swelling over the dorsomedial aspect and in the hypothenar area of his right hand after punching a locker over a dispute with his girlfriend. What is the most likely fi nding on an X-ray of his hand? (A) Dislocation of the fifth metacarpophalangeal joint (B) Fracture of the triquetral bone (C) Fracture of the proximal phalanx of the ring fi nger (D) Fracture of the proximal phalanx of the little fi nger (E) Fracture of the fifth metacarpal bone

*The answer is B.* The lunate is shaped like a moon, thus its name. It is situated in the center of the proximal row of carpal bones where it articulates with the radius. This bone is the most commonly dislocated carpal bone, which leads to severe carpal instabilities. This dislocation often occurs in association with a trans-scaphoid fracture. It is important to note that scaphoid fractures are often difficult to see in radio- graphic imaging; however, these fractures can be detected frequently by applying direct pressure to the anatomical snuff- box. Choice A (Scaphoid) is incorrect. The scaphoid bone is located in the proximal row and is the most frequently frac- tured carpal bone. As the weight of the body is transmitted through the upper limb onto the outstretched hand and the impact of the fall exerts pressure back upon the limb, the scaphoid bone is crushed by these opposing forces and is subsequently fractured, usually along its narrowest part. The scaphoid bone is clinically relevant due to the frequency of fractures in younger patients and the poor vascularization of its proximal part. Avascular necrosis is often a postfracture complication that slows the healing of this bone. Choice C (Capitate) is incorrect. The capitate is located in the distal row of carpal bones, so this selection can be easily eliminated. Choice D (Triquetrum) is incorrect. The triquetrum is located in the proximal row of carpal bones and may be involved in a severe dislocation of the wrist. Specifically, the triquetrum can be displaced in Stage III wrist dislocations when the triquetrol- unate interosseous ligament is damaged. However, tearing this interosseous intercarpal ligament would only occur following dislocation of the lunate bone, which is the most commonly dislocated carpal bone. Choice E (Trapezium) is incorrect. The trapezium bone helps form the distal floor of the anatomic snuffbox and forces would be exerted on this bone during the fall. However, it is not located in the proximal row of carpal bones. Moreover, this four-sided bone is rarely fractured or dislocated due to its shape and construction.

A 19-year-old man arrives at his campus health clinic complaining of soreness in his right wrist. He explains he landed on an outstretched hand when he was tackled in a rugby match. He indicates that the pain worsens with movement and is minimized by stabilization of the wrist. There are no sensory deficits in his hand nor does he have trouble grasping or holding objects. Pressure applied to the anatomic snuffbox between the extensor pollicis brevis and extensor pollicis longus tendons produces no pain. Radiographic studies show no fractures but reveal an anterior dislocation of a bone in the proximal row of carpal bones. What carpal bone is most likely dislocated in this patient? (A) Scaphoid (B) Lunate (C) Capitate (D) Triquetrum (E) Trapezium

*The answer is D.* This case represents a classic presentation of "Saturday Night Palsy," where the radial nerve is compressed against the humerus in the arm. Remember, the radial nerve supplies motor innervation to the posterior compartments of the arm and forearm, so damage to this nerve would cause weakness in extending the elbow and wrist. This patient is unable to extend the wrist when the hand is placed in a pronated position ("wrist drop"), implying damage to the radial nerve. Moreover, the superficial branch of the radial nerve is responsible for cutaneous innervation over much of the dorsum of the hand, which explains the numbness and paresthesia in his hand. Choice A (Axillary nerve) is incorrect. The axillary nerve innervates the deltoid and teres minor muscles as well as the skin overlying the deltoid in the superolateral aspect of the arm. Compressing the axillary nerve would not affect wrist function or cause paresthesia distal to the wrist, which was seen in this student. Choice B (Median nerve) is incorrect. The median nerve is responsible for flexion at the wrist; however, this patient is having trouble with wrist extension. The sensory distribution for the median nerve is primarily on the palmar side of the hand rather than the dorsum of the hand, which is seen in this patient. Choice C (Ulnar nerve) is incorrect. The ulnar nerve is responsible for sensory innervation on the dorsum of the hand that is limited to the ulnar side (medial to the midline of the fourth finger). Damage to the ulnar nerve would not result in "wrist drop," which is an indication of a radial nerve injury. The ulnar nerve does supply motor innerva- tion to most of the intrinsic muscles of the hand. Choice E (Musculocutaneous nerve) is incorrect. The musculocutaneous nerve supplies motor innervation to the anterior compartment of the arm and cutaneous innervation to the lateral aspect of the forearm. Therefore, damage to this nerve would not affect wrist function.

A 21-year-old male college student reports to the student health clinic on Monday morning, the day after the Super Bowl. He explains that he was intoxicated and lost consciousness with his upper limbs draped over the back of a couch. He complains of numbness and paresthesia over the dorsum of his hand on the radial side and is unable to support the weight of his left hand when the hand is placed in a pronated position (see photo). What nerve was most likely damaged in this individual? (A) Axillary nerve (B) Median nerve (C) Ulnar nerve (D) Radial nerve (E) Musculocutaneous nerve

*The answer is B.* The most commonly fractured carpal bone is the scaphoid that forms the floor of the anatomical snuffbox. This area is a fossa located between the three long tendons of the thumb (tendons of abductor pollicis longus and extensor pollicis brevis laterally and tendon of the extensor pollicis longus medially). The scaphoid bone is frequently broken when an individual falls with an outstretched hand and lands on the palm with the hand abducted. A broken scaphoid bone is commonly seen in individuals under the age of 30. The scaphoid is broken due to its unfortunate position between the downward force transmitted by the weight of the upper limb and the upward force due to the impact of hitting the fl oor. The proximal aspect of a broken scaphoid bone can suffer from avascular necrosis due to its blood supply entering the bone distally. Choice A (Capitate) is incorrect. The capitate is located centrally in the distal row of carpal bones and articulates with most of the carpal bones (the triquetrum, pisiform, and trapezium being the exceptions). This bone is not located in the floor of the anatomical snuffbox, so it was not the most likely bone injured in this patient. Choice C (Hamate) is incorrect. The hamate is located in the distal row of carpal bones on the ulnar side. It is not related to the anatomical snuffbox, so it would not produce pain in the base of the thumb when broken. Choice D (Trapezium) is incorrect. The trapezium is located at the base of the thumb, but it is not commonly fractured when a person falls with an outstretched hand. A mnemonic for the position of the trapezium is "trapezium articulated with the thumb." Choice E (Pisiform) is incorrect. The pisiform is located in the proximal carpal row and is a sesamoid bone located within the tendon of the flexor carpi ulnaris muscle. Its position on the ulnar side of the wrist would not cause pain in the anatomical snuffbox.

A 21-year-old man goes to his college campus health clinic complaining of soreness in his left wrist after falling on an outstretched hand during a basketball game the previous day. He is supporting his left wrist and indicates that the pain worsens with movement and is minimized with inactivity. There is no loss of feeling in his hand, nor does he have trouble grasping or holding objects. The physician exacerbates the wrist pain by applying pressure to the base of the thumb in the anatomical snuffbox (see photo). Radiographic imaging will confirm a break of which carpal bone? (A) Capitate (B) Scaphoid (C) Hamate (D) Trapezium (E) Pisiform

*The answer is A.* The flexor retinaculum is a thick connective tissue ligament that spans the space between the medial and lateral sides of the base of the carpal tunnel. It protects and stabilizes the tendons that run beneath it. Damage to the flexor carpi ulnaris tendon, flexor digitorum superficialis tendons, and flexor digitorum profundus tendons result in functional losses in the hand. The palmar skin is loose connective tissue and does not have a shiny, glistening appearance.

A 21-year-old painter sustains a laceration on the anterior surface of his left wrist just distal to the skin fold crease. When he arrives at the emergency department, the physician extends the patient's wrist to determine the depth of the laceration and observes a broad, glistening white structure deep to the superficial fascia. The patient has no numbness or tingling of any of the fingers and is able to discriminate sharp/dull sensation in all of the fingers and palm of the hand. There is no loss of motion in any of the fingers or the hand, and grip strength is normal. Which structure is the physicianmost likely observing? A. Flexor retinaculum B. Flexor carpi ulnaris tendon C. Palmar skin D. Flexor digitorum superficialis tendons E. Flexor digitorum profundus tendons

*The answer is C.* Anterior dislocation of the humerus may damage the nerves located in the axilla or cause tears in the rotator cuff muscles. Internal rotation is the primary function of subscapularis muscle; with this being the only action impaired it is the most likely damaged muscle, probably as a result of injury to the upper and/or lower subscapular nerves that innervate this muscle. The infraspinatus and trees minor muscles are external rotators, and the supraspinatus muscle is the abductor of the arm from 0 to 15 degrees. The pectoralis major is a flexor, adductor, and medial rotator and would not likely be damaged during a shoulder dislocation.

A 21-year-old woman who is an athlete dislocated her glenohumeral joint while playing soccer and the shoulder was reduced in the emergency department. However, after 1 week the physician noted that the woman had lost strength when she attempted internal rotation of her arm at the shoulder. This finding was most likely caused by a tear in which of the following muscles? A. Infraspinatus B. Pectoralis minor C. Subscapularis D. Supraspinatus E. Teres minor

*The answer is B.* The radial artery enters the anatomical snuffbox as it passes to the posterior aspect of the hand to pass between the two heads of the 1st dorsal interosseous muscle. The ulnar artery continues anteriorly and enters the hand on the palmar surface. The anterior and posterior interosseous arteries are found anteriorly and posteriorly, respectively, on the interosseous membrane, which is located between the radius and ulna. The deep palmar arch is an anastomosis on the palmar surface of the hand that is formed by the radial artery and the deep branch of the ulna artery and lies on the anterior surface of the hand.

A 22-year-old football player suffered a wrist injury after falling on his outstretched hand. When the anatomical snuffbox is exposed in surgery, an artery is visualized crossing the fractured bone that provides a floor for this space. Which of the following arteries was most likely visualized? A. Ulnar B. Radial C. Anterior interosseous D. Posterior interosseous E. Deep palmar arch

*The answer is E.* Inability to adduct the thumb is the correct answer because the ulnar nerve travels superficial to the flexor retinaculum and innervates the adductor pollicis muscle, which adducts the thumb. Pronation of the forearm is carried out by muscles innervated by the median nerve, and abduction of the thumb is performed by muscles innervated by the median and radial nerves. Flexion and opposition of the thumb are performed by muscles innervated by the median nerve and would not be injured, as the median nerve travels deep to the flexor retinaculum.

A 22-year-old man accidentally smashes his hand through a window. He is cut across the entire length of the distal transverse crease on the anterior surface of the wrist. The cut is down to the surface of the flexor retinaculum but not into it. During physical examination which is one of the neuromuscular deficits that will be found? A. Weakened pronation of the forearm B. Inability to abduct the thumb C. Weakened flexion of thumb D. Weakened opposition of the thumb E. Inability to adduct the thumb

*The answer is E.* The deep branch of the ulnar nerve arises at the level of the pisiform bone and passes between the pisiform and the hook of the hamate; hence the deep branch of the ulnar nerve is most likely to be injured in this patient. The median nerve enters the forearm between the humeral and ulnar heads of the pronator teres muscle then becomes superficial near the wrist. The recurrent branch of the median nerve branches off after the median nerve enters the palm through the carpal tunnel. The radial nerve divides into superficial and deep branches when it enters the cubital fossa.

A 22-year-old woman is admitted to the hospital after falling from a tree. Radiographic examination reveals fractured pisiform and hamate bones. Which of the following nerves will most likely be injured? A. Median B. Recurrent median C. Radial D. Anterior interosseous E. Deep ulnar

*The answer is C.* The median cubital vein is a superficial vein that lies on the biceps brachii aponeurosis. The biceps brachii aponeurosis, also known as lacertus fibrosus, is a flat sheet of connective tissue that fans out from the medial side of the biceps brachii tendon to blend with the deep fascia of the biceps brachii muscle. It reinforces the cubital fossa and protects the brachial artery, which runs beneath it.

A 22-year-old woman who is in training to become a phlebotomist is performing venipuncture on another student. She places the needle into the median cubital vein but is unable to withdraw blood. She quickly realizes that she passed the needle completely through the vein. Which of the following structures located deep to the median cubital vein has acted as a barrier and has prevented her from puncturing an artery? A. Flexor retinaculum B. Pronator teres muscle C. Bicipital aponeurosis D. Brachioradialis muscle E. Biceps brachii tendon

*The answer is D.* The lateral X-ray reveals chronic microtrauma to the radial tuberosity, which is the insertion site for the biceps brachii muscle. In this weight- lifter, the damage to the radial tuberosity is most likely due to overuse of the biceps brachii and the concomitant stress placed upon the radial tuberosity by lifting significant weight loads. The biceps brachii muscle produces flexion of the elbow (and shoulder) and is the powerful supinator of the forearm, explaining why these actions exacerbated the pain in this patient. Choice A (Supinator) is incorrect. The supinator muscle attaches onto the proximal shaft of the radius rather than the radial tuberosity, so it would not cause damage to the radial tuberosity. Moreover, the supinator muscle acts to produce lower resistance supination and does not flex the elbow. Choice B (Brachialis) is incorrect. The brachialis muscle is a powerful flexor of the elbow. However, it is not related to the radial tuberosity as it attaches onto the coronoid process and tuberosity of the ulna. Furthermore, the brachialis muscle does not act in supination. Choice C (Pronator teres) is incorrect. The pronator teres muscle contributes somewhat to flexion of the elbow. However, it inserts onto the midshaft portion of the radius, and its main action is pronation. In this patient, pain was exacerbated by supination against resistance. Choice E (Brachioradialis) is incorrect. The brachioradials muscle is a notable flexor of the elbow. However, it attaches to the styloid process at the distal end of the radius and is not related to the radial tuberosity. Also, the brachioradialis muscle does not act in supination.

A 23-year-old competitive weight lifter goes to his physician complaining of pain in his proximal forearm. During his examination, the pain is exacerbated by flexion of the elbow and supination of the forearm against resistance. A lateral radiograph shows chronic microtrauma to the proximal radius, marked by the black arrows. Which of the following muscles attaches to, and most likely damaged, this osteological process? (A) Supinator (B) Brachialis (C) Pronator teres (D) Biceps brachii (E) Brachioradialis

*The answer is C.* The anterior interosseous nerve is a branch of the median nerve in the distal part of the cubital fossa, and it courses distally on the interosseous membrane. It supplies the deep forearm flexors, including the flexor digitorum profundus of digits 2 and 3, the flexor pollicis longus, and the pronator quadratus. Loss of this nerve would cause weakness in pronation due to denervation of the pronator quadratus. This injury would also result in inability to flex the distal interphalangeal joints of the index and middle fingers and the interphalangeal joint of the thumb due to denervation of the flexor digitorum profundus and flexor pollicis longus, respectively. This deficit would lead to the collapsed "O.K. sign" indicated in the photo. Because compromising the anterior interosseous nerve would not result in any cutaneous sensory deficits, it is this nerve that was most likely damaged by the displaced end of the left radius. Choice A (Deep branch of the radial nerve) is incorrect. The deep branch of the radial nerve arises from the radial nerve in the cubital fossa, dives deep to pierce the supinator muscle, and supplies all the muscles in the posterior compartment of the forearm. Damage to this nerve would lead to "wrist drop" in the patient due to the flexors of the forearm being unop- posed and an inability to extend at the wrist. Choice B (Super- ficial branch of the radial nerve) is incorrect. The superficial branch of the radial nerve arises from the radial nerve in the cubital fossa. This nerve is entirely a sensory nerve, supplying the dorsum of the hand and fingers. Because the patient has motor deficits, this nerve was not damaged by the displaced radius. Choice D (Median nerve proximal to the carpal tun- nel) is incorrect. The median nerve proximal to the carpal tunnel would not be injured because damaging this nerve would lead to a significant sensory loss over the thumb and the adjacent two and a half fingers in addition to motor loss to the thenar eminence (opponens pollicis, abductor pollicis brevis, and flexor pollicis brevis muscles) and the first two lumbrical muscles. So, the median nerve was not injured proximal to the carpal tunnel. Choice E (Recurrent [thenar] branch of median nerve) is incorrect. The recurrent (thenar) branch of the median nerve innervates most of the thenar muscles, including the abductor pollicis brevis, opponens pollicis, and superficial head of the flexor pollicis brevis. Cutting this nerve would lead to atrophy of the thenar muscular complex, a condition known as "ape hand." In this condition, the patient would not be able to oppose the thumb and second finger. Though this nerve has no sensory distribution, it is unlikely that it was involved due to the weakness when pronating the forearm.

A 23-year-old man was injured in a motor vehicle accident and X-rays confirmed a displaced distal radius fracture in his left forearm. Upon examination, the patient exhibits weakened pronation, weakened flexion of the index and middle fingers at the distal interphalangeal joints, and weakened flexion of the interphalangeal joint of the thumb. When asked to make the "okay" sign (make a circle with the thumb and index finger), the patient is unable to make a round circle, producing a "collapsed circle" on the affected hand (see photo). No areas of sensory loss are detected. Which nerve is most likely damaged? (A) Deep branch of the radial nerve (B) Superficial branch of the radial nerve (C) Anterior interosseous nerve (D) Median nerve, proximal to the carpal tunnel (E) Recurrent branch of the median nerve

*The answer is D.* The sensory deficit occurs in the cutaneous territory of the ulnar nerve. This nerve controls most of the intrinsic muscles of the hand, including the first dorsal interosseous muscle, which controls abduction of the index finger. The four dorsal interosseous muscles of the hand are innervated by the deep branch of the ulnar nerve and function to abduct digits 2 to 4. Remember that two fingers (the thumb and little finger) have their own muscles dedicated to abduction. Also, remember the mne- monic "DAB," which stands for Dorsal interossei ABduct the fingers. Thus, the associated motor deficit for the ulnar nerve on this list would be loss of abduction of the index finger. Choice A (Pronation) is incorrect. Pronation is governed by the median nerve, which supplies the pronator teres and pronator quadratus muscles. Choice B (Abduction of the wrist) is incor- rect. Abduction of the wrist is controlled by the median nerve (supplying the flexor carpi radialis) and the radial nerve (sup- plying the extensor carpi radialis longus and brevis). Choice C (Extension of the wrist) is incorrect. Extension of the wrist is produced by the radial nerve acting mainly on the extensor carpi radialis longus and brevis and the extensor carpi ulnaris muscles. Choice E (Flexion of the interphalangeal joints of the index finger) is incorrect. Flexion of the interphalangeal joints of the index finger is controlled by branches of the median nerve, which supply the flexor digitorum superficialis muscle (to flex the proximal interphalangeal joint of the second fin- ger) and the flexor digitorum profundus muscle (to flex the distal interphalangeal joint of the index finger).

A 23-year-old medical student complains of loss of sensation in the skin on the medial edge of her left hand, including the entire fifth digit. The associated motor deficit probably involves weakness in which of the following? (A) Pronation (B) Abduction of the wrist (C) Extension of the wrist (D) Abduction of the index finger (E) Flexion of the interphalangeal joints of the index finger

*The answer is D.* Recurrent branch of the median is the correct answer. This nerve, which is a branch of the median nerve, is given off after the median nerve passes through the carpal tunnel. The nerve supplies the thenar muscles. The opponens pollicis muscle, which is part of the thenar muscle group, is used while buttoning a shirt, an action that requires thumb opposition. The deep branch of the ulnar nerve supplies motor innervations to all the intrinsic muscles of the hand except the lateral two lumbricals and sensation to the medial one and a half fingers on both the palmar and dorsal sides. The patient can still grip a paper between the second and third digits, a func- tion largely performed by the interossei muscles, which are innervated by the deep branch of the ulnar nerve. The deep branch of the radial nerve is motor to the long extensors of the wrist and fingers.

A 24-year-old man complains of inability to button his shirt. Examination reveals that he can still grip a sheet of paper between his second and third fingers and there is no sensory deficit in the hand. Which nerve has been affected? A. Deep branch of ulnar B. Anterior interosseous C. Median D. Recurrent branch of median E. Deep branch of radial

*The answer is A.* The hallmark fracture caused by a fall on an outstretched hand is a scaphoid-lunate fracture; the scaphoid and lunate are the two wrist bones most proximal to the styloid process of the radius. All the other wrist bones are less likely to be affected by this injury.

A 54-year-old woman is admitted to the hospital after falling from a tree with an outstretched hand. Radiographic examination reveals a wrist dislocation. Which of the following carpal bones will most likely be involved? A. Scaphoid-lunate B. Trapezoid-trapezium C. Hamate-lunate D. Pisiform-triquetrum E. Hamate-capitate

*The answer is E.* Loss of ulnar nerve innervations eventually leads to atrophy of the interossei muscles, which pre- sents as guttering between the metacarpals. Since the median nerve is intact there is no thenar atrophy. Similarly, if the wrist extensors, which are supplied by radial nerve, are intact, then no wrist drop is observed. Radial deviation is not seen due to action of the extensor carpi ulnaris supplied by the radial nerve.

A 24-year-old man sustained multiple injuries including complex fractures in the right wrist as the result of a motor vehicle collision. After his injuries were stabilized and surgical repairs planned, neurological testing reveals decreased sensation along the medial border of the hand, as well as the little and ring fingers, and decreased strength of thumb adduction and finger adduction/abduction but with intact flexion of the distal interphalangeal joints of the ring and little fingers. If the nerve injury is not repaired, which of the following will become apparent in the affected hand over the next few weeks to months? A. Flattening of the thenar eminence B. Wrist drop C. Radial deviation at the wrist D. Ulnar deviation at the metacarpophalangeal joints E. Prominent metacarpal bones with "guttering" between adjacent metacarpals

*The answer is A.* The deficits describe ulnar nerve damage close to its entry into the forearm. The ulnar nerve passes behind the medial epicondyle and is relatively unprotected, making this area prone to nerve injury. In the forearm, via its muscular branches, it innervates the flexor carpi ulnaris muscle and the medial half of the flexor digitorum profundus muscle. In the hand the deep branch of the ulnar nerve innervates the hypothenar muscles, adductor pollicis, abductor digiti minimi, flexor digiti minimi brevis, third and fourth lumbricals, opponens digiti minimi, and pal- maris brevis muscles. The sensory innervation is to the fifth and medial half of the fourth digit and corresponding part of the hand, which can explain the deficits experienced by the patient.

A 25-year-old man falls on a slippery trail and injures his elbow and hand. Inspection reveals abrasions over the olecranon, medial epicondyle, and palm of the hand. Physical examination reveals decreased sensation with "pins and needles" (paraesthesia) along the ulnar border of the hand and medial one and a half digits. There is also weakness of finger abduction/adduction, thumb adduction, and flexion at the DIP of the ring and little fingers. Which structure was most likely injured? A. Ulnar nerve at the medial epicondyle B. Ulnar nerve at Guyon's canal C. Median nerve in the cubital fossa D. Median nerve in the carpal tunnel E. Medial cord of brachial plexus in the axillary inlet

*The answer is D.* Compression on the inferior trunk of the brachial plexus compresses nerves C8 and T1. These nerves contribute to the medial cutaneous nerve of the arm (C8, T1) and the medial cutaneous nerve of the forearm (C8, T1). They also contribute to the median, medial pectoral, ulnar, and radial nerves. This patient has thoracic outlet syndrome, which causes compression of the inferior trunk of the brachial plexus usually by the presence of a cervical rib. Compression of the ulnar nerve at the medial epicondyle, radial nerve at the neck of the radius, or median nerve in the carpal tunnel would cause motor deficits not present in this patient.

A 25-year-old woman experiences numbness and tingling in her right arm and hand while carrying a piece of luggage. Physical examination showed no motor or sensory deficits in the upper limb. When asked to abduct her upper limb to 90 degrees and to maintain this position while repeatedly closing and opening her hands, the symptoms are reproduced along the medial border of the limb, from the axilla to the hand. Which nerve structure(s) is/are most likely compressed? A. Ulnar nerve at the medial epicondyle B. Radial nerve at the neck of the radius C. Median nerve in the carpal tunnel D. Inferior trunk of the brachial plexus E. Divisions of the brachial plexus

*The answer is D.* In thoracic outlet syndrome—sometimes caused by a cervical rib or a cervical band—ventral rami or trunks of the brachial plexus can be com- pressed by these structures as they travel from the neck to the axilla. In this case the inferior trunk of the brachial plexus is being compressed by a cervical rib. The anterior division of the inferior trunk contin- ues as the medial cord of the brachial plexus. The medial brachial cutaneous nerve (medial cutaneous nerve of the arm) and medial antebrachial cutaneous nerve (medial cutaneous nerve of the forearm) are branches of the medial cord of the plexus, with the ulnar nerve as its terminal branch. Additionally, there is medial cord contribution to the median nerve. Compression of the inferior cord of the brachial plexus therefore presents with numbness and paraesthesia on the medial part of the arm, forearm, and hand.

A 25-year-old woman experiences numbness and tingling in her right arm and hand while carrying a piece of luggage. Physical examination showed no motor or sensory deficits in the upper limb. When asked to abduct her upper limb to 90 degrees and to maintain this position while repeatedly closing and opening her hands, the symptoms are reproduced along the medial border of the limb, from the axilla to the hand. Which nerve structure(s) is/are most likely compressed? A. Ulnar nerve at the medial epicondyle B. Radial nerve at the neck of the radius C. Median nerve in the carpal tunnel D. Inferior trunk of the brachial plexus E. Divisions of the brachial plexus

*The answer is B.* The brachioradialis reflex is elicited by tapping the tendon of the brachioradialis muscle. The reflex involves spinal nerves C5, C6, and C7. The major contribution is from C6.

A 54-year-old woman is found unconscious in her bed. She is admitted to the hospital, and during physical examination she has absence of her brachioradialis reflex. The ventral ramus of which spinal nerve is responsible for this reflex? A. C5 B. C6 C. C7 D. C8 E. T1

*The answer is B.* The muscle that attaches into the intertuber- cular sulcus of the humerus is the latissimus dorsi. Nerve supply is via the thoracodorsal nerve, which is a branch of the posterior cord and is made up of roots C6-8. Nerves C2, C3 and C4 are not part of the bra- chial plexus but of the cervical plexus and will supply the "strap" muscles. Nerves C4 and C5 are the main contributions to the phrenic nerve, and C5 does not contribute to the formation of the thoracodorsal nerve.

A 28-year-old telephone company worker falls off a street pole during a telephone line repair and lands directly on his right shoulder. Plain radiographs reveal a vertical fracture through the entire length of the floor of the intertubercular sulcus of the right humerus. The muscle that is most likely affected by the fracture is innervated by a nerve that is composed of which of the following nerve roots? A. C3 and C4 B. C6 to C8 C. C4 and C5 D. C2 to C4 E. C5 to C7

*The answer is A.* The nerve responsible for innervation of the interosseus muscles that are weakened in this patient is the deep branch of the ulnar nerve. Innervation of the muscles responsible for opposition of the thumb is via the recurrent branch of the median nerve. Both of these nerves are formed by the C8 and T1 ventral rami, which combine to form the inferior trunk of the brachial plexus. Damage to either the median or ulnar nerves would not produce both of these symptoms. Median nerve damage would involve all of the flexors of the wrist except the flexor carpi ulnaris and most digits except for the interphalangeal joints of the 4th and 5th fingers. It will also result in loss of function of the thumb entirely. Ulnar nerve damage will result in weakness of the medial half of flexor digitorum profundus (4th and 5th interphalangeal joint flexion), as well as the intrinsic muscles of the hand except for the lateral two lumbricals.

A 29-year-old man presents with difficulty with fine motor control in his hand. A few weeks ago he fell from a ladder; as he was falling he reached out and grabbed a limb of a tree. Examination reveals a deficit in his ability to abduct and adduct his digits and inability to oppose his thumb on his right hand. Which of the following was most likely injured? A. Lower trunk of the brachial plexus B. Median nerve C. Musculocutaneous nerve D. Ulnar nerve E. Upper trunk of the brachial plexus

*The answer is B.* The patient can extend his forearm, which suggests that the triceps brachii muscle is not weakened. Supination appears to be weak along with hand grasp and wrist drop. This would indicate that part of the radial nerve has been lost below the innervation of the triceps brachii and above the branches to the supinator and extensors in the forearm. However, sensation on the forearm and hand is intact, indicating that the superficial branch of the radial nerve is intact. The superficial branch of the radial nerve separates from the deep radial nerve at the distal third of the humerus. The posterior cord of the brachial plexus is responsible for providing innervation of the axially and radial nerves. This patient does have some radial nerve innervation and no loss of axillary nerve function. The patient does not have weakened adduction of the wrist, indicating that the ulnar nerve is not injured. If both the radial and musculocutaneous nerves are injured, supination would not be possible as the supinator and biceps brachii muscles provide supination of the forearm.

A 32-year-old man is admitted to the emergency department after a severe car crash. Radiographic examination reveals multiple fractures of his right upper limb. A surgical procedure is performed and metallic plates are attached to various bony fragments to restore the anatomy. Five months postoperatively the patient visits the outpatient clinic. Upon physical examination the patient can abduct his arm and extend the forearm, and the sensation of the forearm and hand is intact; however, hand grasp is very weak, and he cannot extend his wrist against gravity. Which of the following nerves was most likely injured during the surgical procedure? A. Posterior cord of the brachial plexus B. Radial nerve at the distal third of the humerus C. Radial and ulnar D. Radial, ulnar, and median E. Radial and musculocutaneous

*The answer is D.* A cervical rib (usually found at C7) may cause thoracic outlet syndrome, which is a condition characterized by weak muscle tone in the hand and loss of radial pulse when the upper limb is abducted above the shoulder. The mechanism of injury with the gun being fired overhead suggests a lower trunk injury to the brachial plexus. The axillary artery supplies the shoulder muscles, and there is no loss of function to these muscles. The upper trunk of the brachial plexus also supplies innervation to the shoulder muscles, which are unaffected based on the patient's present- ing abnormalities. The subclavian artery is located anterior to the brachial plexus until the plexus separates into cords as it passes under the clavicle. The brachiocephalic artery and lower trunk of the brachial plexus is only partially correct; the brachiocephalic artery is not directly associated with the brachial plexus due to its location at the midline of the body behind the sternum.

A 32-year-old man who is an expert target shooter reports pain in his right upper limb and slight tingling and numbness of all digits of the ipsilateral hand. However, the tingling and numbness of the fourth and fifth digits is the most severe. The man states that the problem usually occurs when he is firing his gun with his hand overhead. Radiographic studies reveal the presence of a cervical rib and accessory scalene musculature. Which of the following structures is most likely being compressed? A. Axillary artery B. Upper trunk of brachial plexus C. Subclavian artery D. Lower trunk of brachial plexus E. Brachiocephalic artery and lower trunk of brachial plexus

*The answer is B.* "Benediction attitude" of the hand with the index and long fingers straight and the ring and little fingers flexed is caused by an injury to the median nerve. The long flexors of the digits are supplied by the median nerve; the unopposed radial nerve and deep ulnar nerve supply the extensors of the digits 1-3, causing them to be in the extended position. Digits 4 and 5 are slightly flexed because the flexors of the proximal interphalangeal joints are supplied by the ulnar nerve.

A 32-year-old woman visits the outpatient clinic after injuring her elbow falling from her bicycle. Physical examination reveals a "benediction attitude" of the hand with the index and long fingers extended and the ring and little fingers flexed. Which of the following is the most likely diagnosis? A. Injury to median and radial nerves B. Injury to median nerve C. Injury to radial and ulnar nerves D. Injury to ulnar nerve E. Injury to median ulnar and radial nerves

*The answer is A.* The ulnar nerve innervates the dorsal and palmar interossei, which act to abduct and adduct the fingers and assist the lumbricals in their actions of flexing the metacarpophalangeal joints and extending the interphalangeal joints. The recurrent branch of the median nerve innervates the thenar muscle group that functions in the movement of the thumb. The radial and musculocutaneous nerves do not innervate any muscles in the hand. The anterior interosseous innervates the flexor pollicis longus and the pronator quadratus.

A 34-year-old man is admitted to the hospital after a car collision. Radiographic examination reveals a fracture at his wrist. Physical examination reveals paralysis of the muscles that act to extend the interphalangeal joints (Fig. 6-9). Which of the following nerves is most likely injured? A. Ulnar B. Recurrent branch of median C. Radial D. Musculocutaneous E. Anterior interosseous

*The answer is E.* The ulnar nerve enters the hand superficial to the flexor retinaculum and lateral to the pisiform bone and innervates all the interossei via the deep branch. These muscles are responsible for adduction and abduction of the fingers. Flexion of the fingers is spared because the flexor digitorum superficialis and most of the flexor digitorum profundus are innervated by the median nerve, which is unaffected by this injury. Had the median nerve been compressed in the carpal tunnel, one would have difficulty with motion of the thumb as a result of a lack of innervation of the thenar muscles. An injury of the radial nerve in the arm results in extension deficit in the forearm and hand.

A 34-year-old man visits the outpatient clinic with a painful upper limb after a fall onto a concrete floor. Physical examination reveals that the patient has weak abduction and adduction of his fingers but has no difficulty in flexing them. The patient also has decreased sensation over the palmar surface of the fourth and fifth fingers. Which of the following diagnoses is most likely? A. Compression of the median nerve in the carpal tunnel B. Injury of the radial nerve from fractured humerus in the radial tuberosity C. Compression of the median nerve as it passes between the two heads of the pronator teres D. Compression of the radial nerve from the supinator E. Injury of the ulnar nerve by a fractured pisiform

*The answer is A.* The superior trunk of the brachial plexus includes C5 and C6, which give rise to the suprascapular nerve, which innervates the supraspinatus muscle. The supraspinatus muscle is the primary muscle involved in abduction of the arm from 0 to 15 degrees. The deltoid muscle, supplied primarily by C5, abducts the arm from 15 to 90 degrees. The middle trunk is just C7 and has nothing to do with the muscle involved in initial abduction of the arm. The inferior trunk is C8-T1 and does not supply the supraspinatus muscle; therefore, it is not the right answer. The cords are distal to the branching of the supraspinatus muscle; therefore, neither lateral cord nor medial cord is the correct answer.

A 34-year-old woman is admitted to the emergency department after a car crash. Radiographic studies show marked edema and hematoma of the arm, but there are no fractures. During physical examination the patient presents with inability to abduct her arm without first establishing lateral momentum of the limb, and inability to flex the elbow and shoulder. Which of the following portions of the brachial plexus is most likely injured? A. Superior trunk B. Middle trunk C. Inferior trunk D. Lateral cord E. Medial cord

*The answer is E.* The superficial branch of the radial nerve is entirely cutaneous, carrying sensation from the dorsolateral part of the hand from the anatomical snuffbox to the midline of the fourth finger. This nerve is vulnerable as it runs posteriorly between the brachioradialis and extensor carpi radialis longus tendons toward the dorsum of the hand. This nerve was damaged at this location, but the area of sensory loss is less than expected due to the overlap from cutaneous branches of the ulnar and median nerves. Choice A (Dorsal cutaneous branch of the ulnar nerve) is incorrect. The dorsal cutaneous branch of the ulnar nerve passes posterior between the ulna and flexor carpi ulnaris to supply the subcutaneous tissue of the dorsal aspect of the pos- teromedial aspect of the hand, medial to the midline of the fourth finger. Its medial location and its sensory distribution make it an unlikely choice to be involved with this patient. Choice B (Lateral cutaneous nerve of the forearm) is incorrect. The lateral cutaneous nerve of the forearm is the continuation of the musculocutaneous nerve, which exits the arm between the biceps brachii and brachialis muscles. It supplies the skin on the lateral aspects of the forearm and wrist but would not be responsible for the numbness and paresthesia seen in this patient because the sensory deficit is distal to this nerve's normal distribution. Choice C (Posterior cutaneous nerve of the forearm) is incorrect. The posterior cutaneous nerve of the forearm arises from the radial nerve and passes in close prox- imity to the lateral intermuscular septum of the arm near the origin of the brachioradialis. As its name implies, it supplies the posterior aspect of the forearm, but it does not extend past the wrist. Therefore, it would not be the nerve damaged in this patient. Choice D (Deep branch of the radial nerve) is incorrect. The deep branch of the radial nerve is entirely motor in its distribution to the posterior muscles of the forearm. Its lack of cutaneous innervation makes this choice easy to eliminate.

A 36-year-old man broke a window with his fist to rescue his child from a house fire. The man sustained a laceration to the lateral aspect of his right forearm, but he only showed a sensory deficit (numbness and paresthesia) to the dorsolateral aspect of his hand (as denoted by the shaded area within the given photo). What nerve was most likely damaged? (A) Dorsal cutaneous branch of the ulnar nerve (B) Lateral cutaneous nerve of the forearm (C) Posterior cutaneous nerve of the forearm (D) Deep branch of the radial nerve (E) Superficial branch of the radial nerve

*The answer is B.* The biceps brachii reflex is elicited by tapping on the tendon of the biceps near its insertion on the radius. The biceps brachii reflex involves C5 and C6 spinal nerves. C5 provides the motor component; C6 the afferent side of the reflex arc.

A 54-year-old woman is found unconscious in her car. She is admitted to the hospital, and during physical examination her biceps brachii reflex is absent. What is the spinal level of the afferent component of this reflex? A. C5 B. C6 C. C7 D. C8 E. T1

*The answer is E.* The ulnar nerve is responsible for cutaneous innervation to the medial one and a half digits and motor innervation to most of the intrinsic muscles of the hand including the interossei. The interossei muscles are responsible for adduction of the digits, which is the action that would be used to hold a piece of paper between the fingers. The median nerve sup- plies cutaneous innervation to the lateral three and a half fingers and the thenar eminence and lateral two lumbricals. These muscles function to oppose the thumb and flex the MP joints, respectively. The musculocutaneous nerve is responsible for innervation of the anterior compartment of the arm, and muscular nerve fibers of this nerve would not be damaged by a wound in the distal forearm. The radial nerve supplies the dorsum of the hand, with sensation and extension function of the forearm muscles, and damage will not lead to this array of symptoms

A 36-year-old man is brought to the emergency department because of a deep knife wound on the medial side of his distal forearm. He is unable to hold a piece of paper between his fingers and has lost sensation from the fifth digit and the medial side of the fourth digit. Which of the following nerves is most likely injured? A. Axillary B. Median C. Musculocutaneous D. Radial E. Ulnar

*The answer is D.* The supraspinatus is innervated by the suprascapular nerve (C5, C6) and the nerve continues through the spinoglenoid notch and innervates the infraspinatus. The supraspinatus initiates abduction of the arm up to the first 15 to 20 degrees. The subscapular nerve supplies the subscapularis and teres major muscles, which are medial rotators of the arm. The axillary nerve supplies the deltoid and teres minor muscles and also a patch of skin on the lateral side of the shoulder. The deltoid abducts the arm beyond 20 degrees, and the teres minor muscle, although a lateral rotator, does not abduct the arm. The radial nerve supplies muscles in the posterior compartments of the arm and forearm, which are extensors of the elbow, wrist, and fingers in that order. The upper subscapular nerve supplies the subscapularis, a medial rotator of the arm.

A 36-year-old woman is admitted to the emergency department after an athletic injury that has caused weakness in both lateral rotation and the initial 15 to 20 degrees of abduction of the arm. Which nerve was most probably injured? A. Lower subscapular B. Axillary C. Radial D. Suprascapular E. Upper subscapular

*The answer is E.* The proximal segment of the scaphoid bone has a poor supply of blood because the palmar carpal branch of the radial artery enters the distal part of the scaphoid and then supplies blood proximally. This small artery is often severed during fractures of the scaphoid bone leading to avascular necrosis, or death of the bone due to poor blood supply. Remember, the scaphoid is the most commonly fractured carpal bone, but a fracture within this bone is often not seen on initial radiographs. Radiographs taken several weeks later will show the fracture due to bone resorption at the fracture site. Choice A (Distal radius) is incorrect. The distal radius is often fractured in older individuals who fall on their outstretched hand. This fracture of the distal radius is called a Colles fracture ("dinner fork deformity") due to its appearance on a lateral radiograph. However, the distal radius has an adequate blood supply and is not prone to avascular necrosis. Choice B (Midshaft ulna) is incorrect. A fracture of the midshaft ulna is not prone to avascular necrosis due to its adequate blood supply. Choice C (Fifth metacarpal) is incorrect. The fifth metacarpal is often fractured when an individual improperly punches a solid object with a clenched fist, as in a "boxer's fracture." This bone is not prone to avascular necrosis due to its adequate blood supply. Choice D (Lunate) is incorrect. The lunate is the most commonly dislocated carpal bone, which leads to severe carpal instabilities. This dislocation often occurs in association with a trans-scaphoid fracture. If this bone were fractured in this work-related accident, it would not be prone to avascular necrosis due to its adequate blood supply.

A 37-year-old factory worker fractures multiple bones distal to the elbow when his hand and forearm are crushed by equipment dropped by a faulty hydraulic lift. Which of the following bones, if fractured, would most likely develop avascular necrosis? (A) Distal radius (B) Midshaft ulna (C) Fifth metacarpal (D) Lunate (E) Scaphoid

*The answer is B.* The lunate bone is the most commonly dislocated carpal bone. Displacement is almost always anteriorly. Dislocation of the lunate bone can precipitate the signs associated typically with carpal tunnel syndrome.

A 54-year-old woman presents with pain in her right wrist that resulted when she fell forcefully on her outstretched hand. Radiographic studies indicate an anterior dislocation of a carpal bone of the proximal row (see Fig. 6-4). Which of the following bones is most commonly dislocated? A. Capitate B. Lunate C. Scaphoid D. Pisiform E. Triquetrum

*The answer is D.* The supinator muscle attaches to the radius proximally and when fractured would cause a lateral deviation. The pronator teres muscle originates on the medial epicondyle and coronoid process of the ulna and inserts onto the middle of the lateral side of the radius, pulling the radius medially below the fracture. The pronator quadratus muscle originates on the ante- rior surface of the distal ulna and inserts on the ante- rior surface of the distal radius, pulling the radius medially. The brachioradialis muscle originates on the lateral supracondylar ridge of the humerus and inserts at the base of the radial styloid process, far below the fracture. The brachialis muscle originates in the lower anterior surface of the humerus and inserts in the coronoid process and ulnar tuberosity, hence not causing an action on the radius.

A 41-year-old woman is admitted to the hospital after a car crash. Radiographic examination reveals a transverse fracture of the radius proximal to the attachment of the pronator teres muscle. The proximal portion of the radius is deviated laterally. Which of the following muscles will most likely be responsible for this deviation? A. Pronator teres B. Pronator quadratus C. Brachialis D. Supinator E. Brachioradialis

*The answer is B.* Injury to the superior trunk of the brachial plexus can damage nerve fibers going to the supra- scapular, axillary, and musculocutaneous nerves. Damage to the suprascapular and axillary nerves causes impaired abduction and lateral rotation of the arm. Damage to the musculocutaneous nerve causes impaired flexion of the forearm. A winged scapula would be caused by damage to the long thoracic nerve. The long thoracic nerve is formed from spinal cord levels C5, C6, and C7, so the serratus anterior muscle would be weakened from the damage to C5 and C6, but the muscle would not be completely paralyzed. The intrinsic muscles of the hand are innervated by the ulnar nerve, which would most likely remain intact. Paraesthesia in the medial aspect of the arm would be caused by damage to the medial brachial cutaneous nerve (C8-T1; inferior trunk). Loss of sensation on the dorsum of the hand would be caused by damage to either the ulnar or radial nerves (C6 to T1).

A 42-year-old woman is admitted to the hospital with injury to the upper (superior) trunk of the brachial plexus. The diagnosis is Erb-Duchenne palsy. Which of the following conditions is expected to be present during physical examination? A. Winged scapula B. Inability to laterally rotate the arm C. Paralysis of intrinsic muscles of the hand D. Paraesthesia in the medial aspect of the arm E. Loss of sensation in the dorsum of the hand

*The answer is A.* Tinel's sign is used to aid in the diagnosis of carpal tunnel syndrome. It is performed by lightly percussing above the carpal tunnel where the median nerve is located. De Quervain's tenosynovitis describes tenosynovitis of the sheath or tunnel that surrounds tendons that control the thumb. It is tested using Finkelstein's test, where the examiner grips the thumb of the individual being tested and ulnar deviates the hand sharply. Thoracic outlet syndrome is tested using Adson's test. Mallet finger describes a finger deformity due to extensor digitorum tendons. Radial nerve damage is tested by evaluating the cutaneous distribution of the radial nerve or by testing the muscles innervated by the radial nerve.

A 43-year-old woman visits the outpatient clinic with a painful hand. During physical examination, percussion over the flexor retinaculum causes a sharp pain in the lateral three and a half digits. This sign is indicative of which of the following conditions? A. Carpal tunnel syndrome B. De Quervain's tenosynovitis C. Thoracic outlet syndrome D. Mallet finger E. Radial nerve damage

*The answer is C.* The triceps brachii muscle is innervated by the radial nerve (primarily C7), which comes off C5 to T1 spinal nerves. Because the patient's only motor deficit involves the triceps brachii muscles, one can rule out C5 and C6, which supply fibers to the axillary, musculocutaneous, and upper subscapular nerves. Damage to either of these ventral rami would result in additional motor deficits of the shoulder and flexor compartment of the arm. One can also rule out C8-T1 because these ventral rami form the medial pectoral nerve and the medial brachial and antebrachial cutaneous nerves. Damage to these ventral rami would result in loss of pectoral muscle function and cutaneous sensation over the medial surface of the upper limb.

A 45-year-old woman is admitted to the hospital with neck pain. An MRI examination reveals a herniated disc in the cervical region. Physical examination reveals weak triceps brachii muscle. Which of the following spinal nerves is most likely injured? A. C5 B. C6 C. C7 D. C8 E. T1

*The answer is D.* The radial nerve is particularly vulnerable during midshaft humeral fractures because it is located directly against the bone in this region. The radial nerve innervates the extensor muscles of the arm and forearm and also carries sensory innervation from the posterior aspect of the arm, forearm, and hand and the lateral aspect of the arm (but not the forearm). Depending upon the exact placement of the fracture, the nerve fibers that inner- vate the triceps brachii may have already left the radial nerve. However, the fibers innervating the extensors of the wrist and fingers would still be bundled in the radial nerve and would be vulnerable to damage during a fracture of this type. Choice A (Numbness on the lateral [radial] aspect of the forearm) is incorrect. The radial nerve innervates the skin on the posterior aspect of the arm and forearm and the lateral aspect of the lower arm. The lateral (radial) aspect of the forearm is innervated primarily by the lateral cutaneous nerve of the forearm, the terminal branch of the musculocutaneous nerve. Choice B (Numbness of the medial aspect of the upper arm) is incorrect. The skin over the medial aspect of the upper arm is innervated by the medial cutaneous nerve of the arm (medial brachial cutaneous nerve) that originates from the medial cord of the brachial plexus. This nerve would be protected from the humeral fracture due to its location. Choice C (Numb- ness over the superolateral aspect of the upper arm) is incor- rect. The cutaneous innervation of the superolateral aspect of the upper arm is derived from the axillary nerve that shares a common origin with the radial nerve from the posterior cord of the brachial plexus. Though both nerves arise from the posterior cord, the axillary and radial nerves separate proximal to the site of the fracture, thus leaving the axillary nerve undamaged by the fracture. Choice E (Weakness in grip strength) is incorrect. While the pain associated with the fracture would affect the integrity of all muscle activity in the limb, all of the muscles used for gripping (flexing the wrist and digits) are innervated by the median and ulnar nerves, which do not have close relations to the fracture site.

A 48-year-old woman falls on an icy sidewalk and lands on her right elbow. She suffers a midshaft humeral fracture, as seen on the given X-ray. The attending physician wants to assess whether the nerve residing in the spiral groove of the humerus is damaged. What sign or symptom would confirm damage to this nerve? (A) Numbness on the lateral (radial) aspect of the forearm (B) Numbness of the medial aspect of the upper arm (C) Numbness over the superolateral aspect of the upper arm (D) Weakness extending the wrist and fi ngers (E) Weakness in grip strength

*The answer is C.* Carpal tunnel syndrome is a relatively common condition that causes pain, numbness, and a tingling sensation in the hand and fingers. Carpal tunnel syndrome is caused by compression of the median nerve, which supplies the thenar muscles and the first and second lumbricals. Dorsal and palmar interossei and the hypothenar muscles are supplied by the ulnar nerve. The flexor muscles of the forearm are supplied by the median nerve before it passes through the carpal tunnel.

A 48-year-old woman is seen in the orthopedic clinic with symptoms of carpal tunnel syndrome. This could result in weakening of which muscles? A. Dorsal and palmar interossei B. Lumbricals III and IV C. Thenar and lumbricals I and II D. Flexor digitorum superficialis and profundus E. Hypothenar

*The answer is B.* This woman has experienced a lower brachial plexus injury due to forced abduction of the upper limb during the accident. This injury presents with numbness and paresthesia in the C8 and T1 dermatomes, which supply the axilla and medial aspect of her upper limb. These nerve roots primarily supply the medial cord of the brachial plexus, which creates the ulnar nerve. Due to damage to the ulnar nerve, she is experiencing weakness in the movement of her left hand. The abduction and adduction of the fingers are controlled by the deep branch of the ulnar nerve by supplying the dorsal interosseous and palmar interosseous muscles, respectively. Choice A (Upper trunk of the brachial plexus) is incorrect. Damage to the upper trunk of the brachial plexus results in Erb-Duchenne palsy ("waiter's tip malformation"). In this injury, the patient presents with significant weakness in abduction and lateral rotation of the shoulder, flexion of the shoulder and elbow, and supination of the forearm. Choice C (Posterior cord of the brachial plexus) is incorrect. The posterior cord of the brachial plexus gives rise to the axillary and radial nerves. Damage to the axillary nerve causes weakness in abduction of the shoulder due to loss of the deltoid muscle. Damage to the radial nerve results in inability to extend at the elbow or wrist (leading to "wrist drop"). However, this patient had significant weakness in the movements of the hand, which implies damage to the contributions of the ulnar nerve. Choice D (Lateral cord of the brachial plexus) is incorrect. Damage to the lateral cord of the brachial plexus results in signs and symptoms similar to those seen in Erb-Duchenne palsy, or the "waiter's tip malformation." In this injury, the patient presents with significant weakness in abduction and lateral rotation of the shoulder, flexion of the shoulder and elbow, and supination of the forearm. Choice E (Long tho- racic nerve) is incorrect. A lesion of the long thoracic nerve leads to a "winged scapula" due to the subsequent paralysis of the serratus anterior muscle. Additionally, the affected arm cannot be abducted above the horizontal plane because the serratus anterior is not available to superiorly rotate the glenoid cavity of the scapula to allow full abduction. This deficit was not seen in this patient.

A 50-year female equestrian is thrown from a startled horse and dragged by the reins, which were wrapped around her left wrist, for some distance. At the ER, she is experiencing pain and paresthesia in the axilla and medial aspect of her upper limb. Despite being left-handed, she has marked weakness in the movements of her dominant hand, especially abduction and adduction of the fingers. What structure was most likely damaged in this woman? (A) Upper trunk of the brachial plexus (B) Lower trunk of the brachial plexus (C) Posterior cord of the brachial plexus (D) Lateral cord of the brachial plexus (E) Long thoracic nerve

*The answer is A.* The illustration shows an injury in which the cervicobrachial angle (the angle between the neck and shoulder) is stretched widely. This abnormal impact eventually results in the postural presentation of a "waiter's tip" deformity (Erb-Duchenne palsy). This combination of injury and postural deformity is related to damage to both the C5 and C6 roots or upper trunk of the brachial plexus. The C5 and C6 roots converge to form the upper trunk of the brachial plexus and contribute heavily to the suprascapular, axillary, and musculocutaneous nerves. The suprascapular nerve supplies the supraspinatus and infraspinatus muscles. The axillary nerve controls the deltoid and teres minor muscles. The musculocutaneous nerve supplies the anterior compartment of the arm (coracobrachialis, biceps brachii, brachialis muscles). Therefore, a significant weakness in abduction and lateral rotation of the shoulder, flexion of the shoulder and elbow, and supination of the forearm would result from this brachial plexus injury. The ultimate postural deformity is a contracture effect in which the intact muscles act unopposed to draw the limb into a position that is the opposite of the actions of the affected muscles. Choice B (Lower roots of the brachial plexus) is incorrect. The lower roots (C8, T1) have a strong projection into the ulnar nerve. Trauma here would result in an ultimate postural deformity of "claw hand" due to the loss of flexion of the medial digits. That injury does not match this patient's clinical presentation. Choice C (Posterior divisions of the brachial plexus) is incorrect. The posterior divisions supply the radial, axillary, upper and lower subscapular, and thoracodorsal nerves. The primary postural effect resulting from trauma here would be a case of "wrist drop" expressed due to loss of the extensor muscles innervated by the radial nerve. Choice D (Medial cord of the brachial plexus) is incorrect. The medial cord projects into the ulnar and median nerves. Damage to the ulnar nerve will result in "claw hand."

A 50-year old man falls off a ladder while cleaning his windows, landing on the ground as seen in the given drawing. He does not seek medical aid, believing his general soreness will go away with time. However, after several months, he develops a postural deformity of his left upper limb that includes an adducted, medially rotated, and extended shoulder, extended elbow, and pronated forearm. The injury and subsequent condition reflect damage to what structure? (A) Upper trunk of the brachial plexus (B) Lower roots of the brachial plexus (C) Posterior divisions of the brachial plexus (D) Medial cord of the brachial plexus (E) Lateral root of the median nerve

*The answer is B.* The posterior interosseus nerve innervates the extensors of the wrist, abductor pollicis longus, extensor indicis, digiti minimi, and extensor pollicis longus muscles. The posterior interosseus nerve does not have any cutaneous branches, making it the best answer. The anterior interosseous nerve innervates flexors of the forearm. Although the radial nerve does give rise to the posterior interosseous nerve, there are no sensory deficits mentioned, so the radial nerve proper was not affected. The ulnar nerve also innervates flexors in the hand but since no sensory deficits were noted ulnar nerve injury can be ruled out. The superficial radial nerve is a purely cutaneous nerve.

A 52-year-old man is admitted to the emergency department after a fall. Imaging studies show a fracture at the neck of the radius and a hematoma at the fracture site. Examination reveals weakness of wrist extension, abduction and extension of the thumb, and extension of the metacarpophalangeal and interphalangeal joints of the fingers. However, there was no sensory deficit. Which nerve is most likely affected? A. Anterior interosseous B. Posterior interosseous C. Radial D. Ulnar E. Superficial radial

*The answer is B.* The posterior interosseous nerve is an extension of the deep branch of the radial nerve after it emerges distal to the supinator. It is responsible for innervation of several muscles in the extensor compartment of the posterior aspect of the forearm, including extension of the metacarpophalangeal joints. The deep radial nerve courses laterally around the radius and passes between the two heads of the supinator muscle and is thus likely to be compressed by a hematoma between the fractured radius and the supinator muscle. Though the radial nerve gives rise to the posterior interosseous nerve, this answer choice is too general and would not indicate the precise injured branch of the radial nerve. Both the deep branch of the ulnar nerve and the median nerve traverse the medial and anteromedial aspect of the arm, respectively. These nerves primarily supply the flexor compartment of the arm. The anterior interosseous nerve is a branch of the median nerve and supplies the flexor digitorum profundus, flexor pollicis longus, and the pronator quadratus muscles.

A 52-year-old man is admitted to the emergency department after falling on wet pavement. Radiographic examination reveals fracture of the radius. An MRI study reveals a hematoma between the fractured radius and supinator muscle. Upon physical examination the patient has weakened abduction of the thumb and extension of the metacarpophalangeal joints of the fingers. Which of the following nerves is most likely affected? A. Anterior interosseous B. Posterior interosseous C. Radial nerve D. Deep branch of ulnar nerve E. Median nerve

*The answer is B.* The ulnar nerve can become compressed between the pisiform and hook of the hamate at the wrist in a condition termed "ulnar canal syndrome" or "Guyon tunnel syndrome." This entrapment syndrome is especially seen in professional cyclists who spend countless hours placing pressure on the hook of the hamate bone as they grasp their handlebars. This "handle- bar neuropathy" presents with hyperextension of the meta- carpophalangeal joints and flexion at the interphalangeal joints of the fourth and fifth fingers. The "clawing" of these two fingers is accompanied by sensory loss in the medial side of the hand. Choice A (Ulnar nerve in the elbow) is incorrect. The ulnar nerve crosses the elbow in a narrow space between the olecranon process and the medial epicondyle of the humerus, on the posteromedial (ulnar) side of the joint. Compression of the ulnar nerve at this location leads to "cubital tunnel syndrome" and presents with hand deficits similar to those seen in this patient. However, this patient does not exhibit weakness in flexion or adduction of the wrist. Sparing of these actions indicates that damage to the ulnar nerve must be distal to the elbow. Choice C (Median nerve in the wrist) is incorrect. The median nerve is often compressed at the wrist in "carpal tunnel syndrome" because it travels deep to the transverse carpal ligament (flexor retinaculum of the wrist) within the carpal tun- nel. This entrapment syndrome presents with paresthesia of the lateral fingers as well as an inability to oppose the thumb and a wasting of the thenar eminence. Damage to the median nerve would not lead to the postural deformity seen in the medial two fingers in this patient. Choice D (Median nerve in the elbow) is incorrect. The median nerve is not usually compressed by forced flexion of the elbow because it lies loosely on the flexor surface of the joint deep to the bicipital aponeurosis. The median nerve can be compressed in the proximal forearm as it passes between the two heads of the pronator teres. However, damage to the median nerve would not lead to this patient's postural deformity in the medial two fingers. Choice E (Median nerve in the axilla) is incorrect. The median nerve would not be damaged in the axilla without significant trauma. Damage to the median nerve in the axilla would lead to weakness in flexing the wrist, loss of pronation of the forearm, and wasting of the thenar muscles. However, these signs and symptoms were not reported.

A 52-year-old retired professional cyclist, who still rides his bike 400 miles per week, comes to his physician complaining of hand problems. The physician notes hyperextension of the ring and little fingers at the metacarpophalangeal joints and flexion at the interphalangeal joints within the same fingers (see photo). During examination, the patient has no weakness in flexion or adduction of the wrist. What nerve is compressed at what location? (A) Ulnar nerve in the elbow (B) Ulnar nerve in the wrist (C) Median nerve in the wrist (D) Median nerve in the elbow (E) Median nerve in the axilla

*The answer is C.* The anterior interosseous nerve runs distally and anterior to the interosseous membrane supplying the deep forearm flexors (except the ulnar part of th flexor digitorum profundus muscle, which sends tendons to the fourth and fifth fingers), it passes deep to and supplies the pronator quadratus muscle, hence the weakness in pronation and wrist flexion

A 54-year-old man presents to his primary care physician complaining of weakness in his fingers. His attempt to make a ring between his thumb and index finger by bringing the tips together is shown in Figure 6-11. He is able to successfully hold a piece of paper between his thumb and index finger. Pronation and wrist flexion are weakened. Which of the following nerves is most likely affected? A. Ulnar nerve at Guyon's canal B. Median nerve in the carpal tunnel C. Anterior interosseous nerve beneath the ulnar head of pronator teres D. Posterior interosseous nerve beneath the supinator E. Median nerve beneath the bicipital aponeurosis

*The answer is D.* In the midshaft region of the humerus the radial nerve runs in the radial groove; fracture of the humerus at this point will likely impinge directly on the radial nerve, producing a sensory deficit along the posterior aspect of the forearm. The lateral aspect of the forearm is innervated by the lateral antebrachial cutaneous nerve of the forearm, which comes from the musculocutaneous nerve. These nerves may not be affected by a midshaft fracture of the humerus because they are well separated from the bone by muscle. The medial aspect of the arm and forearm is supplied by the intercostobrachial nerve and the medial antebrachial cutaneous nerve that takes its origin from the medial cord of the brachial plexus where it runs superficially, making it extremely difficult to injure both nerves during a midshaft fracture of the humerus. The lateral and posterior aspect of the forearm is an unlikely choice because the displaced bone not only has to impinge on the radial nerve but must also affect the very superficially located lateral antebrachial cutaneous nerve as well.

A 54-year-old woman is admitted to the emergency department after a serious motor vehicle accident. Physical examination shows soft tissue edema and bruising around the neck. A radiograph of the humeroscapular region reveals a fracture of the midhumerus. Which of the following areas will most likely have impaired or absent sensation? A. Lateral aspect of the forearm B. Medial aspect of the arm C. Medial aspect of the arm and forearm D. Posterior aspect of the forearm E. Lateral and posterior aspect of the forearm

*The answer is D.* The lateral epicondyle is the common extensor origin. Most of the extensor muscles of the forearm originate from this area. Putting those muscles in action will exacerbate pain on the lateral epicondyle, a condition nicknamed "tennis elbow." Radial and lateral deviations have no effect because the movement is at the wrist joint. Flexion exacerbates pain on the medial epicondyle if the patient has "golfer's elbow."

A 55-year-old right-handed woman presents to the clinic with a 1-week history of right elbow pain. The pain started after a long game of competitive tennis. The pain begins in the elbow and at times radiates into the forearm. Splinting of the elbow decreases the intensity of the pain. During physical examination of the elbow mild swelling and tenderness are noted over the lateral epicondyle. Which one of the following wrist movements, if carried out by the patient with a closed fist and against resistance, will most likely exacerbate the pain? A. Radial deviation B. Ulnar deviation C. Flexion D. Extension E. Flexion and ulnar deviation

*The answer is A.* The suprascapular artery arises as a major branch of the thyrocervical trunk from the subclavian artery. It has rich anastomoses with the circumflex scapular artery and could provide essential blood supply to the scapula. The dorsal scapular artery would be lost with the graft. None of the other vessels listed is in position to provide adequate supply to the scapula.

A 56-year-old woman is admitted to the hospital after a severe car crash. A large portion of her chest wall needed to be surgically removed and replaced with a musculo-osseous scapular graft involving the medial border of the scapula. Which of the following arteries will most likely recompensate the blood supply to the entire scapula? A. Suprascapular B. Dorsal scapular artery C. Posterior circumflex humeral artery D. Lateral thoracic E. Supreme thoracic artery

*The answer is C.* The scaphoid and lunate carpal bones have a direct articulation with the radius, which is fractured in a Colles' fracture; therefore, they would most likely be disrupted or fractured. The other carpal bones listed do not have direct contact with the radius and have a more distal location; therefore, they would not be as likely to be injured with a Colles' fracture.

A 56-year-old woman visits the emergency department after falling on a wet pavement. Radiographic examination reveals osteoporosis and a Colles' fracture. Which of the following carpal bones are often fractured or dislocated with a Colles' fracture? A. Triquetrum and scaphoid B. Triquetrum and lunate C. Scaphoid and lunate D. Triquetrum, lunate, and scaphoid E. Triquetrum and pisiform

*The answer is A.* Situations in which peripheral nerves are compressed or otherwise entrapped where they pass through narrow spaces ("tunnels") in muscles and/or osseo-fascial units are generally referred to as "tunnel syndromes." Such conditions may result in periodic or constant motor and/or sensory deficits. The ulnar nerve crosses the elbow in a narrow space between the olecranon process and the medial epicondyle of the humerus, on the posteromedial (ulnar) side of the joint. The ulnar nerve can be compressed between these bony landmarks or between the humeral and ulnar heads of the attachment of the flexor carpi ulnaris. Compression of the ulnar nerve within these areas leads to "cubital tunnel syndrome." The symptoms are exacerbated during events where flexion of the elbow narrows these passageways and compresses the ulnar nerve at the elbow joint. The patient's cubital tunnel syndrome would explain the paresthesia and numbness on the medial aspect of the hand and the diminished fine motor control of the intrinsic hand muscles. Her condition originated due to the forced flexion of the elbow in the motor vehicle accident, which compressed the ulnar nerve in the cubital tunnel. Choice B (Ulnar nerve in the wrist) is incorrect. The ulnar nerve can be compressed between the pisiform and hook of the hamate at the wrist in a condition termed "ulnar canal syndrome" or "Guyon tunnel syndrome." This entrapment syndrome presents with similar signs and symptoms as seen in this patient. However, the ability to flex the wrist would not be affected. In this patient, trauma to the wrist was not reported. Choice C (Median nerve in the wrist) is incorrect. The median nerve is often compressed at the wrist in "carpal tunnel syndrome" because this nerve travels deep to the transverse carpal ligament (flexor retinaculum of the wrist) within the carpal tunnel. This entrapment syndrome presents with paresthesia of the lateral fingers as well as an inability to oppose the thumb and a wasting of the thenar eminence. These symptoms were not reported in this patient. Choice D (Median nerve in the elbow) is incorrect. The median nerve is not usually compressed by forced flexion of the elbow because it lies loosely on the flexor surface of the joint deep to the bicipital aponeurosis. The median nerve can be compressed in the proximal forearm as it passes between the two heads of the pronator teres. However, the symptoms in this patient were due to forced flexion of the elbow and involve the ulnar nerve. Choice E (Median nerve in the axilla) is incorrect. The median nerve would not be damaged in the axilla without significant trauma. Damage to the median nerve in the axilla would lead to weakness in flexing the wrist, loss of pronation of the forearm, and wasting of the thenar muscles. However, these signs and symptoms were not reported.

A 56-year-old woman was stopped at a light when her car was rear-ended by another car. She had her right arm on the steering wheel, and the impact caused forced flexion at her elbow. Several months later, she comes to her physician complaining of numbness and a "pins and needles" sensation in her right little finger when she talks on the phone, rests her head on her right hand at work, or spends most of her day typing at work. She also notices the quality of her typing and her ability to play the violin have diminished. Which nerve is compressed at what location? (A) Ulnar nerve in the elbow (B) Ulnar nerve in the wrist (C) Median nerve in the wrist (D) Median nerve in the elbow (E) Median nerve in the axilla

*The answer is B.* The radial artery enters the palm through the anatomic snuffbox. The artery then moves on to pierce through the two heads of the first dorsal inter- osseous muscle and enter the deep aspect of the palm. The flexor pollicis longus tendon runs on the palmar aspect of the hand and the radial artery runs on the dorsal aspect of the hand before entering the deep aspect of the palm, and therefore the radial artery does not run below this tendon. The radial artery does not run between the first and second interosseous muscle and therefore cannot be used as a landmark to identify the artery. Finally, the artery does not run between the first dorsal interosseous muscle and the adductor pollicis longus.

A 59-year-old woman is admitted to the hospital in a state of shock. During physical examination, several lacerations are noted in her forearm and her radial pulse is absent. Where is the most typical place to identify the radial artery immediately after crossing the radiocarpal joint? A. Between the two heads of the first dorsal interosseous muscle B. In the anatomic snuffbox C. Below the tendon of the flexor pollicis longus D. Between the first and second dorsal interossei muscles E. Between the first dorsal interosseous muscle and the adductor pollicis longus

*The answer is C.* Injury to the radial nerve can be caused by a blow to the midhumeral region because the nerve winds around the shaft of the humerus. The symptoms described include the loss of wrist and finger extension and a loss of sensation in an area of skin supplied by the radial nerve.

A 61-year-old man was hit by a cricket bat in the midhumeral region of his left arm. Physical examination reveals normal elbow motion; however, he could not extend his wrist or his metacarpophalangeal joints and he reported a loss of sensation on a small area of skin on the dorsum of the hand proximal to the first two digits. Radiographic examination reveals a hairline fracture of the shaft of the humerus just distal to its midpoint. Which of the following nerves is most likely injured? A. Median B. Ulnar C. Radial D. Musculocutaneous E. Axillary

*The answer is D.* The ulnar nerve enters the forearm by passing between the two heads of the flexor carpi ulnaris and descends between and innervates the flexor carpi ulnaris and flexor digitorum profundus (medial half) muscles. It enters the hand superficial to the flexor retinaculum and lateral to the pisiform bone, where it is vulnerable to damage and provides the deep ulnar branch. The deep branch of the radial nerve arises proximally in the forearm.

A 62-year-old man visits the outpatient clinic with pain after falling on his outstretched hand. Radiographic examination reveals a fracture of the pisiform bone and hematoma of the surrounding area. Which of the following nerves will most likely be affected? A. Ulnar B. Radial C. Median D. Deep ulnar E. Deep radial

*The answer is A.* The patient has a classic claw hand due to the damage of the deep branch of the ulnar nerve by the fractured hamate at Guyon's canal. This nerve supplies the intrinsic muscles of the hand except the lateral two lumbricals and the thenar muscles. This nerve also supplies cutaneous innervations to the medial one and a half fingers (ring and little fingers) in the palmar and dorsal sides. The lumbricals and interossei insert at the back of the fingers via the dorsal (extensor) hood. This hood extends from the metacarpophalangeal joint to the distal phalanx. Through this mechanism, the muscles flex the meta- carpophalangeal joint and extend the interphalangeal joint. With damage to the deep branch of the ulnar nerve, this function is lost. The result is that there will be flexion of the interphalangeal joints and extension of the metacarpophalangeal joint, giving the appearance as shown in the photograph.

A 62-year-old woman is seen in the outpatient clinic. A photograph of her hand is shown in Figure 6-10. A radiograph reveals a hairline fracture of the hamate at Guyon's canal. Which of the following will also be present during physical examination? A. Numbness and weakness of the little and ring fingers B. Wrist drop C. Atrophy of the thenar muscles D. Positive Tinel's test E. Trouble turning her forearm outward

*The answer is B.* The axilla is a large, pyramidal space between the side of the chest and the upper part of the brachium. Its major importance is as a passageway from the root of the neck to the upper limb. The axilla is demarcated by four walls: anterior, posterior, medial, lateral. The posterior wall is composed of the latissimus dorsi, teres major, and subscapularis muscles. The posterior axillary fold forms the palpable lower margin of the wall and is composed of the latissimus dorsi and teres major. The subscapularis is not part of the posterior axillary fold. Damage to the latissimus dorsi would severely hinder adduction, extension, and medial rotation of the arm. Choice A (Pectoralis minor) is incorrect. The pectoralis minor contributes to the formation of the anterior wall of the axilla. The pectoralis major forms the bulk of the anterior wall and creates the noticeable ante- rior axillary fold. Choice C (Levator scapulae) is incorrect. The levator scapulae muscle passes out of the neck to attach onto the superior angle of the scapula. It is far removed from any of the walls of the axilla and does not contribute to rotation and adduction of the arm. Choice D (Serratus anterior) is incorrect. The serratus anterior lies against the thoracic wall and forms much of the medial wall of the axilla along with the thoracic wall. It fixes the scapula to the thoracic wall and has no affect on rotation of the arm. Choice E (Teres minor) is incorrect. The teres minor is located immediately above the teres major. However, it is not a component of the posterior wall of the axilla and normally contributes to lateral rotation of the arm.

A 65-year-old man is brought to the emergency room after being attacked in his office by a disgruntled co-worker. The attacker reportedly used a long, narrow-bladed letter-opener to inflict multiple stab wounds to the man's back. Physical examination shows a puncture wound in the posterior axillary fold. The patient presents with weakness in extension, adduction, and medial rotation of his arm. Which of the following muscles is most likely cut in this injury? (A) Pectoralis minor (B) Latissimus dorsi (C) Levator scapulae (D) Serratus anterior (E) Teres minor

*The answer is B.* The radiograph shows a fracture of the humerus at the surgical neck. The bruising and dimpling of the upper arm would result from this injury. The axillary nerve leaves the brachial plexus as a terminal branch of the posterior cord. It passes through the quadrangular space and wraps around the head of the humerus on its way to provide inner- vation to the teres minor, the deltoid, and the portion of skin over the lower aspect of the deltoid that is known as the "sergeant's patch." The radial nerve travels in the radial groove along the shaft of the humerus and would be injured in a fracture of the shaft of the humerus. The ulnar nerve would be injured in a fracture of the medial epicondyle. The median nerve travels too deep to be injured here and could be compressed at the carpal tunnel or at the cubital fossa. The musculocutaneous nerve is likewise within the tissue and will not be affected by this injury.

A 67-year-old woman with osteoporosis injured her left shoulder/arm in a fall. Examination reveals bruising and dimpling of the upper part of the arm with exquisite tenderness over the affected area. The shoulder radiograph is shown in Figure 6-13. Which nerve is most likely to be injured? A. Radial B. Axillary C. Ulnar D. Median E. Musculocutaneous

*The answer is A.* The median nerve supplies sensory innervation to the thumb, index, and middle fingers as well as to the lateral half of the ring finger. The median nerve also provides motor innervation to muscles of the thenar eminence. Compression of the median nerve in the carpal tunnel explains these deficits in conjunction with normal functioning of the flexor compartment of the forearm. The ulnar nerve is not implicated in these symptoms. Compression of the brachial plexus could not be attributed to pressure from hypertrophy of the triceps brachii muscle, it is located distal to the plexus. In addition, symptoms would include several upper limb deficits rather than the focal symptoms described in this instance. Osteo- arthritis of the cervical spine would also lead to increasing complexity of symptoms.

A 69-year-old man has numbness and pain in the middle three digits of his right hand at night. He retired 9 years ago after working as a carpenter for 30 years. He has atrophy of the thenar eminence (see Fig. 6-3). Which of the following conditions will be the most likely cause of this atrophy? A. Compression of the median nerve in the carpal tunnel B. Formation of the osteophytes that compress the ulnar nerve at the ulnar condyle C. Hypertrophy of the triceps brachii muscle compressing the brachial plexus D. Osteoarthritis of the cervical spine E. Repeated trauma to the ulnar nerve

*The answer is D.* The median nerve provides innervation to most of the muscles in the flexor compartment of the forearm; cutaneous innervation of the second, third, and fourth digits and palmar and dorsum aspects of the hand; and innervation of four intrinsic hand muscles: first and second lumbricals, abductor pollicis brevis, opponens pollicis, and flexor pollicis brevis. The thenar compartment contains the abductor pollicis brevis, opponens pollicis, and flexor pollicis brevis muscles, and these muscles are innervated by the recurrent branch of the median nerve. The patient has weakening of the first two lumbricals and not simply the thenar muscles, so the median nerve is most likely to be compressed. Another indication that the median nerve is compressed is the vigorous shaking of the wrist. Because the median nerve tranverses the carpal tunnel, carpal tunnel compression could lead to this action on part of the patient. The ulnar nerve provides innervation for part of the flexor digitorum profundus and flexor carpi ulnaris muscles. These muscles are not weakened in this patient. The radial nerve provides cutaneous supply to the dorsum of the hand and forearm as well as extensor muscles of the forearm. The posterior interosseous nerve is a branch of the radial nerve and provides innervation of the extensor muscles in the forearm.

A 69-year-old woman visits the outpatient clinic with a complaint of numbness and tingling of her hand for the past 3 months. Physical examination reveals she has numbness and pain in the lateral three digits of her right hand that are relieved by vigorous shaking of the wrist. In addition, the abductor pollicis brevis, opponens pollicis, and the first two lumbrical muscles are weakened. Sensation was decreased over the lateral palm and the volar aspect of the first three digits. Which of the following nerves is most likely compressed? A. Ulnar B. Radial C. Recurrent median D. Median E. Posterior interosseous

*The answer is D.* The radial and ulnar arteries anastomose to form the superficial and deep palmar arches. However, the superficial palmar arch is formed mainly by the ulnar artery, whereas the deep palmar arch is formed primarily by the radial artery. Thus, the insufficiency in this case is most likely due to reduced flow in the radial artery. Choice A (Posterior interosseous artery) is incorrect. The posterior interosseous artery branches off the common interosseous artery, which is derived from the ulnar artery in the proximal forearm. The posterior interosseous artery supplies the posterior compartment of the forearm. This artery does not reach the wrist and does not contribute blood to the palmar arterial arches. Choice B (Ulnar artery) is incorrect. The ulnar artery provides the primary supply into the superficial pal- mar arterial arch. The deep palmar arch is formed primarily by the radial artery, and it is the reduced flow of the radial artery most likely present in this patient. Choice C (Anterior interosseous artery) is incorrect. The anterior interosseous artery branches off the common interosseous artery, which is derived from the ulnar artery in the proximal forearm. The anterior interosseous artery supplies the anterior compart-ment of the forearm as well as the distal posterior forearm via its posterior terminal branch. This artery does reach the wrist, but it does not contribute blood to the palmar arterial arches. Choice E (Inferior ulnar collateral artery) is incorrect. This artery branches off the brachial artery just above the elbow. It passes distally across the anterior side of the medial epicon- dyle of the humerus to form a collateral connection with the anterior ulnar recurrent branch of the ulnar artery. This vessel is far removed from the hand.

A 74-year-old man complains of pain in his right hand and fingers when he works with his hands for a while. Thorough testing reveals insufficient blood flow into the deep palmar arch. Occlusion of which of the following arteries is the most likely cause of this condition? (A) Posterior interosseous artery (B) Ulnar artery (C) Anterior interosseous artery (D) Radial artery (E) Inferior ulnar collateral artery

*The answer is A.* The accessory nerve (CN XI) traverses the posterior triangle of the neck to reach the deep surface of the trapezius muscle after it innervates the sternocleidomastoid muscle. Its position within the triangle is superficial, and it is at this location that this nerve is vulnerable to injury. Damage to the distal accessory nerve would inhibit elevation of the scapula and lateral rotation of the scapula during abduction greater than 90 degrees. Both of these actions were affected in this patient. Choice B (Axillary nerve) is incorrect. The axillary nerve travels through the quadrangular space to reach the deltoid and teres minor muscles. While a lesion of this nerve would affect abduction of the arm, the axillary nerve is not found in the neck, instead arising distally from the posterior cord of the brachial plexus. Moreover, dam- age to the axillary nerve would cause a sensory deficit in the upper lateral arm, which was not reported. Choice C (Dorsal scapular nerve) is incorrect. This nerve courses into the upper, medial part of the back and the lower neck to supply the levator scapulae and rhomboid muscles. Paralysis of these muscles would result in weakness in elevation and retraction of the scapula and perhaps wasting of the contour of the back under the trapezius muscle. Choice D (Long thoracic nerve) is incor- rect. The long thoracic nerve is a branch of the upper three roots (C5-C7) of the brachial plexus. It descends along the medial wall of the axilla and travels superficial to the serratus anterior muscle, which it innervates. This nerve is coalescing in the deep, most inferior region of the neck, so it would not be injured in the location of the incision. Damage to the long thoracic nerve would lead to "winging of the scapula," when the patient is asked to protract the affected shoulder. Choice E (Thoracodorsal nerve) is incorrect. This nerve runs inferiorly through the axilla to supply the latissimus dorsi muscle. Loss of the nerve would result in weakness in extension and medial rotation of the arm, plus wasting of the posterior axillary fold.

A dermatologist performed a biopsy on a suspicious mole on the right side of the posterior neck of a 57-year-old male construction worker. Pathology confirmed a malignant melanoma, so the physician excised a substantial amount of tissue surrounding the mole. After the procedure, the patient experienced difficulty elevating his right shoulder and lifting his right arm over his head. No sensory deficits were seen. What nerve was most likely damaged in this patient? (A) Accessory nerve (B) Axillary nerve (C) Dorsal scapular nerve (D) Long thoracic nerve (E) Thoracodorsal nerve

*The answer is E.* The superior thoracic artery is a direct branch of the axillary artery. The thoracoacromial trunk has four branches: the pectoral, clavicular, acromial, and deltoid.

A patient comes in with gunshot wound and requires surgery in which his thoracoacromial trunk needs to be ligated. Which of the following arterial branches would maintain normal blood flow? (A) Acromial (B) Pectoral (C) Clavicular (D) Deltoid (E) Superior thoracic

*The answer is D.* A lesion of the radial nerve causes paralysis of the supinator and brachioradialis. The biceps brachii muscle is a flexor of the elbow and also a strong supinator; thus, supination is still possible through action of the biceps brachii muscle. Other muscles cannot supinate the forearm

A patient has a lesion of the radial nerve. Supination is possible through contraction of which of the following muscles? (A) Supinator (B) Pronator teres (C) Brachioradialis (D) Biceps brachii (E) Supraspinatus

*The answer is E.* A positive response to the myotatic biceps reflex confirms the integrity of the musculocutaneous nerve and the C5 and C6 spinal segments, from which this nerve arises. The musculocutaneous nerve supplies motor innervation and proprioception to the muscles in the anterior compartment of the arm, including the coracobrachialis, biceps brachii (tested here), and brachialis. Lesioning the musculocutaneous nerve would lead to loss of proprioception and weakness in flexing the elbow (via the biceps brachii and brachialis muscles) and supinating the forearm (via the biceps brachii), resulting in a negative myotatic biceps reflex. Choice A (Axillary nerve) is incorrect. The axillary nerve innervates the deltoid and teres minor muscles as well as the skin overlying the deltoid in the superolateral aspect of the arm. Loss of innervation to the deltoid muscle would cause weakness in abduction of the upper limb. The teres minor and posterior head of the deltoid are responsible for external rotation of the shoulder. Choice B (Median nerve) is incorrect. The median nerve innervates 6.5 of the 8 anterior forearm muscles. This nerve would help with flexion of the forearm at the elbow; however, the myotatic biceps reflex test specifically tests the innervation of the biceps brachii muscle, which is innervated by the musculocutaneous nerve. Choice C (Ulnar nerve) is incorrect. The ulnar nerve innervates only 1.5 muscles of the forearm, specifically the flexor carpi ulnaris and the ulnar side of the flexor digitorum profundus. This nerve also innervates most of the intrinsic mus- cles of the hand. It is not involved with the myotatic biceps reflex test. Choice D (Radial nerve) is incorrect. The radial nerve supplies motor innervation to the posterior compart- ments of the arm and forearm. Damage to this nerve would cause weakness in extension at the elbow and wrist joints. The radial nerve is responsible for the posterior cutaneous nerves of the arm and forearm as well as the inferior lateral cutaneous nerve of the arm. However, it is responsible for extension at the elbow, not the flexion of the elbow tested with the myotatic biceps reflex test.

A physician tests the myotatic biceps reflex as shown. A normal response of involuntary contraction of the biceps brachii muscle is noted. This reflex confirms the integrity of what nerve? (A) Axillary nerve (B) Median nerve (C) Ulnar nerve (D) Radial nerve (E) Musculocutaneous nerve

*The answer is A.* Flexion of the elbow is achieved by contraction of the biceps brachii, brachialis, and brachioradialis muscles. The brachialis muscle is the major flexor of the elbow joint and together with the brachioradialis will continue to achieve flexion if the biceps brachii is damaged. The flexor carpi ulnaris and radialis produce flexion of the wrist, and the flexor digitorum superficialis and profundus produce flexion of the digits at the metacarpophalangeal and interphalangeal joints, respectively. The pronator teres and supinator are responsible for pronation and supination, respectively. The coracobrachialis does not cross the elbow joint and acts only on the shoulder, while the triceps brachii is the elbow extensor.

After a fall on her outstretched arm, a 72-year-old woman presents with elbow pain. Physical examination reveals a palpable defect over her biceps brachii tendon. Elbow flexion causes pain but does not limit activemovement. Radiographs do not show fractures or dislocations. She is diagnosed with a biceps brachii tendon rupture. Which of the following muscles most likely allow the patient to continue to flex her elbow? A. Brachialis and brachioradialis B. Flexor carpi ulnaris and flexor carpi radialis C. Flexor digitorum superficialis and flexor digitorum profundus D. Pronator teres and supinator E. Triceps brachii and coracobrachialis

*The answer is C.* The photo demonstrates a case of "winged scapula," indicative of a lesion of the long thoracic nerve and subsequent paralysis of the serratus anterior muscle. The nerve runs down the lateral thoracic wall, on the superficial aspect of the serratus anterior, where it is unusually exposed (for a motor nerve) and vulnerable to injury, especially when the limb is elevated. Lesion of the nerve denervates the serratus anterior. This results in the medial border and inferior angle of the scapula pulling away from the posterior chest wall, giving the scapula a wing-like appearance when the affected limb is protracted. Additionally, the affected arm cannot be abducted above the horizontal plane because the serratus anterior is not available to superiorly rotate the glenoid cavity of the scapula to allow full abduction. Choice A (Axillary nerve) is incorrect. This nerve passes deeply through the axilla, around the surgical neck of the humerus, to supply the teres minor and deltoid muscles. Lesion here would result in significant weakness in abduction of the arm and wasting of the rounded contour of the shoulder. Choice B (Thoracodorsal nerve) is incorrect. This nerve runs inferior through the axilla to supply the latissimus dorsi muscle. Loss of the nerve would result in weakness in extension and medial rotation of the arm, plus wasting of the posterior axillary fold. Choice D (Dorsal scapular nerve) is incorrect. This nerve courses into the upper, medial part of the back and the lower neck to supply the levator scapulae and rhomboid muscles. Paralysis of these muscles would result in weakness in elevation and retraction of the scapula and perhaps wasting of the contour of the back under the trapezius muscle. Choice E (Suprascapular nerve) is incorrect. The suprascapular nerve runs through the suprascapular notch and into the supraspinous and infraspinous fossae to supply the supraspinatus and infraspinatus muscles. Lesion of this nerve would result in weakness in the rotator cuff affecting the initiation of abduction (supraspinatus) and external rotation (infraspinatus) of the shoulder and wasting of the muscular contour of the posterior aspect of the scapula.

An 18-year-old boy is cut severely on the lateral wall of his right chest during a knife fight. Following healing, his scapula moves away from the thoracic wall when he leans on his right hand, giving the appearance indicated in the given photo. Which of the following nerves is likely damaged? (A) Axillary nerve (B) Thoracodorsal nerve (C) Long thoracic nerve (D) Dorsal scapular nerve (E) Suprascapular nerve

*The answer is C.* The patient likely has damage to the ulnar nerve, which affected both the interossei and medial two lumbricals. The lumbricals extend the interphalangeal joints of the ring and little fingers, while the interossei are responsible for abduction and adduction of the digits. The dorsal interossei are responsible for abduction, while the palmar interossei are responsible for adduction of the digits. The extensor digit minimi is responsible for extension of the little finger only and if damaged will not affect the ring finger. If the extensor digitorum were damaged it would lead to weakness of all four digits, not only the ring and little fingers.

An emergency department physician evaluates a 28-year-old man who injured his hand in a knife fight. The physician notes that the ring and little fingers cannot be extended at the interphalangeal joints, and the patient cannot spread the fingers of his injured hand. Weakness of which of the following muscles is the major reason for the loss of interphalangeal extension of the medial two fingers? A. Dorsal interosseus muscles B. Extensor digitorum C. Lumbrical muscles D. Palmar interosseus muscles E. Extensor digiti minimi

*The answer is A.* The greater tubercle of the humerus is the insertion site of three (of the four) rotator cuff muscles: supraspinatus, infraspinatus, and teres minor. Avulsion of this structure could result in detachment of any of these rotator cuff muscles, depending upon the size and scope of the fracture. However, the wrestler is unable to initiate abduction of the upper limb, which implies damage to the supraspinatus muscle. Choice B (Long head of biceps brachii) is incorrect. This muscle originates from the supraglenoid tubercle of the scapula and passes between the greater and lesser tubercles of the humerus, in the intertubercular (bicipital) groove. Detachment of the tendon of this muscle causes the biceps brachii to bulge in the anterior arm. Avulsion of the biceps brachii muscle is not related to the greater tubercle of the humerus. Choice C (Long head of triceps) is incorrect. The long head of the triceps brachii muscle originates from the infraglenoid tubercle of the scapula and inserts on the olecranon process of the ulna. It would not be involved in avulsion of the greater tubercle of the humerus. Choice D (Subscapularis) is incorrect. The fourth rotator cuff muscle, subscapularis, inserts onto the lesser tubercle of the humerus, so it would not be directly involved with this avulsion injury. Choice E (Infraspinatus) is incorrect. The infraspinatus muscle does insert onto the middle aspect of the greater tubercle of the humerus; however, damage to this muscle would result in weakness in external rotation at the shoulder joint, not the problems with abduction seen in this patient.

As a result of chronic stress associated with an intense high school weight-lifting program, a 15-year-old boy suffers an avulsion fracture of the greater tubercle of the humerus. In the ER, he displays difficulty initiating abduction of the upper limb. Which of the following muscles was involved in this fracture? (A) Supraspinatus (B) Long head of biceps brachii (C) Long head of triceps (D) Subscapularis (E) Infraspinatus

*The answer is D.* The deep branch of the ulnar nerve innervates most of the intrinsic muscles of the hand, including the hypothenar muscles, medial two lumbrical muscles, the palmar and dorsal interossei, adductor pollicis, and the deep head of the flexor pollicis brevis. Specifically, the palmar interossei, dorsal interossei, and lum- brical muscles insert into the extensor digital expansion to provide, collectively, flexion of the metacarpophalangeal joints and extension of the interphalangeal joints of the fingers. All of these muscles, with the exception of the first and second lumbrical muscles, are innervated by the deep branch of the ulnar nerve, so this nerve is being tested by placing the hand in the Z-position. If the deep branch of the ulnar nerve were severed, the hand would assume the opposite of the Z-posi- tion, which is called "claw hand." Choice A (Deep branch of radial nerve) is incorrect. The radial nerve descends from the arm into the cubital fossa, where it divides into superficial and deep branches. The deep branch of the radial nerve pierces the supinator muscle, winds around the proximal end of the radius within the substance of that muscle, and passes into the deep posterior compartment of the forearm as the posterior interosseous nerve. The nerve may be entrapped within the supinator, resulting in supinator syndrome. Such a condition would affect the deeper, more distal extensor muscles arising in the forearm and some sensory areas in the wrist joints. It is not responsible for the Z position of the hand. Choice B (Superficial branch of radial nerve) is incorrect. The superfi- cial branch of the radial nerve arises from the radial nerve in the cubital fossa. This nerve is purely a sensory nerve, supply- ing the dorsum of the hand and fingers. Because it does not have a motor component, this nerve would not be responsible for assuming the Z position of the hand. Choice C (Recurrent branch of median nerve) is incorrect. The recurrent (thenar) branch of the median nerve innervates most of the thenar muscles, including the abductor pollicis brevis, opponens pollicis, and superficial head of the flexor pollicis brevis. Cutting this nerve would lead to atrophy of the thenar muscular complex, a condition known as "ape hand." The recurrent branch of the median nerve does not supply muscles outside of the thumb, so it would not be responsible for assuming the hand's Z position. Choice E (Superficial branch of ulnar nerve) is incorrect. The superficial branch of the ulnar nerve arises from the ulnar nerve distal to the flexor retinaculum. This nerve sup- plies cutaneous branches to the anterior surface of the medial one and a half fingers. The palmaris brevis is the only muscle supplied by this nerve, and this muscle tightens the skin of the medial surface of the palm. Therefore, this nerve would not be responsible for the Z position of the hand.

As part of a physical examination to evaluate intrinsic hand muscle function, a physician asks the patient to assume the Z-position (seen in photo) with his hand, which involves flexion of the metacarpophalangeal joints and extension of the interphalangeal joints of the fingers. Which of the following nerves is being tested in assuming this position? (A) Deep branch of radial nerve (B) Superfi cial branch of radial nerve (C) Recurrent branch of median nerve (D) Deep branch of ulnar nerve (E) Superfi cial branch of ulnar nerve

*The answer is E.* The FDS muscle flexes the proximal interphalangeal joints in digits 2 to 5. In this patient, this muscle is being tested in the third (middle) finger. By holding the other fingers in extension, the physician is eliminating the influences of the FDP muscles, which flex the distal interphalangeal joints in digits 2 to 5. Choice A (Flexor digitorum profundus [FDP]) is incorrect. Flexion of the distal interphalangeal joint in digits 2 to 5 is produced by the FDP; however, the physician is testing flexion of the proximal interphalangeal joint in the middle finger, which is controlled by the FDS muscle. Choice B (Extensor digitorum) is incorrect. The extensor digitorum extends fingers #2 to #5. This muscle can easily be eliminated because the physician is testing a muscle on the flexor side of the hand. Choice C (Second lumbrical) is incorrect. The second lumbri- cal muscle extends the interphalangeal joints of the middle (third) finger and flexes the metacarpophalangeal joint of the same finger. The third lumbrical is an intrinsic hand muscle that arises off the tendon of the FDP and inserts into the exten- sor expansion of the middle finger. This muscle does not con- trol flexion of the proximal interphalangeal joint. Choice D (Dorsal interosseous) is incorrect. The primary action of the dorsal interosseous muscles is to abduct the fingers. However, because these muscles insert into the extensor expansion, they also extend the interphalangeal joints of the middle (third) finger and flex the metacarpophalangeal joint of the same finger. Although the dorsal interossei are not involved with flexion of the distal interphalangeal joint, it is important to remember the middle (third) finger forms the axis of movement of the hand. Therefore, this finger has two dorsal interossei attached to it to enable abduction away from the midline in either direction.

As part of a physical examination to evaluate intrinsic hand muscle function, a physician holds three fingers in the extended position, and instructs the patient to flex the proximal interphalangeal joint of the free finger, as shown. Which of the following muscles is the doctor specifically testing? (A) Flexor digitorum profundus (FDP) (B) Extensor digitorum (C) Second lumbrical (D) Dorsal interosseous (E) Flexor digitorum superficialis (FDS)

*The answer is C.* The primary action of the dorsal interossei is to abduct the fingers, which is being tested in this photo. It is also important to remem- ber the dorsal interossei insert into the extensor digital expansion, so these muscles work with the palmar interos- sei and the lumbrical muscles to extend the interphalangeal joints and flex the metacarpophalangeal joints. The nerve being tested in this patient, via abduction of the fingers, is the deep branch of the ulnar nerve. A mnemonic is "DAB", which stands for Dorsal interossei ABduct the fingers. Choice A (Lumbrical Muscles) is incorrect. The lumbrical muscles arise off the flexor digitorum profundus tendons and insert into the lateral (radial) aspect of the extensor digital expansion. Due to their course and insertion, the lumbrical muscles extend the interphalangeal joints and flex the metacarpophalangeal joints. These intrinsic hand muscles are not being tested when a patient abducts the fingers against resistance. Choice B (Palmar interosseous muscles) is incorrect. The primary action of the three palmar interossei is to adduct the index finger, ring finger, and little finger toward the axis of the hand, which is the middle finger. The deep branch of the ulnar nerve supplies the palmar interossei. A mnemonic is "PAD", which stands for Palmar interossei ADduct the fingers. It is also important to remember the palmar interossei insert into the extensor digital expansion, so these muscles work in concert with the dorsal interossei and the lumbrical muscles to extend the interphalangeal joints and flex the metacarpophalangeal joints of the fingers. Choice D (Flexor digitorum superficialis) is incorrect. The flexor digitorum superficialis muscle flexes the proximal interphalangeal joint in digits 2 to 5, and influences the distal interphalangeal joint by binding the tendons of the flexor digitorum profundus muscle. Flexion of the proximal interphalangeal joints of the fingers is not being tested in this patient. Choice E (Flexor digitorum profundus) is incorrect. The flexor digitorum profundus muscle is responsible for flexion of the distal interphalangeal joint in digits 2 to 5; however, this action is not being tested in this photo.

As part of a physical examination to evaluate muscle function in the hand, a physician holds the four fingers (digits 2 through 5) and asks the patient to spread their fingers, as shown below. What muscle(s) is/are the doctor testing? (A) Lumbrical muscles (B) Palmar interosseous muscles (C) Dorsal interosseous muscles (D) Flexor digitorum superfi cialis (E) Flexor digitorum profundus

*The answer is A.* Flexion of the distal interphalangeal joint in digits 2 to 5 is produced by the FDP. The actions of this muscle are being tested in this illustration. Choice B (Extensor indicis) is incorrect. The extensor indicis extends the index finger (digit 2), which enables this finger to extend independent of the other fingers. Because the muscle arises from the distal third of the ulna and the interosseous membrane, it also acts to extend the hand at the wrist. The extensor indicis muscle is not involved in flexion of the distal interphalangeal joint, which is being tested in this patient. Choice C (First lumbrical) is incorrect. The first lumbrical muscle extends the interphalangeal joints of the index (second) finger and flexes the metacarpophalangeal joint of the same finger. The first lumbrical is an intrinsic hand muscle that arises off the tendon of the flexor digitorum profundus and inserts into the extensor expansion of the index finger. This muscle is not involved with flexion of the distal interphalangeal joint. Choice D (First dorsal interosseous) is incorrect. The primary movement of the first dorsal interosseous is abduction of the index finger. However, because it inserts into the extensor expansion, it also extends the interphalangeal joints of the index (second) finger and flexes the metacarpophalangeal joint of the same finger. This muscle is not involved with flexion of the distal interphalangeal joint. Choice E (Flexor digitorum superficialis [FDS]) is incorrect. The FDS acts at the proximal interphalangeal joint in digits 2 to 5 and influences the distal interphalangeal joint by binding the tendons of the FDP. However, when the proximal interphalangeal joint is held in extension, the influence of the FDS is eliminated, allowing testing of only the FDP.

As part of a physical examination to evaluate muscle function in the hand, a physician holds the proximal interphalangeal joint of his patient's index finger in the extended position and instructs him to try to flex the distal interphalangeal joint, as shown below. Which of the following muscles is the doctor testing? (A) Flexor digitorum profundus (FDP) (B) Extensor indicis (C) First lumbrical (D) First dorsal interosseous (E) Flexor digitorum superficialis (FDS)

*The answer is C.* The cubital fossa is a triangular intermuscular space located anterior to the elbow, comparable to the popliteal fossa in the lower limb. It is an important transition zone between the arm and the forearm, containing major blood vessels and nerves. The roof of the cubital fossa is subcutaneous tissue carrying superficial veins, such as the median cubital vein, which is the most common site for venipuncture in the upper limb. The floor of the fossa is formed by two muscles: brachialis and supinator. Either of these muscles could house the deep abscess caused by repeated needle injections in the cubital fossa. Choice A (Brachioradialis muscle) is incorrect. The brachioradialis muscle forms the lateral wall of the cubital fossa. Choice B (Pronator teres muscle) is incorrect. The pronator teres muscle forms the medial wall of the cubital fossa. Choice D (Head of the radius) is incorrect. The head of the radius lies deep to the muscular floor of the cubital fossa, within the elbow joint complex. Choice E (Olecranon fossa of the humerus) is incorrect. The olecranon fossa is located on the posterior distal aspect of the humerus, well removed from the cubital fossa.

Because of repeated bad needle sticks, a heroin addict develops an infected abscess in the floor of the cubital fossa. Which of the following structures is the abscess most likely to invade first? (A) Brachioradialis muscle (B) Pronator teres muscle (C) Brachialis muscle (D) Head of the radius (E) Olecranon fossa of the humerus

*The answer is E.* The location of the cut, superficial to the flexor retinaculum, indicates injury to the ulnar nerve. This nerve controls most of the intrinsic muscles of the hand, including the adductor pollicis. Because this muscle is the sole adductor of the thumb, the result will be inability to adduct that digit. Choice A (Weakness in pronation) is incorrect. Pronation is controlled by branches of the median nerve to the pronator teres and pronator quadratus muscles. These are located proximal to the site of the injury. Choice B (Inability to abduct the thumb) is incorrect. Abduction of the thumb is governed by the radial nerve (to abductor pollicis longus) and the recurrent branch of the median nerve (to abductor pollicis brevis) at sites removed from the injury. Choice C (Weakness in flexion of the thumb) is incorrect. Flexion of the thumb is controlled by branches of the median nerve at locations proximal (to flexor pollicis longus) and distal (to flexor pollicis brevis) to the cut flexor retinaculum (transverse carpal ligament). However, the reported laceration was superficial to this ligament and proximal to the pisiform bone, so the median nerve would not have been damaged. Choice D (Weakness in opposition of the thumb) is incorrect. Opposition of the thumb is controlled by the recurrent branch of the median nerve (to opponens pollicis), distal to the injury site.

During an attempted suicide, a depressed young woman slashes her wrist with a straight razor. She cuts just proximal to the pisiform bone to the depth of the superficial aspect of the flexor retinaculum before passing out at the sight of her own blood. As a result of this wound, she may suffer a neuromuscular deficit that results in which of the following? (A) Weakness in pronation (B) Inability to abduct the thumb (C) Weakness in flexion of the thumb (D) Weakness in opposition of the thumb (E) Inability to adduct the thumb

*The answer is E.* The flexor retinaculum (transverse carpal ligament) is a thickening of investing deep fascia on the anterior (ventral) aspect of the wrist that forms the roof of the carpal tunnel. All issues related to the carpal tunnel revolve around an understanding of what structures are contained within the carpal tunnel versus the positions of structures outside the tunnel. The tendons of the flexor carpi radialis and palmaris longus muscles, plus the ulnar nerve and ulnar artery, lie against the superficial aspect of the flexor retinaculum. Any of these structures may be lesioned by a superficial cut across the front (anterior side) of the wrist. Choice A (Flexor digitorum superficialis) is incorrect. The tendons of this muscle are contained within the carpal tun- nel, deep to the flexor retinaculum. Choice B (Brachioradialis) is incorrect. This muscle inserts onto the base of the styloid process of the radius. Thus, its tendon is proximal and lateral to the flexor retinaculum, and therefore unrelated to the car- pal tunnel. Choice C (Flexor pollicis longus) is incorrect. The tendons of this muscle are contained within the carpal tunnel, deep to the flexor retinaculum. Choice D (Abductor pollicis longus) is incorrect. This muscle is a member of the poste- rior compartment of the forearm. Its tendon loops out of that compartment to attach onto the lateral side of the base of the first metacarpal (the lateral base of the thumb). Thus, it is well removed from the flexor retinaculum and the carpal tunnel.

During an attempted suicide, a depressed young woman slashes the front of her wrist with a razor blade. However, she cuts only to the depth of the superficial aspect of the flexor retinaculum before passing out at the sight of her own blood. Which of the following muscle tendons may be severed? (A) Flexor digitorum superficialis (B) Brachioradialis (C) Flexor pollicis longus (D) Abductor pollicis longus (E) Flexor carpi radialis

*The answer is A.* Injuries to superior parts of the brachial plexus (C5-C6) usually result from an excessive in- crease in the angle between the neck and shoulder during a difficult delivery. Injury to the superior trunk of the plexus is apparent by the characteristic position of the limb ("waiter's position"), in which the limb hangs by the side in medial rotation. Injuries to the lower trunk of the brachial plexus (Klumpke paralysis) are much less common. These events injure the inferior trunk of the brachial plexus (C8 and T1) and may avulse the roots of the spinal nerves from the spinal cord. The short muscles of the hand are affected, and a claw hand results.

Following a difficult delivery, a 3-day-old infant girl showed limited movement of the right upper limb, with the arm adducted and internally/medially rotated, the forearm extended at the elbow and pronated, and the wrist slightly flexed. Tearing of fibers in which ventral rami of the brachial plexus best accounts for these symptoms? A. C5 and C6 B. C6 and C7 C. C7 and C8 D. C8 and T1 E. C5 to T1

*The answer is D.* Successful transposi- tion of muscle flaps in reconstructive surgery hinges largely on maintaining the neurovascular pedicles that supply the muscle. The latissimus dorsi muscle is innervated by the tho- racodorsal (middle subscapular) nerve, a branch of the pos- terior cord of the brachial plexus. Lesion of this nerve during dissection of the chest wall will cause loss of the muscle flap. Choice A (Long thoracic nerve) is incorrect. The long thoracic nerve supplies the serratus anterior muscle. Damage to this nerve would cause a "winged scapula." Choice B (Inter- costobrachial nerve) is incorrect. The intercostobrachial nerve is actually the lateral cutaneous branch of the second thoracic nerve and is entirely sensory in innervation. This nerve arises from the chest wall underneath the second rib and travels dis- tally to give sensory innervation to the skin of the medial arm. Choice C (Medial pectoral nerve) is incorrect. The medial pectoral nerve innervates the pectoralis minor and major muscles. It would have been cut during the removal of these muscles during the radical mastectomy. Choice E (Axillary nerve) is incorrect. The axillary nerve traverses the quadrangular space (accompanying the posterior circumflex humeral artery) to supply the teres minor and deltoid muscles as well as give cutaneous innervation to the upper lateral aspect of the arm.

Following a radical mastectomy procedure, a surgeon plans to conduct a breast reconstruction utilizing a latissimus dorsi muscle flap. What nerve will the surgeon need to keep intact during the surgical dissection of the chest wall to prevent atrophy of the muscle flap? (A) Long thoracic nerve (B) Intercostobrachial nerve (C) Medial pectoral nerve (D) Thoracodorsal nerve (E) Axillary nerve

*The answer is E.* Throwing motions are complex mechanical events that involve multiple muscles interacting in moment-to-moment changing ways, with rotation of the humerus being one important outcome. The four rotator cuff muscles (supraspinatus, infrapsinatus, teres minor, subscapu-laris), plus several other muscles that cross the glenohumeral joint, contribute significant forces to rotation of the arm. The teres minor and infraspinatus (i.e., half the rotator cuff group) lie completely across the posterior aspect of the glenohumeral joint and are primary lateral (external) rotators. These muscles are aided by the posterior fibers of the deltoid muscle. Choice A (Supraspinatus) is incorrect. The supraspinatus is one of the rotator cuff muscles. However, it does not produce rotation. It lies across the superior aspect of the glenohumeral joint and initiates abduction of the arm from the rest position. Choice B (Teres major) is incorrect. The teres major arises pos- teriorly from the inferior angle of the scapula and crosses the glenohumeral joint to its anteriorly located insertion into the medial lip of the intertubercular groove of the humerus. Thus, the teres major muscle acts to adduct and medially (internally) rotate the arm rather than laterally rotate. It is not a member of the rotator cuff muscles. Choice C (Latissimus dorsi) is incorrect. The latissimus dorsi muscle arises posteriorly from the spinous processes of the inferior six thoracic vertebrae and the thoracolumbar fascia and crosses the glenohumeral joint anteriorly to insert into the floor of the intertubercular sul- cus of the humerus. Thus, the latissimus dorsi muscle acts to extend, adduct, and medially rotate the humerus rather than laterally rotate. It is not a member of the rotator cuff muscles. Choice D (Subscapularis) is incorrect. The subscapularis mus- cle is part of the rotator cuff. In common with the teres major and latissimus dorsi muscles, it crosses the glenohumeral joint from posterior to anterior, thus producing medial rotation and adduction of the arm. It is not responsible for external (lateral) rotation of the arm.

Lateral rotation of the arm is an important mechanical component of "bringing the arm back" when preparing to throw an object. What muscle acts to produce lateral rotation of the arm? (A) Supraspinatus (B) Teres major (C) Latissimus dorsi (D) Subscapularis (E) Teres minor

*The answer is B.* The hamate bone is identified by the white arrow in the given X-ray. This bone resides in the medial (ulnar) aspect of the distal row of carpal (wrist) bones. Stress fractures can occur to this bone, particularly within its hook, which appears as a radiodense oval on this radiograph. This type of fracture is frequently seen in golfers due to the positioning of the proximal aspect of the golf club within their grip. A fracture of the hamate results in pain, which is exacer- bated by gripping and point tenderness in the skin overlying the bone. The numbness and weakness in the medial aspect of the hand, seen in this patient, are due to impingement of the ulnar nerve. Remember, the eight carpal (wrist) bones form two rows that contain four bones each. The proximal row (from lateral to medial) is composed of the scaphoid, lunate, tri- quetrum, and pisiform. The distal row (from lateral to medial) is composed of the trapezium, trapezoid, capitate, and hamate. To remember the carpal bones as listed, some students use the mnemonic: "Some Lovers Try Positions That They Can't Handle". In a standard posteroanterior plane film, the medial aspect of the wrist can be deceiving to the untrained eye. Here, the pisiform overlies the triquetrum, and the seemingly single radiopacity might not be distinguished as two separate bones. Instead, the distinctive hook of the hamate bone may be mis- taken for the pisiform. Choice A (Capitate) is incorrect. The capitate (L: head) is a head-shaped bone that is the largest of the carpal bones. Its position is noted by its articulation with the third metacarpal distally. Choice C (Lunate) is incorrect. The lunate (L: moon) is a moon-shaped bone between the scaphoid and triquetral bones. It articulates with the radius proximally, and its position makes it the most commonly dislocated carpal bone. Choice D (Pisiform) is incorrect. The pisiform (L: pea) is a pea-shaped sesamoid bone that lies on the pal- mar aspect of the triquetrum bone. Choice E (Scaphoid) is incorrect. The scaphoid (G: boat) is a boat-shaped bone that is the largest of the carpal bones located in the proximal row. Its articulation with the radius proximally makes it the most commonly fractured carpal bone, especially when a person falls and impacts an abducted, outstretched hand.

On his downswing, an amateur golfer strikes the hard earth with his club and feels pain in his right wrist. During a subsequent physical examination, he complains of wrist pain that is exacerbated by gripping, displays point tenderness in his medial wrist, and complains of numbness and weakness in his pinky finger (fifth digit). What carpal bone, identified by the white arrow on the given X-ray, is most likely fractured in this patient? (A) Capitate (B) Hamate (C) Lunate (D) Pisiform (E) Scaphoid

*The answer is A.* The median nerve controls pronation through the actions of the pronator teres and pronator quadratus muscles in the anterior compartment of the forearm. It also controls much of flexion of the wrist and lateral digits via the actions of most of the other muscles in that compartment. Choice B (Ulnar nerve) is incorrect. The ulnar nerve controls 1½ muscles in the anterior compartment of the forearm (flexor carpi ulnaris and the ulnar half of the flexor digitorum profundus) and most of the intrinsic muscles of the hand. However, neither of these 1½ forearm muscles produces pronation. Choice C (Superficial branch of the radial nerve) is incorrect. The superficial branch of the radial nerve is entirely cutaneous, carrying sensation from the dorsolateral part of the hand. So, cutting this nerve would not result in weakness in pronation of the forearm or flexion of the wrist. Choice D (Deep branch of the radial nerve) is incorrect. The deep branch of the radial nerve supplies the posterior compartment of the forearm. It influences supination via motor control of the supinator muscle, but not pronation. Choice E (Musculocutaneous nerve) is incorrect. The musculocutaneous nerve supplies the anterior compartment of the arm, and then continues distally via its termination as the lateral cutaneous nerve of the forearm. It contributes significantly to control of supination by its innervation of the biceps brachii muscle, but has no effect on wrist flexion.

Physical examination of a 40-year-old man injured in an automobile accident indicates that he has suffered nerve damage affecting his left upper limb. The patient exhibits significant weakness when pronating his left forearm and flexing his left wrist. What nerve is most likely damaged? (A) Median nerve (B) Ulnar nerve (C) Superficial branch of the radial nerve (D) Deep branch of the radial nerve (E) Musculocutaneous nerve

*The answer is D.* The quadrangular space of the shoulder is an important passageway allowing the posterior humeral circumflex vessels and their companion axillary nerve to pass from the axilla to the posterior aspect of the shoulder. The neurovascular bundle runs across the surgical neck of the humerus to enter the quadrangular space. The space itself (sometimes termed the lateral axillary hiatus) is formed by four structures: teres major, teres minor, long head of the triceps, surgical neck of the humerus. The vessels con- tribute to the collateral network around the shoulder. The axil- lary nerve supplies the teres minor and deltoid muscles and a cutaneous area on the superolateral aspect of the arm (i.e., the skin overlying the lower aspect of the deltoid muscle). Choice A (Musculocutaneous nerve) is incorrect. The musculocutaneous nerve is a terminal branch of the lateral cord of the brachial plexus within the axilla. It supplies the anterior compartment of the arm and is not related to the quadrangular space. Choice B (Lateral cord of the brachial plexus) is incor- rect. The lateral cord occupies a relatively lateral position in the axilla but is not related to the quadrangular space. The axillary nerve originates from the posterior cord of the brachial plexus. Choice C (Radial nerve) is incorrect. The radial and axillary nerves are the terminal branches of the posterior cord of the brachial plexus. The radial nerve runs through the radial (spiral) groove in the midshaft of the humerus to emerge through the lower triangular space, just below the quadrangular space, and enter the posterior compartment of the arm. Choice E (Medial cutaneous nerve of the arm) is incorrect. The medial cutaneous nerve of the arm is a branch of the medial cord of the brachial plexus. It is far removed from the quadrangular space.

Physical examination of a 45-year-old man who had been stabbed in the back of the shoulder shows a deep wound penetrating into the quadrangular space of the shoulder, causing bleeding from the severed blood vessels there. Which of the following neural structures is most likely damaged as well? (A) Musculocutaneous nerve (B) Lateral cord of the brachial plexus (C) Radial nerve (D) Axillary nerve (E) Medial cutaneous nerve of the arm

*The answer is A.* The radiograph shows a fracture of the surgical neck of the humerus. The posterior humeral circumflex artery, accompanied by the axillary nerve, lies against the posterior aspect of the surgical neck as it passes into the quadrangular space of the shoulder. This fracture places both of these structures in immediate danger. Choice B (Brachial artery) is incorrect. The brachial artery travels down the midline of the arm, close to the median nerve. It is not immediately endangered by the fracture of the surgical neck. Choice C (Deep brachial, or profunda brachii, artery) is incorrect. The deep brachial artery wraps tightly around the midshaft of the humerus, in the radial (spiral) groove. It, along with the radial nerve with which it travels, would be endangered by a fracture through the radial groove of the humerus. Choice D (Subscapular artery) is incorrect. The subscapular artery is the largest branch of the axillary artery. It descends along the axillary border of the scapula and is not in contact with the humerus. Choice E (Superior ulnar collateral artery) is incorrect. This artery is a branch of the brachial artery. It descends through the arm, moves into company with the ulnar nerve, and takes a close relation to the posterior aspect of the medial epicondyle of the humerus. This vessel would be endangered by a fracture of the medial epicondyle rather than a surgical neck fracture.

The given X-ray reveals a fracture of the proximal humerus, indicated by the black arrow. Given the location of the fracture, what artery is most likely damaged in this patient? (A) Posterior circumflex humeral artery (B) Brachial artery (C) Deep brachial (profunda brachii) artery (D) Subscapular artery (E) Superior ulnar collateral artery

*The answer is D.* It is important to recognize where neurovascular structures have close positional relations to each other and to underlying bony struc- tures in order to predict the likely second-order functional consequences of damage to the bones. In the given AP X-ray, the midshaft of the humerus is fractured slightly distal to the radial groove. At this point, the profunda brachii vessels (deep vessels of the arm) and the radial nerve emerge from the radial groove in a bundle tightly wrapped against the body of the humerus. A fracture here may readily damage any of these neurovascular structures. Lesion of the vessels may produce swelling in the posterior compartment of the arm and loss of supply to the muscles therein. Lesion of the nerve will result in major motor and sensory deficits in the posterior aspect of the forearm and hand. Choice A (Posterior circumflex humeral artery) is incorrect. This vessel travels in companionship with the axillary nerve around the surgical neck of the humerus. Fractures here are the most common injuries to the proximal end of the humerus, especially in the elderly. Choice B (Ulnar nerve) is incorrect. In the arm, the ulnar nerve travels with the superior ulnar collateral artery. Both lie in contact with the posterior side of the large, projecting medial epicondyle of the humerus as they cross the elbow. Choice C (Axillary nerve) is incorrect. The axillary nerve travels in companionship with the posterior circumflex humeral artery around the surgical neck of the humerus. Fractures here are the most common injuries to the proximal end of the humerus, especially in the elderly. Choice E (Median nerve) is incorrect. This large nerve travels with the brachial vessels down the anterior midline of the arm. The nerve lies close to the distal end of the humerus, where it may be damaged due to fractures of the condyle.

The given anteroposterior (AP) X-ray depicts a humeral shaft fracture in a 22-year-old man. Given the location of the fracture, which of the following structures is most likely damaged? (A) Posterior circumflex humeral artery (B) Ulnar nerve (C) Axillary nerve (D) Profunda brachii artery (E) Median nerve

*The answer is B.* The brachial plexus is divided into five geographic parts: Roots (or ventral rami of C5-T1), Trunks, Divisions, Cords, terminal Branches (or Nerves). These sections of the brachial plexus can be remem- bered with the mnemonics, "Real Truckers Drink Cold Beer" or "Remember Those Darn Cervical Nerves". The cords are named according to their important positional relationship to the second part of the axillary artery, deep to the pectoralis minor muscle. Here, the nerves form a cradle-like bed for this segment of the vessel as it passes through the axilla. Thus, the lateral cord is located lateral, the medial cord is medial, and the posterior cord is posterior to the axillary artery. Choice A (Long head of the biceps brachii muscle) is incor- rect. The long head of the biceps brachii muscle lies lateral to all three cords of the brachial plexus. Choice C (Subclavian vein) is incorrect. This vessel is located well proximal to the cords of the brachial plexus, medial to the first rib. Choice D (Surgical neck of the humerus) is incorrect. This part of the humerus is located distal and lateral to all three cords of the brachial plexus. Choice E (Pectoralis minor muscle) is incorrect. The pectoralis minor lies superficial (anterior) to the cords of the brachial plexus. This muscle divides the axillary artery into its three parts. Thus, it defines the second part of the artery and creates the situation for naming the cords of the plexus.

The lateral cord of the brachial plexus is named because it lies immediately lateral to which of the following structures? (A) Long head of the biceps brachii muscle (B) Axillary artery (C) Subclavian vein (D) Surgical neck of the humerus (E) Pectoralis minor muscle

*The answer is B.* The lateral thoracic wall receives significant arterial supply from the branches of the axillary artery. The axillary artery is divided into three parts by the overlying pectoralis minor muscle. The lateral thoracic artery, along with the thoracoacromial trunk, typically branches from the second part of the axillary artery, deep to the pectoralis minor. It descends along the lateral border of the pectoralis minor to supply the lateral aspect of the chest wall, including much of the breast and the serra- tus anterior muscle. However, this vessel is variable and may originate from other source points. Remember, arteries are often named for the regions they supply, not necessarily for their branching patterns. Choice A (First part of the axillary artery) is incorrect. This segment typically gives rise to one branch: the superior (or supreme) thoracic artery, which has a small distribution to the superolateral chest wall. Choice C (Third part of the axillary artery) is incorrect. This part typi- cally gives rise to three branches: the subscapular artery and the anterior and posterior circumflex humeral arteries. The lateral thoracic and subscapular arteries may originate as a common trunk. Choice D (Third part of the subclavian artery) is incorrect. The subclavian artery is divided into three parts by the anterior scalene muscle. Usually, the third part of the subclavian artery has no branches; however, the dorsal scapular artery may occasionally arise from this location. More importantly, none of the branches of the subclavian artery supply the lateral chest wall. Choice E (First part of the brachial artery) is incorrect. The distribution of the brachial artery is limited to the upper limb; it does not supply the lateral chest wall.

The lateral thoracic artery provides the main blood supply to the lateral side of the chest wall, including much of the breast. To deter excessive blood loss during a surgical procedure involving the breast, a surgeon can clamp the lateral thoracic artery near its origin. Which of the following arteries gives rise to this artery? (A) First part of the axillary artery (B) Second part of the axillary artery (C) Third part of the axillary artery (D) Third part of the subclavian artery (E) First part of the brachial artery

*The answer is E.* The pectoralis minor muscle overlies the axillary artery in such a way as to divide it into three parts: first (prepectoral; medial), second (subpectoral; deep), third (postpectoral; lateral). The first part is medial to the pectoralis minor, running from the lateral border of the first rib to the medial border of the pectoralis minor. The second part is deep to the muscle. The third part is lateral to the pectoralis minor, running from the lateral border of the muscle to the inferior border of the teres major muscle. Choice A (The lateral cord of the brachial plexus lies lateral to the muscle) is incorrect. The cords of the brachial plexus are so named by their positions relative to the second part of the axillary artery. Choice B (The clavipectoral triangle lies lateral to the muscle) is incorrect. The clavipectoral triangle lies medial to the pectoralis minor, between the muscle and the clavicle. This fascia-roofed space is noteworthy because it is pierced by the lateral pectoral nerve (on its way to the clavicular head of the pectoralis major), branches of the thoracoacromial artery, and the cephalic vein (on its way to empty into the axillary vein). Choice C (The anterior axillary lymph nodes lie along the medial border of the muscle) is incorrect. The anterior (pectoral; level 1) group of axillary lymph nodes lie along the lateral border of the pec- toralis minor. Choice D (The lateral wall of the axillary fossa includes the muscle) is incorrect. The pectoralis minor forms the anterior wall of the axillary fossa, along with the pectoralis major.

The pectoralis minor muscle is an important landmark in identifying and describing neighboring structures in the chest and axillary regions. Which of the following relationships of the pectoralis minor is correct? (A) The lateral cord of the brachial plexus lies lateral to the muscle (B) The clavipectoral triangle lies lateral to the muscle (C) The anterior axillary lymph nodes lie along the medial border of the muscle (D) The lateral wall of the axillary fossa includes the muscle (E) The second part of the axillary artery lies deep to the muscle

*The answer is C.* The most common location for measuring pulse rate is on the radial artery at the wrist. Here, the vessel lies on the anterior side of the distal end of the radius, lateral to the tendon of the flexor carpi radialis. It is covered only by skin and a thin superficial fascia and can be palpated easily against the radius. Note the placement of the index and middle fingers of the physician in the given photo. Choice A (Across the anterior aspect of the lateral epicondyle of the humerus) is incorrect. The radial artery originates in the cubital fossa as one of the two terminal branches of the bra- chial artery (the ulnar artery is the other). The radial recur- rent artery branches off the radial artery just below its origin and ascends across the anterior aspect of the lateral epicondyle of the humerus. However, the radial recurrent artery lies on muscle and is not normally palpable. Choice B (Between the tendons of the palmaris longus and flexor carpi ulnaris) is incorrect. The ulnar artery enters the hand superficial to the flexor retinaculum, lateral to the pisiform and medial to the hook of the hamate. The ulnar pulse may be palpable slightly lateral to the insertion of the flexor carpi ulnaris onto the pisiform, between it and the tendon of the palmaris longus. Choice D (Superficial to the tendons of the extensor pollicis brevis and abductor pollicis longus) is incorrect. The radial pulse is also available in the anatomical snuffbox, where the radial artery crosses the floor of that space between the ten- dons of the extensor pollicis longus and brevis muscles. In entering the snuffbox, the artery passes deep (not superficial) to the tendons of the abductor pollicis longus and extensor pollicis brevis muscles. Choice E (Superficial to the carpal tunnel) is incorrect. As noted above, the ulnar artery enters the hand superficial to the flexor retinaculum, thus, super- ficial to the carpal tunnel. The superficial palmar branch of the radial artery usually runs through the thenar muscles and is not palpable.

The pulse of the radial artery is readily palpable where the vessel passes which of the following structures? (A) Across the anterior aspect of the lateral epicondyle of the humerus (B) Between the tendons of the palmaris longus and flexor carpi ulnaris (C) Lateral to the tendon of the flexor carpi radialis (D) Superficial to the tendons of the extensor pollicis brevis and abductor pollicis longus (E) Superficial to the carpal tunnel

*The answer is A.* In a traditional (Halsted) radical mastectomy, both the pectoralis major and minor muscles are removed with the breast and associated axillary tissue. The primary actions of the pectoralis major are adduction, flexion, and medial rotation of the arm. These actions are weakened but not lost postoperatively. Other neighboring muscles (e.g., subscapularis, latissimus dorsi, biceps brachii, anterior deltoid) also perform these actions and can compensate somewhat for loss of the pectoralis major. Choice B (Abduction of the arm) is incorrect. Abduction of the arm is performed by the supraspinatus, deltoid, and serratus ante- rior muscles. Removing the pectoralis major would not affect abduction of the arm. In fact, abduction, extension, and lateral rotation of the arm are the antagonistic actions to the pecto- ralis major. Choice C (Extension of the arm) is incorrect. The pectoralis major helps with flexion of the arm, not extension (the antagonistic action). The latissimus dorsi and posterior part of the deltoid help with extension of the arm. Choice D (Lateral rotation of the arm) is incorrect. The pectoralis major helps with medial rotation of the arm, not lateral rotation of the arm (the antagonistic action). The infraspinatus, teres minor, and posterior part of the deltoid help with lateral rotation of the arm. Choice E (Depression of the arm) is incorrect. The pectoralis major muscle acts to adduct, flex, and medial rotate the arm. Therefore, a radical mastectomy would affect these actions.

The traditional radical mastectomy includes removal of the pectoralis major muscle. Which of the following movements is most affected postoperatively by this surgical procedure? (A) Adduction of the arm (B) Abduction of the arm (C) Extension of the arm (D) Lateral rotation of the arm (E) Depression of the arm

*The answer is D.* The recurrent branch of the median nerve usually originates from the lateral side of the median nerve at the distal margin of the flexor retinaculum. It innervates the three thenar muscles: the opponens pollicis, flexor pollicis brevis, and abductor pollicis brevis muscles. Injury of the median nerve in the carpal tunnel, as well as injury of the recurrent and superficial branch of the median nerve, causes both sensory and motor deficits. Injury of the superficial palmar branch of the median nerve results in loss of sensation only of the palm. Injury to the radial and ulnar nerves results in a greater number of sensory and motor deficits in the distribution of theses nerves.

While using a wood-carving gouge, a 34-year-old woman lacerates the proximal aspect of her palm from the base of the thumb across to the pisiform bone. Neurological examination reveals pronounced weakness in opposition of the thumb, with intact sensation in the hand. Which of the following injuries best accounts for her findings? A. Injury of the median nerve in the carpal tunnel B. Injury of the superficial palmar branch of the median nerve C. Injury of the recurrent and superficial branch of the median nerve D. Injury of the recurrent median nerve at the wrist E. Injury of the radial and ulnar nerves

*The answer is C.* A midshaft humeral fracture can result in injury to the radial nerve and deep brachial artery because they lie in the spiral groove located in the midshaft. Injury to the median nerve and brachial artery can be caused by a supracondylar fracture that occurs by falling on an outstretched hand and partially flexed elbow. A fracture of the surgical neck of the humerus can injure the axillary nerve and posterior circumflex humeral artery. The suprascapular artery and nerve can be injured in a shoulder dislocation. The long thoracic nerve and lateral thoracic artery may be damaged during a mastectomy procedure.

While walking to his classroom building, a first-year medical student slipped on the wet pavement and fell against the curb, injuring his right arm. Radiographic images showed a midshaft fracture of the humerus. Which pair of structures was most likely injured at the fracture site? A. Median nerve and brachial artery B. Axillary nerve and posterior circumflex humeral artery C. Radial nerve and deep brachial artery D. Suprascapular nerve and artery E. Long thoracic nerve and lateral thoracic artery

*The answer is B.* The radial pulse is best located on the forearm (antebrachium) just proximal to the wrist joint. At this point the radial artery travels on the distal radius between the flexor carpi radialis and brachioradialis tendons. The palmaris longus tendon travels more medially to the radial artery and above the flexor retinaculum. The flexor pollicis longus tendon is a deeper structure in the antebrachium and is also located medially to the radial artery.

While working out with weights, a 28-year-old woman experiences a severe pain in her chest. The pain is referred to the anterior chest wall, radiating to the mandible and her left arm. The woman felt dizzy and after 10 minutes she collapsed and was unconscious. A physician happened to be near the woman and immediately tried to feel her radial pulse. The radial artery lies between two tendons near the wrist, which are useful landmarks. Which of the following is the correct pair of tendons? A. Flexor carpi radialis and palmaris longus B. Flexor carpi radialis and brachioradialis C. Brachioradialis and flexor pollicis longus D. Flexor pollicis longus and flexor digitorum superficialis E. Flexor pollicis longus and flexor digitorum profundus


संबंधित स्टडी सेट्स

Psych 211 Exam #3 (Study questions)

View Set

Test 2 Practice questions: Heme and Immune

View Set

Chapter 22: The Lymphatic System and Immunity

View Set

Fundamentals of Nursing Practice Questions

View Set

Lit Final Study Guide (7th Grade)

View Set

BYU Fitness For Living Unit 1, BYU Fitness for Living Unit 2, BYU Fitness for Living Unit 3, BYU Fitness for Living Unit 4, BYU Fitness for Living Unit 6, BYU Fitness for Living Unit 7

View Set

PrepU Management of Patients with Chest and Lower Respiratory Tract Disorders

View Set

BCOR350 Ch 12 Marketing Channels

View Set